Professional Responsibility MPRE Outline

Professional Responsibility MPRE Outline INTRODUCTION

    1. The Role of Judgment in Law
      1. Judgment. Lawyers solve problems by exercising judgment. Judgment is the ability to evaluate all aspects of a problem in both practical and logical terms.
      2. Three features of the concept of judgment. It is:
        1. Dynamic – Lawyers must think ahead – you have think over time. Problems result because people react or selfishly act.
        2. Interactive – A lawyer must evaluate what is best given what you expect the other parties to do in response to your choice and in response to each other’s choices.
          1. Who all the players are and know what they are after. Look at how people relate to each other. Look into player’s interest. Draw diagram of situation they are in and a plausible exam answer.
        3. Probabilistic – Lawyers must determine how much uncertainty to live with
      3. MR 3.3 – You cannot lie to the court – In an ex part hearing the lawyer shall inform the tribunal of all material facts known to the lawyer that will enable the tribunal to make an informed decision, whether or not the facts are adverse.
      4. What do you do your superior is lying about something you said in front of court
        1. 3.3(a) and (e) – give judges facts – if partner lies about you, you need to affirmatively say they are lying about you.
        2. Mistakes were made –I am really sorry, I must not have been clear – she did not say that. Objective is to get out of here with my reputation and job intact. Gives the judge a way to save face because he is also worried about his own reputation.
        3. Take it upon yourself and signal that you are going to fix the record. Make it as easy as humanly possible to for the lawyer to do the right thing

 

    1. Five sources of potential trouble for a lawyer:
      1. General Legal Rules. Lawyers are for the most part subject to the general legal rules that everyone else has to obey.
      2. Rules creating duties to your client. (Your Client). The client is the person you represent and to whom you owe duties: The purpose of the duties is to vindicate the trust clients are presumed to place in lawyers.
        1. Duty of loyalty. This duty requires that you place your client’s interests ahead of your own.
        2. Duty of care. This duty requires that you perform your work competently.
        3. Duty of confidentiality. Requires that you not disclose to others or use for your own benefit confidential information you learn while representing a client.
      3. Rules creating duties to non-clients (Related Parties). Related parties are people your client deals with and whom you run across while representing your client. You can owe duties to non-clients and those duties can lead to liability or professional discipline.
      4. Rules of Professional Conduct (Disciplinary Rules). Rules of professional conduct are the formal rules adopted by states to regulate lawyers. Violation of these rules creates a basis for administrative discipline of lawyers. Model rules, not actual rules, states individually base their own set of ruls on the model rules
      5. Government Institutions. Courts can sanction lawyers in some cases. Agencies may also discipline lawyers.

 

    1. Six Rules of Survival
      1. Never create a duty you don’t want to create.
      2. Always be prepared to walk away. (Whether by (a) quitting or (b) being fired).
      3. Assume everything you do or say will become publicly known.
      4. Never mistake the client’s problem for your own.
      5. Never do as a lawyer anything you find repugnant as a person.
      6. If you mess up, fess up.

 

 

  1. CHAPTER 1 – DUTIES LAWYER OWN TO CLIENTS
    1. Lawyers are Fiduciaries and Agents
      1. Agency Law. Fiduciary duties are related to agency law. Agency is a type of fiduciary relationship and agency law is the foundation for many rules that specifically apply to lawyers.
      2. Definition of Fiduciary Duty. (Burdett v. Miller (1992)). A fiduciary duty is the duty of an agent (lawyer) to treat his principal (client) with the utmost candor, rectitude, care, loyalty, and good faith (to treat the principal as well as the agent would treat himself).
      3. The duty is owed when the disparity in knowledge or power is so vast that it is a reasonable inference that had the parties negotiated in advance, they would have expressly agreed that the agent owed the principal the duty (otherwise the principal would be at the agent’s mercy).

 

    1. Duty of Loyalty True fiduciary duty
      1. Owen v. Pringle (1993). Fiduciary duties may be summarized under the general rubric of the duty of loyalty.
      2. Restatement 16(3) – put the clients interest first and do nothing to harm them. – Lawyer must Comply with obligations concerning the client’s confidences and property, avoid impermissible conflicting interests, deal honestly with the client, and not employ advantages arising from the client-lawyer relationship in a manner adverse to the client
      3. Restatement § 8.01. An agent has a fiduciary duty to act loyally for the principal’s benefit in all matters connected with the agency relationship. Agents may not
        1. Acquire a material benefit from a third party in connection with the agent’s actions as an agent (§ 8.02);
        2. Take a position adverse to principal or on behalf of a party adverse to principal, regarding a matter related to scope of agency (§ 8.03);
        3. While an agent, compete with principal/assist principal’s competitors (though can prepare to compete with principal) (§ 8.04);
        4. Use principal’s property, or either use or communicate principal’s confidential information for benefit of agent or a third party (§ 8.05)
        5. Engage in conduct that is likely to damage the principal’s enterprise (§ 8.10).
        6. Fiduciaries must also segregate principal’s property form their own, keep/render an account of money/property received/paid (§ 8.12).
      4. Concern with conflicts of interest
        1. MR 1.7 – Conflict of interests – Current clients
        2. MR 1.8 – Conflict of interests – Current clients – specific rules
        3. MR 1.9 – Former Clients
      5. Ex) Daniel Bibb – Always be prepared to walk away. Lawyer (prosecutor) threw the case (lost on purpose) against a man he knew to be innocent. This violated the duty of loyalty. While a lawyer is entitled to his conscience, that does not entitle him to subvert his client’s (the government’s) claim. He should have withdrawn or quit, not thrown case!

 

    1. Duty of Care – Not being negligent
      1. The duty of care requires that lawyers act carefully—as judged by the prevailing standards of professional competence in the relevant field of law and geographic region—in performing work for clients.
      2. MR 1.1 – must act carefully
      3. Restatement 16(2)act with reasonable competence and diligence
      4. Restatement 52(1) a lawyer who owes a duty of care must exercise the competence and diligence normally exercised by lawyers in similar circumstances
      5. Agents must:
        1. Comply with the express and implied terms of any contract with the principal (§ 8.07);
        2. Act only w/in scope of actual authority, and comply w/ all lawful instructions from principal regarding agent’s actions him (§ 8.09);
        3. Inform principal of facts material to agency relation and all facts agent knows/has reason to know principal would want to have (§ 8.11).

 

    1. Duty of Confidentiality
      1. MR 1.6You may not use to detriment or disclose confidential information you obtain during the scope of assistance
        1. Exception: allows for disclosure only when a lawyer reasonably believes a client intends to commit an act reasonably certain to result in injury or death
        2. Exception: Generally known
      2. MR 1.9 – Duties to Former Clients – concern with conflict of interest
      3. Restatement § 8.05(2)): An agent has a duty “not to use or communicate confidential information of the principal for the agent’s own purposes or those of a third party.” However, the comment to this section says that this duty is not absolute.
        1. Exception: An agent may reveal information to protect a superior interest of the agent or third party (like if the principal is going to commit a crime that will harm a third party).
      4. Disqualification motions – conflict of interest rules. Different standard than liability. Disqualified if two cases are similar even if you do not have any information. But not tort liable if you don’t have confidential information.
      5. Different from the attorney client privilege – MUCH BROADER THAN ATTORNEY-CLIENT PRIVILEDGE
        1. The duty to maintain client confidences is broader; it prohibits lawyers from using the information for the lawyer’s benefit as well as disclosing the information
      6. Ex) Attorney found out that their client was guilty of an unrelated murder for which B had been charged. They could not tell authorities about their clients guilt until after death due to the duty of confidentiality.
    2. Reasons for making lawyers agents and fiduciaries for their clients:
      1. Eliminates contractual costs by providing default rules
      2. Protect client from opportunistic behavior by lawyer
      3. Some reasons are utilitarian: when lawyer is agent, sometimes law of agency imputes to client the consequences of lawyer’s acts/omissions. This protects third parties and gives lawyers a reason to act carefully. In other cases, it is just protection: lawyer may take advantage of the client or harm third parties.
      4. Sometimes, when lawyer is a fiduciary the normal burdens of production or persuasion are altered, making it easier for a plaintiff to prevail.

 

    1. An Overview of the Implications of Care, Loyalty, and Confidentiality: – CLIENT IS BOUND BY LAWYER’S ACT
      1. United States v. 7108 West Grand Avenue, 15 F.3d 632 (7th Cir. 1994)
        1. Clients suffered because their lawyer had the power to act for them but failed to act.  Because the lawyer had the power, the court attributed the lawyer’s failure to the client. 
        2. The principal has to suffer the good and bad that come with the attorney’s actions. Client must suffer
        3. Remedy – Sue lawyer for malpractice – We bind you to the actions of your agent because it is more efficient to do that and you can recover from agent if there has been malpractice – breach of duty of care. Harming client in court means that she will have a cause of action for malpractice
        4. Reasoning
          1. Holding the client responsible for the lawyer’s deeds ensures that both clients and lawyers take care to comply. Principal is bound by the acts of the agent.  The errors and misconduct of an agent redound to the detriment of the principal (and ultimately, through malpractice litigation, of the agent himself) rather than of the adversary in litigation.
      2. Tante v. Herring, 453 S.E.2d 694 (Ga. 1994) – Breach of duty of loyalty and disciplinary rule – not duty of care.
        1. Attorney used confidential information shared by client to sleep with her. No breach of duty of care because he won the case.
        2. By using information available to him solely because of the attorney client relationship to his advantage and to the Herring’s disadvantage, he breached that fiduciary duty. There was Misuse of the client’s information for the lawyer’s own benefit
        3. As a fiduciary, an attorney owes his client the utmost good faith and loyalty. Part of the utmost good faith and loyalty means that the attorney will not use the confidential information to his advantage/his client’s disadvantage.
        4. The breach of the duty of confidentiality (and loyalty).
      3. Barbara A. v. John G., 145 Cal. App. 3d 369 (1983) (Barry Deal)
        1. Lawyer is obligated to tell client the truth in personal matters too. Court views lawyer as smart, powerful, sophisticated parties even if they are not.
        2. The essence of a fiduciary or confidential relationship is that the parties do not deal on equal terms, because the person in whom trust and confidence is reposed and who accepts that trust and confidence is in a superior position to exert unique influence over the dependent party.
        3. Finding of duty – presumed that the one in whom trust and confidence is reposed has exerted undue influence and shift burden on to lawyer to show consent or unjustified reliance.
        4. MR 1.8(j) provides that a lawyer “shall not have sexual relations with a client unless a consensual sexual relationship existed between them when the lawyer-client relationship commenced.”
        5. Comment 17 to this rule says: The relationship between a lawyer and a client is a fiduciary one in which the lawyer occupies the highest position of trust and confidence. The relationship is almost always unequal; thus a sexual relationship between lawyer and client can involve unfair exploitation of the fiduciary role, in violation of the lawyer’s basic ethical obligation not to use the trust of the client to the client’s advantage.
  1. CHAPTER 2 – DIVISION OF AUTHORITY BETWEEN LAWYERS AND CLIENTS
    1. Authority
      1. Authority – Power to alter relations of the principal that the principal gives to the agent
      2. Apparent authority – based on what appears to a third person. When a third party reasonably believes that the actor has authority to act on behalf of the principle (Client) and that belief is traceable to the client’s manifestations.

 

Type of Authority

Created by

Grants power to

Authority – Restatement 26 Assent of client, manifested to lawyer – think in contract terms or ratifies the act. Alter client’s legal rights and obligations within bounds of client assent
Implied Authority (Authority) Implication from assent of client, manifested to lawyer Do things necessary to carry out client instruction (think necessary & proper cl.)
Apparent Authority – Restatement 27 Manifestations by client to third party that lawyer has authority Alter client’s legal rights and obligations within bounds of manifestation
Inherent Agency Power Appearance in court proceeding Alter client’s rights and obligations as needed to enforce result of proceeding

 

      1. Restatement §§26A Lawyer’s Actual Authority
        1. A lawyer’s act is considered to be that of a client in proceedings before a tribunal or in dealings with third persons when:
          1. The client has expressly or impliedly authorized the act;
          2. Authority concerning the act is reserved to the lawyer as stated in § 23; or
          3. The client ratifies the act.
      2. Restatement §§27A Lawyer’s Apparent Authority
        1. A lawyer’s act is considered to be that of the client in proceedings before a tribunal or in dealings with a third person if the tribunal or third person reasonably assumes that the lawyer is authorized to do the act on the basis of the client’s (and not the lawyer’s) manifestations of such authorization.

 

    1. Client Calls
      1. Introduction
        1. Client – ends / resolution. Lawyer shall abide by the client’s decisions with respect to ends (plead, settle, etc.)
        2. Lawyer – means, strategy, ways to accomplish ends
        3. Model Rule of Professional Conduct 1.2(a)SCOPE OF REPRESENTATION AND ALLOCATION OF AUTHORITY BETWEEN CLIENT AND LAWYER – A lawyer may take such action on behalf of the client as is impliedly authorized to carry out the representation. A lawyer shall abide by a client’s decision whether to settle a matter. In a criminal case, the lawyer shall abide by the client’s decision, after consultation with the lawyer, as to a plea to be entered, whether to waive jury trial and whether the client will testify.
        4. Model Rule of Professional Conduct 1.4 – COMMUNICATION – KEEP CLIENT INFORMED

(a) A lawyer shall:

            1. (1) Promptly inform the client of any decision or circumstance with respect to which the client’s informed consent, as defined in Rule 1.0(e), is required by these Rules;
            2. (2) Reasonably consult with the client about the means by which the client’s objectives are to be accomplished
            3. (3) Keep the client reasonably informed about the status of the matter;
            4. (4) Promptly comply with reasonable requests for information; and
            5. (5) Consult with the client about any relevant limitation on the lawyer’s conduct when the lawyer knows that the client expects assistance not permitted by the Rules of Professional Conduct or other law.
          1. (b) A lawyer shall explain a matter to the extent reasonably necessary to permit the client to make informed decisions regarding the representation.
        1. Model Rule of Professional Conduct 1.8(g) – A lawyer who represents two or more clients shall not participate in making an aggregate settlement of the claims of or against the clients, or in a criminal case an aggregated agreement as to guilty or nolo contendere pleas, unless each client gives informed consent, in a writing signed by the client. The lawyer’s disclosure shall include the existence and nature of all the claims or pleas involved and of the participation of each person in the settlement.

 

      1. Authority to Settle Civil Matters
        1. In re: Grievance Proceeding, 171 F.Supp.2d 181 (D. Conn. 2001)
          1. Lawyer’s standard fee agreement/contract included a clause which said he gets to decide whether or not the case would settle.
          2. Holding – Cannot get client to turn over settlement authority to attorney. A lawyer must communicate all settlement offers to the client and the client must decide whether or not to accept the offer. Violated 1.2(a)
          3. Lawyer was not penalized because he did not enforced and stopped using when he found out it was not ok. ABA sending message that they are keeping an eye on lawyers.
          4. You cannot reverse the default allocation of authority

 

        1. Diversification, Risk Tolerances, and Client ConflictsA lawyer is usually willing to tolerate a greater risk of low verdict than the client is because lawyer is diversified; the client, however, is undiversified for the most part.

 

        1. Lawyers have adopted various tactics to try to regain some of the control Rule 1.2 denies them
          1. Assigning AroundCan a client assign to a lawyer a right to claim attorneys’ fees? The court in Pony v. County of Los Angeles (9th Cir. 2006) said NO. The right to seek attorney’s fees is a substantive cause of action which cannot be transferred contractually.
          2. A settlement refusal tax? Under 1.2(a) a lawyer cannot write a contingent fee contract that’s says that the lawyer will cover litigation expenses unless the client unreasonably rejected a settlement offer, in which case the client would be liable for such expenses; or a provision holding the client liable only for expenses incurred after rejecting the offer. Lawyers may not include a provision in a written contingent fee agreement requiring a client to pay costs advanced towards litigation of the client rejects a settlement offer that appears to counsel to be fair and reasonable under the circumstances.
          3. Anti gag-rule provisions? An attorney may ethically include in an engagement agreement terms that include a reduction of the attorney’s fee as long as the client does not agree to keep any settlement confidential, provided the client retains the authority to settle the case without the lawyer’s consent and without the imposition of any penalty that would constitute an unconscionable fee.
          4. Majority rule settlement – Lawyer cannot ask clients in aggregate litigation to approve a retainer in which they consent to any settlement approved by a majority of clients?

 

        1. Clients can play that game, too – If bad faith – cannot dismiss claim
          1. A client refusing a settlement offer because she believes her claim is worth more and thinks the attorney has not advocated on her behalf is not acting in bad faith and the client has the right to terminate the attorney in such circumstances.
          2. Clients can use their approval power as leverage against lawyers too. A client may breach the covenant of good faith and fair dealing implied in every contract (and every retainer agreement).
          3. A client offered a settlement could not dismiss the claim so that her attorney would be deprived of attorney’s fees. Dweck Law Firm, LLP v. Mann (SDNY 2004).

 

        1. Prospect Theory and Valuation
          1. Neoclassical analysis presumes that people evaluate gains and losses in a particular way:
            1. They evaluate the amount of a gain or loss by itself, not by reference to something else;
            2. They value uncertain gains and losses by multiplying the nominal gain or loss by the probability that it will occur; and
            3. The resulting expected gains and losses are evaluated in a linear fashion, which is to say that people value the marginal gain or loss at the same rate as the initial gain or loss (i.e., the 1,000th dollar counts for $1, just like the first dollar).
          2. Prospect theory holds each of these assumptions is wrong. It takes into account several observations about how people evaluate gains and losses:
            1. They do so with respect to some reference point, rather than in purely nominal terms;
            2. They weigh losses more heavily than gains, and thus display “loss aversion”; and
            3. They show diminishing sensitivity to the size of gains or losses (the 1,000th dollar counts less to them than the first). As a result, loss curves are convex and gain curves are concave, rather than being linear.
          3. When it comes to clients and lawyers, reference points are different for each person. If a lawyer understands loss aversion, she may frame problems for clients or jurors in order to take advantage of or defuse that phenomenon.

 

      1. Apparent Authority to Settle Civil Matters
        1. Authority – manifestation by client. Authority is created by communications from the principal to the agent
        2. Apparent authority – representation to third party. Requires an act of volition by the client with respect to a third party. – Apparent authority is created by representation from the principal to a third party that wants to bind the client to actions taken by the lawyer.
        3. Fennell v. TLB Kent Co., 865 F.2d 498 (2d Cir. 1989
          1. Facts – Lawyer settled a matter for client over the phone for 10k. Lawyer on the other end assumed that layer has authority to settle because nothing has indicated there is no authority.
          2. Holding
            1. Lawyer did not have authority to settle. C did not give authority to lawyer and there is no apparent authority because C did not to indicate to the lawyer on the other end that he authorized settlement.
            2. If an attorney has apparent authority to settle a case and the opposing counsel has no reason to doubt that authority, the settlement will be upheld. (Note an agent cannot create apparent authority, only the principal).
            3. Essential to the creation of apparent authority are words or conduct of the principal, communicated to a third party, that give rise to the appearance and belief that the agent possesses authority to enter into a transaction.
            4. ie Client MUST communicate with the opposing counsel that there is authorization to settle

 

      1. Inherent Agency Power
        1. There is a third type of power, other than apparent authority and authority, that is inherent authority otherwise called “inherent agency power.”
        2. Inherent agency power is a term used to indicate the power of an agent which is derived not from authority, apparent authority or estoppel, but solely from the agency relation and exists for the protection of persons harmed by or dealing with a servant or other agent.
        3. The point of this power is to avoid harm to third parties by binding a principal (client) to an agent’s (lawyers) acts regardless of the consent or manifestations of the principal.
        4. There are very few cases where courts will find inherent agency power for lawyers. Inherent authority only comes into play when there is no actual authority.
        5. This concept requires the distinction between general and special agents.
          1. General agent – agent authorized to conduct a series of transactions involving a continuity of service. (like insurance agents and store clerks)
          2. Special agent – agent authorized to conduct a single transaction or a series of transactions not involving a continuity of service. (lawyers). Principal is not bound by the acts of the special agent, if he exceeds the limit of his authority. And it is the duty of every person who deals with a special agent to ascertain the extent of the agent’s authority before dealing with him. If this is neglected, such person will deal at his peril, and the principal will not be bound by any act which exceeds the particular authority given.

 

      1. Performative Utterances and the Practice of Law
        1. Because (i) lawyers practice through words, which can be deeds; (ii) the meaning of words depends on the context in which they are used; (iii) contexts are created by the intersection of facts, rules, norms and expectations; (iv) in order to exercise judgment about your own conduct you must be sensitive to the intersection you are in, and change it if you do not want to be there.
        2. If you do not want to operate in this context, you can change it by disclaiming authority to bind the client. Your disclaimer is a performative utterance—it performs the important task of letting everyone know where you stand, and where they stand in relation to you—but the task the words perform is to change the context rather than to accomplish a result within a particular context

 

      1. Blanton v. Womancare, Inc., 38 Cal. 3d 396 (1985)
        1. Facts: Malpractice suit over botched abortion, Lawyer agreed to binding arbitration despite what client wanted
        2. Holding
          1. This is a substantive right (an end). Lawyer cannot take that away from client with authority. Attorney acted not only without the client’s express authority but contrary to her express instructions.
          2. Attorney is not authorized to impair the client’s substantial rights or cause of action itself
          3. Absent express authority, an attorney does not have implied authority to enter into contracts on behalf of his client.
          4. Even if you are subordinate in the firm, and you work for another lawyer who is making the decisions, you still are the lawyer for the client and may be sued individually for malpractice
            1. You have to act as her lawyer even if your boss is telling you not to.
          5. Unauthorized acts may be binding if the client later ratifies the acts. In this case, there was no ratification and plaintiff immediately objected and fired her attorney.
            1. The court therefore reversed the judgment.

 

      1. Client Ratification
        1. The principle of ratification allows principals to accept the benefits of actions their agents have taken without authority or apparent authority.
        2. But a principal who ratifies an act must also accept the costs of that act.
        3. Ratification may also create an agency relationship after the fact. If A engaged in conduct benefiting B. B could ratify the conduct, retroactively making A and Agent for B.
        4. Requires Consent – Sole requirement for ratification is a manifestation of assent or other conduct indicative of consent by the principal.
        5. Ratification is not effective if the principal lacked knowledge of material facts to the transaction, unless the principal ratified knowing he lacked such knowledge.
        6. Also under the equal dignities rule, ratification of an action required to be in writing must itself be written.

 

      1. Where Does Trouble Come From?
        1. Doubling down is when an attorney realizes she has violated some duty or rule and, then decides to violate another rule or duty in the hope that the second violation will cover up the first. MESS UP, FESS UP
        2. Loss aversion makes some people unwilling to take risks that might lead to losses, this is called risk aversion.
        3. But loss aversion may also make people more willing to take risks to avoid losses they are otherwise uncertain to incur—this leads to doubling down.

 

    1. Lawyer Calls
      1. Criminal Matters
        1. Model Rules 1.2(c)SCOPE OF REPRESENTATION AND ALLOCATION OF AUTHORITY BETWEEN CLIENT AND LAWYER – In a criminal case, the lawyer shall abide by the client’s decision, after consultation with the lawyer, as to a plea to be entered, whether to waive jury trial and whether the client will testify.
          1. (c) A lawyer may limit the scope of the representation if the limitation is reasonable under the circumstances and the client gives informed consent.
        2. Model Rules 1.14CLIENT WITH DIMINISHED CAPACITY
          1. (a) When a client’s capacity to make adequately considered decisions in connection with a representation is diminished, whether because of minority, mental impairment or for some other reason, the lawyer shall, as far as reasonably possible, maintain a normal client-lawyer relationship with the client.
          2. (b) When the lawyer reasonably believes that the client has diminished capacity, is at risk of substantial physical, financial or other harm unless action is taken and cannot adequately act in the client’s own interest, the lawyer may take reasonably necessary protective action, including consulting with individuals or entities that have the ability to take action to protect the client and, in appropriate cases, seeking the appointment of a guardian ad litem, conservator or guardian.
          3. (c) Information relating to the representation of a client with diminished capacity is protected by Rule 1.6. When taking protective action pursuant to paragraph (b), the lawyer is impliedly authorized under Rule 1.6(a) to reveal information about the client, but only to the extent reasonably necessary to protect the client’s interests.

 

        1. Arko v. People, 183 P.3d 555 (Co. 2008) – whether to include less non-included defense = strategy = lawyer call
          1. Lawyer objected to include the less non-included offenses. Client demanded the lawyer do so.
          2. Judge decided client’s call and client was subsequently convicted and appeals claiming error that trial court wrongly gave the D what the D asked for
          3. Holding
            1. This should have been within the lawyers control and therefore conviction reversed.
            2. Means – strategic and tactical lawyers decision, because lawyers know how the system works. The system goes more smoothly if the lawyer get to make the calls.
            3. If more likely to affect third parties lawyer.
            4. Ends – like pleading guilty clients decision 

 

        1. United States v Kacysnski – 9th cir – 2001- Defending the Unabomber – mental insanity defense = strategic = lawyer call
          1. Unabomber did not want lawyers to use insanity defense. Lawyers did and trick him- lied to him, put on evidence that you are sane, expert comes in but actually to prove that he is insane.
          2. Holding
            1. This decision was within the strategic means controlled by lawyers. Lawyers can choose whether to put on mental impairment defense. Client does not have the power to decide whether or not his lawyers argue insanity.
            2. Model rule does not specify
            3. McGowan would be more hands off – more libertarian.

 

        1. Appointed Counsel and Client Control – LAWYERS CONTROL OVER FIRVOLOUS CLAMS AND APPEALS
          1. Model Rule 3.1 – Lawyers cannot make frivolous arguments.
          2. You have a duty to follow LAWFUL client instruction when they involve ends!
          3. The constitutional rules are lawyer-friendly.
          4. A client DOES NOT have a constitutional right to present frivolous arguments on appeal, and therefore has no right to have counsel present such arguments.
          5. Anders v. California (US 1967) – A lawyer could request to withdraw if she concluded an appeal would be wholly frivolous, wrote a brief stating that fact but also pointing to anything in the record that might arguably support an appeal, gave a copy of the brief to the client, and gave the client a chance to raise any points he wanted argued. The appellate court would then review this request to withdraw and dismiss or dispose of the appeal. If it found any arguable points, however the court would have to provide counsel to argue them.
          6. Smith v. Robbins (US 2000), the Court held that the Constitution permits but does not require the precise procedure the Court specified in Anders.
          7. Jones v. Barnes (US 1983) held that the constitution does not require appellate counsel to raise on appeal all non-frivolous arguments a client instructs a lawyer to make.

 

      1. Civil Litigation
        1. Lawyers get to make some decisions even if a client disagrees. The more a particular question affects the efficiency of courts, the more likely it is for lawyers to have the final say on that question. Questions of tactics are in the lawyer’s discretion.
        2. Where litigation costs are asymmetric, they can become a bargaining tool in and of themselves. Disciplinary rules make some attempt to limit the third-party effects of litigation.
          1. Model Rule 3.1 forbids lawyers from asserting frivolous claims.
          2. Model Rule 3.2 requires counsel to make reasonable efforts to expedite litigation.
          3. Model Rule 3.3 forbids lawyers lying to tribunals or from introducing false evidence the lawyer knows to be false.
          4. Model Rule 3.4 forbids lawyers from obstructing access to evidence, falsifying evidence, making frivolous discovery requests, or asking third parties to refrain from giving evidence.

 

    1. Unbundling – Agreement between a lawyer and client that the lawyer will provide some but not all services necessary to resolve the client’s problem. This method provides some level of representation to people who cannot afford to have a lawyer handle all aspects of some matter.
      1. Model Rule 1.2(c) allows lawyers to limit the scope of their representation of a client if the limitation is reasonable and if the client gives informed consent to it.
      2. Model Rule 1.1.- the Duty of Care require that the lawyer be able to provide competent service to the extent he or she undertakes to do so. It is conceivable that some proposed limitations might make competent representation impossible, in which case both disciplinary and tort rules should cause counsel to decline the arrangement.

 

 

  1. CHAPTER 3 – THE AUTHORITY OF CONFIDENTIALITY
    1. May not *USE* or DISCLOSE confidential information related to representation that is not generally known if there is reasonable prospect of client harm or clients says not to
      1. You can use information without disclosing it; like trading securities based on client confidences. The converse is generally not true: a lawyer who discloses client confidences generally does so to make use of them
      2. May disclose IF
        1. Doing so advanced client’s interest – Restatement of the Law Governing Lawyers §61
        2. The client consent – Restatement of the Law Governing Lawyers §62
        3. Disclosure if required by law – Restatement of the Law Governing Lawyers §63
        4. Disclosure is necessary to defend yourself or recover a fee – Restatement of the Law Governing Lawyers§ 64 and § 65
        5. Disclosure is reasonably necessary to prevent death or serious bodily harm after efforts to dissuade – Restatement of the Law Governing Lawyers§ 66
        6. Disclosure is necessary
      3. MR §1.6 –CONFIDENTIALITY OF INFORMATION – (a) A lawyer shall not reveal information relating to the representation of a client unless the client gives informed consent, the disclosure is impliedly authorized in order to carry out the representation or the disclosure is permitted by paragraph (b).
        1. (b) Exceptions
          1. Prevent reasonably certain death or bodily harm
          2. Prevent crime or fraud and reasonably certain to harm financial interest of third party to rectify harm from same cause – Restatement of the Law Governing Lawyers
          3. Seek advice about compliance with the rules
          4. Establish defense against allegations – Restatement of the Law Governing Lawyers
          5. Comply with law or court order – Compelled – Restatement of the Law Governing Lawyers
      4. MR §1.8 – Don’t use confidential information to client’s disadvantage
      5. MR §1.9(c)  Don’t use former client’s information against them unless it has become generally known

 

      1. The duty of confidentiality distinguished from the attorney-client privilege
        1. The basic rules governing this relationship hold: (i) counsel may neither use nor disclose confidential information in their own initiative; (ii) counsel must disclose confidential information if required to do so by law; unless (iii) the confidential information is also privileged, in which case counsel must assert the privilege against disclosure unless the client instructs otherwise.
        2. SUMMARY OF DUTY OF CONFIDENTIALITY
          1. Covers: all information relating to representation not generally known
          2. Applies when: at all times
          3. Effect: forbids voluntary disclosure (but not disclosure required by law) OR use of information that harms client or benefits lawyer outside the practice of law
          4. Controlled by: client
          5. Exceptions
            1. Prevent death or seriously bodily injury
            2. Prevent or rectify serious financial harm in which L’s services have been used
            3. Self defense – response to allegation of representation-related misconduct
            4. Seek advice about compliance with the rules
            5. Disclosure required by law
            6. NO CRIME FRAUD EXCEPTION
            7. Generally known and former client – 1.9(c)(1)).
            8. NOT IF GENERAL KNOWN AND CURRENT CLIENT – Rule 1.6,

 

 

Duty of Confidentiality Privilege
Covers All information relating to representation not generally known Confidential communications between attorney and client for the purpose of providing legal services
Applies At all times – arising out of duty of loyalty and care. Rules of evidence apply
Effect Forbids voluntary disclosure (but not disclosure required by law) or use of information that harms client or benefits lawyer outside the practice of law Prevents the kinds of disclosure evidence rules could otherwise compel
Controlled by Exceptions ClientNo Crime fraud exception Client Crime fraud exception

 

        1. Brennan’s, Inc. v. Brennan’s Restaurants, Inc., 590 F.2d 168 (5th Cir. 1979) – APPLIES EVEN WHEN PRIVILEDGE DOES NOT.
          1. Facts
            1. Wegmann was general attorney for family businesses.
            2. Two family groups had dispute over operation and management of the family businesses. The dispute was resolved by dividing the stock between the two opposing family groups. Wegmann continued to represent defendants and severed his connections with plaintiff and its shareholders..
            3. Wegmann retained an attorney to assist him in the defense of the case. Plaintiff moved to disqualify both attorneys on the ground that the representation was at offs with the interests of plaintiff, his former client.
          2. Holding – Disqualification
            1. Lawyer owes a duty of confidentiality to his former client even when the information at issue is not privilege. Lawyer cannot use that confidential information in prosecuting against P now.
            2. A lawyer should not use information acquired in the course of the representation of a client to the disadvantage of the client.
            3. This applies to all “information” not just “confidences.”

 

      1. USE of client information for personal benefit
        1. David Welch Co. v. Erskine & Tulley, 250 Cal.Rptr. 339 (1988)
          1. Facts
            1. Welsh had companies assign him bad debts and he established a system to collect on them. W hired law firm for help.
            2. Before undertaking the representation of Welch, neither E&T nor Carroll had experience in collection agency work for trust funds.
            3. After law firm and Welch terminated their attorney-client relationship, law firm gradually acquired the collection business activities formerly performed by Welch in behalf of several employee benefit trust funds.
            4. Defendants claimed = activity required no specialized knowledge
          2. Holding – Firm used confidence from prior representation as business knowledge
            1. Lawyer cannot directly compete with former client if providing the services requires the use of confidential information learned in the course of representation. Even though the company was sold, the duty owned to the company transferred to the new owner.
            2. Critical factor in finding breach of duty in this case is the fact that defendants in no way informed Welch that they were preparing proposals designed to undercut Welch’s business relationships.
            3. Due to the pre-existing attorney-client relationship during which defendants were in a position to and did obtain confidential information about Welch’s business, these defendants had a higher duty, which was to refrain from acquiring any pecuniary interest involving collection work for these trust funds unless they first notified and obtained the informed consent of Welch to submit their business proposals. They therefore breached their fiduciary duty towards Welch.

 

        1. United States v. O’Hagan, 541 U.S. 642 (1997)
            1. Facts
              1. O’Hagan was a partner at a law firm. They represented Grand Met regarding a potential tender offer for the common stock of Pillsbury.
              2. O’Hagan did not take part in the representation. The firm withdrew from representing the company and a month later they publicly announced the tender offer.
              3. While the firm still represented Grand Met, O’Hagan began purchasing call options = insider trading
            2. Holding – misuse of client information, without client’s consent, is still breached
              1. A fiduciary who pretends loyalty to the principal while secretly converting the principal’s information for personal gain, dupes or defrauds the principal.
              2. In Carpenter v. US, the court said that a company’s confidential information qualifies as property to which the company has a right of exclusive use. Misappropriation of that information constitutes fraud akin to embezzlement.
              3. Deception through nondisclosure is central to the theory of liability.
              4. You can use information without disclosing it – just as bad. Use or disclosure is prohibited.

 

      1. DISCLOSURE of Client Information for Personal Benefit
        1. Model Rule s 1.6(a)CONFIDENTIALITY OF INFORMATION (a) A lawyer shall not reveal information relating to the representation of a client unless the client gives informed consent, the disclosure is impliedly authorized in order to carry out the representation or the disclosure is permitted by paragraph (b).
        2. Model Rule 4.1 – TRUTHFULNESS IN STATEMENTS TO OTHERS – In the course of representing a client a lawyer shall not knowingly make a false statement of material fact or law to a third person;
        3. Model Rule 1.9(c) – DUTIES TO FORMER CLIENTS – A lawyer who has formerly represented a client in a matter or whose present or former firm has formerly represented a client in a matter shall not thereafter use information relating to the representation to the disadvantage of the former client except as these Rules would permit or require with respect to a client, or when the information has become generally known
        4. In re Wood, 634 A.2d 1340 (N.H.1993) – CANNOT DISCLOSE ANYTHING THAT IS COFIDENTIAL (NOT GENERALLY KNOWN)
          1. Facts
            1. Attorney represented a builder/developer who was putting a strip mall in lawyer’s community right next to his backyard.
            2. L withdraws because of conflict of interest
            3. Lawyer then gives interview to local paper about the development and say things which he only knew from his representation of client.
              1. Increases traffic, ambient light scares animals generally known information
              2. My firm would not represent their scum bags this is a lie!
              3. Willing to increase tax base 20 miles away from my house – disclosure of confidential information in order to avoid costs = obtain benefited
          2. Holding – violated 1.9(c) – using information to person’s disadvantage
            1. Violated 19(c) – A lawyer who has formerly represented a client in a matter or whose present or former firm has formerly represented a client in a matter shall not thereafter use information relating to the representation to the disadvantage of the former client except as these Rules would permit or require with respect to a client, or when the information has become generally known
            2. However some are information that Wood retained from representing them and used them to their disadvantage. These facts had not become generally known at the time the statements were revealed. Therefore, Wood violated rule 1.9(c).
            3. Not 1.9(a) – representing another client with adverse interest to a former client in the same or substantially related manner, since there was no other client.

 

      1. Disclosure not for personal benefit – no self serving – PERSONAL BENEFIT NOT REQUIRED
        1. Personal benefit is not required to show a breach of duty. Sometimes the violation can take the form of inadvertent or defensive disclosure.
        2. In re Pressly, 628 A.2d 927 (Vt. 1993)
          1. Facts
            1. Lawyer represented woman in connection with relief from abuse and divorce. Lawyers asks that client would retain custody and husband would get unsupervised representation.
            2. Client told lawyer that she thought father had abused 9-year-old daughter, based on a consultation with a counselor. Client asked lawyer not to discuss suspicions with husband’s lawyer.
            3. In response to a question asking whether sexual abuse was an issue in the case, lawyer revealed to opposing lawyer the client’s suspicions.
          2. Holding
            1. Lawyer disciplined for revealing confidential information to opposing counsel without consent of C. Not only was there no consent, this was explicitly what client asked attorney not to do.
            2. 4.1 says you cannot lie to opposing counsel, however, he did not have authority to disclose this piece of information. The lawyer must be truthful in dealing with others on client’s behalf, but there is no affirmative duty to inform an opposing party of relevant facts.
            3. Solutions – “We are not going to have this conversation now, set up a meeting with me.” – “if we make an argument, you won’t have to ask me about it, you’ll know.”

 

      1. Disclosure Authorized By Implication
        1. Restatement 61 – “a lawyer may use or disclose confidential client information when the lawyer reasonably believes that doing so will advance the interests of the client in representation.”
        2. The general rule of authority of section 21(3) says: a lawyer may take any lawful measure within the scope of representation that is reasonably calculated to advance a client’s objectives as defined by the client, consulting with the client as required by section 20.
        3. Clients consent > disclosing to advance the interest of the client representation.
          1. Because this authority is implied as a default rule, a client has the power to instruct you NOT to use or disclose confidential information, even if the rule stated in Section 61 would otherwise allow use or disclosure.
        4. Adams v. Franklin (D.C. 2007) – This case illustrates the principle of implied authorization in the context of the attorney-client relationship.
          1. Adams accused defendants of defrauding her into selling her property below market price. They used SOL as a defense. The privilege only protects communications from a client that are at the time they are communicated intended to be confidential. Because information was to be basis of demand letter, the intent was that it could not be confidential.
          2. You have to disclose any information that pertains to the issue that you are asserting in the defense. no just good but also stuff that’s bad for you.

 

      1. Confidentiality With Multiple Clients
        1. Privilege rules – joint clients have no privilege as between them as to subject matter about representation. But privilege against rest of the world
        2. Confidence Rule – Each joint client the full fiduciary duties – including confidentiality. You keep confidences one from the other and from the rest of the world. To be safe – “joint representation has certain implications and here they are, nothing you say is kept from the other “ (waive the duty as against each other). UNLESS WAIVER
        3. A v. B, 158 N.J. 51 – This case did not involve such a waiver. Confidences must be kept from one another.
          1. Firm representing H and W to make a will. Firm also represents W already who has illegitimate child.
          2. Here there are two duties
            1. Duty of confidentiality – 1.6(a) – husband
            2. Duty to inform clients of material facts – 1.4(b) – we need to keep them informed of material information – need to tell wife
            3. Firm can disclose under NJ RPC 1.6(c) which allows a lawyer to reveal confidential information to the extent the lawyer reasonably believes necessary to rectify the consequences of a client’s criminal, illegal, or fraudulent act in furtherance of which the lawyer’s services had been used.
            4. Two clients with mutually exclusive duties. In California, there is no way to get out.

 

      1. Entity Representation and Entity Constituents – LLP, Corporation, documents from an entity, etc.
        1. Model Rule 1.13(f)-(g) – YOUR CLIENT IS YOUR ENTITY – NOT THE HUMAN BEINGS INVOLVED IN IT
          1. If you work for an entity, such as a company, entity is default client. Person who hires you, the person who has power to fire you, etc. is not your client. The company is.
          2. If you choose to make an constituent (officer, director, employee of entitled) your client, subject to model rule 1.7 and conflict of interest, you cannot violate those. If you create an atty-client relationship with the individual employee, you cannot turn over his statement to prosecutors.
          3. If the employee misunderstand who you represent it is ATTY’S DUTY to clarify
        2. Perez v. Kirk & Kerrigan, 822 S.W.2d 261 (Tex. 1992) – Disclosure Case
          1. Facts
            1. Driver of a company car hits a school bus. Company hired lawyers and lawyers go talk to employee in the hospital. Lawyers told him that they were his lawyers too and that anything he said would be kept confidential. Lawyers of entity said they represented P, but later arranged for another lawyer.
            2. Lawyers call client entity and tell them what information they obtained from Perez.
            3. Perez sue attorneys for breach of fiduciary duty – confidentiality
          2. Holding
            1. Default Rules – You represent entity. You owe confidences in situations where you lead people to believe it is said in confidence. However this is entity’s privilege – entity can waive and usually ntity can turn over notes
            2. Attorneys were at least under a fiduciary duty not to misrepresent to Perez that his conversation with them was confidential.
            3. Should have said – “I am a lawyer, I represent the entity, I don’t represent you. I need to ask you some questions and conversation is privileged but the company holds the privilege. Thus mean entity can disclose if it thinks it should” – Upjohn Warning
            4. Grant client damages for emotional distress and mental anguish.

 

        1. Must They Ask or Must You Tell
          1. It is lawyer’s job to clarify the situation. The lawyer has the responsibility of preempting the formation of an atty-client privileging with the constituent where such a relationship is not intended.

 

    1. EXCEPTIONS TO THE DUTY – 1.6
      1. PERMITS disclosure to
        1. Prevent reasonably certain death or sustain bodily harm,
        2. prevent or rectify serious financial harm caused by a client and in which your services are used
        3. secure advice about your own obligations under the rules
        4. to establish a claim or defense in a dispute with a client
        5. to respond to allegations in any forum concerning your representation to comply with the law or court order
      2. Lawyers Self-Defense – Requires an allegation, not just a fear without accusations.
        1. CALIFORNIA DOES NOT HAVE THIS – does allow introduction of otherwise privileged communications pertaining to “an issue of breach, by the lawyer or by the client, of a duty arising out of the lawyer-client relationship
        2. Model Rule 1.6, CONFIDENTIALITY OF INFORMATION, comment [10] Where a legal claim or disciplinary charge alleges a complicity of the lawyer in a client’s conduct or other misconduct of the lawyer involving representation of the client, the lawyer may respond to the extent the lawyer reasonably believes necessary to establish a defense
        3. First Federal Savings & Loan v. Oppenheim, Appel, Dixon & Co.,110 F.R.D. 557 (S.D.N.Y. 1986)– SUIT DOES NOT HAVE TO HAVE ALREADY BEEN FILED – JUST ACCUSATION OF WRONG DOING.
          1. Client sues bankrupt company/lawyers. Lawyer discloses confidential information for his defense.
          2. Self Defense Exception – Can disclose is reasonably necessary for complaint or defense
            1. The principle has long been accepted that, in appropriate circumstances, an attorney may disregard the privilege of a current or former client, and disclose otherwise protected attorney-client communications.
            2. An attorney may make disclosure of confidences or secrets necessary to establish or collect his fee or to defend himself or his employees or associates against an accusation of wrongful conduct.
            3. This applies to when an attorney is being sued by the client or by someone other than the client, or when an accusation of misconduct has been leveled against an attorney, even if a suit has not been filed.
            4. However, the documents must be necessary to the defense and the attorney must disclose both favorable and unfavorable information.
        4. *3-6: If no charge or allegation, can you disclose confidences as self-defense exception? No
        5. Matter of Ponds (DC 2005 – IF NOT ACCUSATION – NO DEFENSE EXCEPTION
          1. Ponds represented criminal defendant. Client violated conditions of bail and warrant was out for his arrest. Client appeared at Pond’s office and demanded money from Ponds and said if Ponds didn’t give it to him, someone would get hurt.
          2. Ponds called the prosecutor and told him the facts (worried he would be implicated). The Bar found Ponds violated Rule 1.6. Though Ponds worried he might be implicated as an accomplice, no accusations had been made.

 

      1. Physical and Economic Harm – APPLIES IF LAWYER DOES NOT KNOW THE PERSON IS ALREADY DEAD.
        1. MR 1.6(b)(2) – FINANCIAL HARM – A lawyer may reveal information to the extent the lawyer reasonably believes necessary to prevent client from committing a crime or fraud that is reasonably certain to result in substantial injury to financial interests or property of another and in furtherance of which the clinet has used or is using the lawyer’s services.
        2. MR 1.6(b)(1) – PHYSICAL HARM – A lawyer may reveal information to the extent the lawyer reasonably believes necessary to prevent reasonably certain death or substantial bodily harm.
          1. It does not have to be a criminal act – that was the old rule
          2. It is permissive -it allows but does not require disclosure.
          3. You can disclose to prevent execution – because that is death, but no imprisonment
        3. McClure v. Thompson, 323 F.3d 1253 (9th Cir. 2003)
          1. Client told attorney where he would hide the two children. Attorney disclosed this information and the children were found, dead.
          2. The court found that the attorney’s disclosure was permitted under 1.6(b)(1) to prevent reasonably certain death or substantial bodily harm. Attorney was not sure if the children were alive or not so he reasonable believed revealing client information could prevent reasonably certain death or harm.
        4. Don’t Threaten the Judge – client left attorney angry messages threatening the judge and saying he was going to “raise hell.” As the lawyer saw client get more and more mad he finally withdrew and told judge about the message. Court held that he was permitted to do so under 1.6(b).
        5. No Matter Where You Go, There You Are II: The Dead Bodies Case – DOES NOT APPLY IF PERSON IS ALREADY DEAD
          1. Man killed three people and told his lawyers where the bodies were. The attorney’s did not disclose the locations of the dead bodies. They would not have been able to under the exception to the duty of confidentiality because they knew that the people were already dead.

 

      1. Confidentiality and Compelled Production – Unlike privilege, confidentiality is not a defense to compelled production. If a court determines that privilege does not protect some evidence, the duty of confidentiality does not protect it either.

 

  1. CHAPTER 4 – ATTORNEY CLIENT PRIVILEDGE
    1. Restatement §§68-72; 77; 86
    2. § 68. Attorney–Client PrivilegeExcept as otherwise provided in this Restatement, the attorney-client privilege may be invoked as provided in § 86 with respect to:
      1. When the privilege applies. The privilege applies only where rules of evidence apply and would otherwise compel testimony or production of documentary evidenc
      2. (1) a communication – NOT FACES
        1. The client cannot be compelled to answer that question, what did you say or write to the attorney? But may not refuse or disclose any relevant fact within his knowledge merely because he incorporated a statement of such fact into his communication to his attorney.
        2. Lefcourt v. United States, 125 F.3d 79 (2d Cir. 1997) – Fact of relationship alone is not usually privileged
          1. Facts: Lawyer refused to disclose identity of client who gave him over $10,000 on IRS form as required by the reporting requirements. He claimed it was privileged information.
          2. Holding:
            1. Client identity and fee information = fact and therefore not privileged.
            2. Exception – legal advice exception – where there is a strong probability that disclosure would implicate the client in the very criminal activity for which legal advice was sought. There is a special circumstances exception, but the possibility that the client may be incriminated by disclosure is not one of them. (Some circuits do recognize a legal advice exception, where here is a strong possibility that the disclosure would implicate the client in the very criminal activity for which the legal advice was sought).
        3. Adams v. Franklin. Attorney was allowed to be deposed on letter he sent to say whether it (1) was authentic; (2) whether he sent it; (3) and whether he represented the plaintiff; (4) and where the lawyer learned the information in the letter. This all was not protected by privilege.
        4. East Missouri Baptist Church v. Regions Bank (Mo 2007). The production of conflict memoranda, new matter forms, and correspondence regarding conflicts is not protected by the attorney-client privilege or work product doctrine.

 

      1. (2) made between privileged persons – clients, client’s lawyers, agents.
        1. Between an attorney and a client – attorney includes anyone who acts as an agent of attorney.
        2. United States v Kovel, 296 F.2d 918 (2d Cir. 1961)
          1. Facts: Court instructs a law firm’s accountant to disclose information about a client. The accountant claims that the information is privileged. Court asks whether a nonlawyer can claim the privilege.
          2. Holding: The privilege includes all persons who act as an attorney’s agents. What is vital to the privilege is that the communication be made in confidence for the purpose of obtaining legal advice from the lawyer. If what is sought is not legal advice but only an accounting service, for example, or if the advice sought is the accountant’s then no privilege exists.
        3. Privilege with accountant and lawyer
          1. X’s conversation with his accountant is not privileged. There is no accountant-client privilege, and X cannot shield the conversation in attorney-client privilege by subsequently having the same conversation with this lawyer. Since the advice is not sought for legal advice or if the advice sought is accountants rather than attys NO PRIVILEGE EXISTS” because the purpose of the communication it not to obtain legal advice.
          2. Y’s conversation is privileged. Y’s situation is exactly analogous to that in Kovel. The lawyer directed Y to speak with the accountant as a sort of “translator” to help understand and to better provide legal services. Since lawyer has directed the client, either in specific case or generally, to tell his story in the 1st instance to an accountant engaged by the atty, who is there to interpret it so that the atty may better give legal advice, communication by the client reasonably related to that purpose ought to fall w/i the privilege.” Y’s communications will thus be privileged.
          3. The result does not make sense. In both scenarios, both the accountant and the lawyer receive the information from the client. The only difference is the timing with which they receive the information. Even if it doesn’t make sense, Kovel indicates that is the proper result, which highlights the importance of showing that the conversation at issue was for the purpose of securing legal services

 

      1. (3) in confidence – at the time and in the circumstances of the communication, the communicating person reasonably believes that no one will learn the contents of the communication except a privileged person
        1. No third party present – Minnesota v. Rhodes, 627 N.W. 2d 24 (MN 2001 – Communication is not in confidence if there is a present third party which destroys the confidential nature of the community.
          1. Facts: Man accused of killing his wife. Attorney testified that man and wife had visited him to discuss a possible divorce. Man claimed this was protected by privilege.
          2. Holding: The court held that this was not protected by the attorney client privilege because the husband was the client and the presence of the third party, the wife, destroyed the privilege. They were not joint clients. So the testimony was admitted.

 

      1. (4) for the purpose of obtaining or providing legal assistance for the client.
        1. Relating to legal advice – for purpose of obtaining legal advise – requiring the primary purposes of a communication be to to obtain such assistance or that legal advise predominate other aspects of a communication.
        2. Neuder v. Battelle Pacific Northwest National Laboratory, 194 F.R.D. 289 (D. D.C. 2000) – conversations whose primary purpose is to make a business decision, not to obtain legal service are not privileged.
          1. PRIMARY PURPOSE TEST – communications that are mixed, business and legal, are privileged if the primary purpose of the communication was to obtain legal advice.
          2. Litmus Test: Would the communication have sounded the same if the person weren’t a lawyer? whether person acted in a legal capacity. It is better to focus on particular communications then on general capacities
        3. In re Ford Motor Company (3d 1997)
          1. Facts: Ford sued on the theory that its Bronco had a defective design. The company’s lawyer drafted a report and presented it to a committee at a meeting. The defendant sought discovery.
          2. Holding: The court found that the decisions of the meeting were made only after securing legal advice and thus the minutes of the meeting are privileged.
            1. Handwritten notes by attorney was also protected under the work-product doctrine because
        4. Compare Burton v. RJ Reynolds Tobacco Co, Inc. (Kan. 1977).
          1. Facts: Plaintiffs sued RJ Reynolds on products liability theories. The company involved its lawyers in every aspect of its research into the health effects of cigarettes. Plaintiffs moved to compel discovery of the research and defendant claimed it was privileged.
          2. Holding: Court held the documents were not privileged. It is not the case that every document generated by or that references an attorney is privileged. Some courts look at whether the work being performed required the services of an attorney or could be performed equally well by a non-lawyer.

 

        1. In the Matter of Feldberg, 862 F.2d 622 (7th Cir. 1988)
          1. Facts – Attorney serving company in two functions: as a document custodian and as an attorney. Court wants to question him on questions related to client charged with mail-fraud for forward dating contracts for athletes who at the time were amateur athletes but were being signed on professional forward dated contracts.
          2. Holding
            1. Since questions about the adequacy of the search do not entail legal advice, the topic is not off limits just because an attorney plays a role.
            2. There is no need for a privilege to cover information exchanged in the course of document searches, which are mostly mechanical yet which entail great risks of dishonest claims of complete compliance.
            3. A corporation may not throw a veil of privilege over the details of how files were searched, and by whom, through the expedient of involving a lawyer in the process.
            4. The distinction between the mechanical and advisory portions of the lawyer’s role shows that the district court was right to compel answers to several but not all of the questions.

 

        1. I’m Only Your Lawyer When You’re Not Trying To Kill People
          1. Feldberg distinguishes within a single representation between communications for the purpose of obtaining legal services and communications for some other purpose.
          2. The following case considers if a lawyer represents a client in one matter and has a discussion related to a different matter (or acts that would lead to a different matter).
          3. State v Branham
            1. Kelly was a friends to both the husband and wife and make it clear he would not represent either of them in a divorce proceedings. After husband murdered wife, Kelly told government that weeks before husband had threatened to kill her.
            2. Holding – this was not privileged. D never asked for any legal advice and Kelly never gave any legal advice. D did not consult Kelly with purpose of obtain legal services and Kelly did not render legal servies to D.

 

    1. § 77. Duration Of The PrivilegeUnless waived (see §§ 7880) or subject to exception (see §§ 8185), the attorney-client privilege may be invoked as provided in § 86 at any time during or after termination of the relationship between client or prospective client and lawyer.

 

    1. ENTITIES
      1. When you represent an entity such as a corporation, partnership, or LLC, you represent the entity itself, as an entity. (MR 1.13). You do not represent the individuals working for or involved in the entity (constituents).
      2. The entity itself holds the attorney client privilege. Entity constituents such as officers do not hold the privilege even though it is their communications that are or are not protected.
      3. Restatement §73 The Privilege For An Organizational Client – When a client is a corporation, unincorporated association, partnership, trust, estate, sole proprietorship, or other for-profit or not-for-profit organization, the attorney-client privilege extends to a communication that:
        1. (1) otherwise qualifies as privileged under §§ 6872;
        2. (2) is between an agent of the organization and a privileged person as defined in § 70
        3. (3) concerns a legal matter of interest to the organization; and
        4. (4) is disclosed only to:
          1. (a) privileged persons as defined in § 70; an
          2. (b) other agents of the organization who reasonably need to know of the communication in order to act for the organization.
      4. Client Information As Client Property III – When a client is an entity, the rights in the communications are like one of the company’s assets. When one entity buys another, the new company acquires the assets.
      5. Tekni-Plex, Inc. v. Meyner & Landis, 674 N.E. 2d 663 (N.Y. 1996)
        1. Facts
          1. Law firm represents company and sole shareholder. After merger the firm wants to repesent the individual.
          2. The acquiring company filed litigation against Tang alleging breach of representations and warranties regarding the former company’s compliance with environmental laws.
          3. Tang retained M&L to represent him.
          4. D’s want to remove law firm from post merger representation because in the current matter, the firm is adverse to former client – the company.
        2. Holding
          1. Company holds the privilege. When a company is sold or merger takes place, the attorney-client privilege goes with it.
          2. Law firm should be disqualified.
          3. Enjoined the firm from representing Tang, and disclosing to him any information from the old company, and directed the firm to return all files on the old company to the new company. The court denied both motions to dismiss.
          4. The new company, however, does not control the privilege with regard to discrete communications made by the old company or Tang individually to the firm concerning the acquisition—a time when the old company and tang were joined in an adversarial relationship to the acquirer.
          5. Therefore, the new company cannot assert the privilege in order to prevent the firm from disclosing the contents of such communications to Tang. Now is the new company entitled to the firm’s confidential communications concerning its representation of the old company with regard to the acquisition.
          6. New company must show the following in seeking disqualification – if they prove this gives rise to an irrefutable presumption of disqualification – must prove
            1. The existence of a prior attorney-client relationship between the moving party and opposing counsel
            2. That the matters involved in both representations are substantially related,
            3. That the interests of the present client and former client are materially adverse.

 

      1. In re Grand Jury Subpoena, 274 F.3d 563 (1st Cir. 2001)
        1. Facts
          1. Federal grand jury subpoeanas corporation seeking records pertaining to the affairs of a subsidiary.
          2. The corporation and subsidiary waived all claims of privilege
          3. Subsidiary’s former attorney and two of its former officers intervened and moved to quash the subpoena – claimed the subsidiary had entered into a longstanding joint defense agreement with the former officers and contended that the subpoenaed materials were privileged
        2. Holding
          1. An individual privilege may exist in these circumstances only to the extent that the communications made in a corporate officer’s personal capacity are separable from those made in his corporate capacity.
          2. Individual Attorney-Client Privilege Claim – The two officers can only claim the privilege to the extent that the lawyer represented them individually. If only the privilege of the employer is at stake then the company’s waiver is effective since current management can waive the privilege.
            1. The burden is on the officers to show that the attorney represented them individually, the presumption is against this.
            2. 5 Factor Test
              1. They approached counsel for the purpose of seeking legal advice
              2. Made it clear that they were seeking legal advice in their individual (personal) rather than in the representative capacity
              3. Counsel agreed and saw fit to communicate with them in their individual capacities, knowing that a possible conflict could arise
              4. Their conversation with counsel were confidential and
              5. Substance of their conversation with counsel did not concern matters within the company of general affairs of the company.
            3. Here the lawyer could have represented the officers, but the privilege would only extend to matters concerning their individual rights arising out of their actions as officers of the corporation.
            4. Officers may only assert the privilege to the extent that the communications regarding individual acts and liabilities are segregable from discussions about the corporation. Here the officers have not shown this and the documents do not lend themselves to separation. Therefore all of the communications here are corporate communications. This dooms the interveners claim of attorney-client privilege.
            5. ONLY IF SEPARATE FROM CORPORATE CONVERSATIONS.
          3. If privileged conversations are not separate from corporate priviledge – coroproation can waive the entire privilege.
          4. Because the interveners do not allege any of the documents are solely privileged to them but rest instead on the theory that all documents are jointly privileged, their claim, as a matter of law, does not survive the subsidiary’s waiver.

 

      1. IN HOUSE COUNSEL – Intra-firm communications and privilege
        1. A firm may not withhold from a current client at least come privilege or work product information produced during the firm’s investigation of its conduct regarding the clients matter.
        2. Scope is not well defined because of differing views
          1. There is a fiduciary obligation to client – duty to inform client of material developments and this vitiates the privilege
          2. Lawyers misconduct creates a conflict between the firm and the client. Firm must disclose enough information so client can determine whether to fire the firm.

 

    1. EXCEPTIONS
      1. CRIME FRAUD EXCEPTION – Communications furthering crime or fraud
        1. Restatement §§ 82. – Client Crime Or Fraud – The attorney-client privilege does not apply to a communication occurring when a client:
          1. (a) consults a lawyer for the purpose, later accomplished, of obtaining assistance to engage in a crime or fraud or aiding a third person to do so, or
          2. (b) regardless of the client’s purpose at the time of consultation, uses the lawyer’s advice or other services to engage in or assist a crime or fraud.
        2. In re Sealed Case, 107 F.3d 46 (D.C. Cir. 1997) – Requires intent by actor holding privlege – in this case government must prove company seeking legal advice with intent to further its illegal conduct.
          1. Facts
            1. Company contributed maximum amount of money to political candidate. Vice-president asked individuals to contribute to candidate and them reimbursed them with company money.
            2. Two memorandums involved these actions. One memorandum was from the vice-president. The other was from the company’s general counsel, a memorandum to the file. It recites the actions the company took to correct the VP’s use of corporate funds to reimburse the donors.
            3. The district court had ordered the company to turn over the documents – company claim crime fraud.
          2. Holding
            1. Two conditions must be met for the crime-fraud exception to apply:
              1. The client must have made or received the otherwise privileged communications with the intent to further an unlawful or fraudulent act
              2. The client must have carried out the crime or fraud. So the exception does not apply even though, at one time, the client had bad intentions.
            2. The government in this case had the burden of showing the application of the crime-fraud exception.
            3. A third party’s bad intent cannot remove the protection of the privilege. The client was the company, not the VP and so the government had to show the company intended to further and did commit a crime to invoke the exception. The government had to show the company sought the legal advice with the intent to further its illegal conduct. Showing temporal proximity between the communication and the crime is not enough.
            4. The government also did not show that the company was responsible for the crimes of its agent.

 

        1. In The Matter of Michael Feldberg, 862 F.2d 622 (7th Cir. 1988)
          1. To drive away the privilege, there must be “something to give colour to the charge;” there must be prima facie evidence that it has some foundation in fact.
            1. A prima facie case is one that requires an adverse party, the one with superior access to the evidence and in the best position to explain things, to come forward with that explanation. If the court finds the explanation satisfactory, the privilege remains.
            2. Once the evidence is supplied, the seal of secrecy is broken.

 

        1. Show What?
          1. In re Sealed Case and Feldberg agree that a party asserting the crime-fraud exception must make a prima facie showing that the communication it wants disclosed was made in furtherance of a crime or fraud. But they define that showing differently.
            1. Sealed case gives two definitions:
              1. (1) one requires enough evidence to establish the elements of an ongoing or imminent crime or fraud;
              2. (2) the other requires evidence sufficient to establish probable cause to believe that a crime or fraud had been committed and that the communications were in furtherance thereof.
            2. The Feldberg court said: evidence sufficient to require an explanation rather than evidence that by itself satisfied a more likely than not standard.
            3. The 9th Circuit uses a different test.
              1. In a civil case, the burden on the party seeking disclosure is preponderance of the evidence. In these cases, the party seeking to preserve the privilege has the right to introduce countervailing evidence.
              2. Grand Jury -“Reasonable cause to believe that the attorney’s services were utilized in furtherance of the ongoing unlawful scheme.” The reasonable cause standard is more than a suspicion but less than a preponderance of the evidence.

 

      1. JOINT CLIENT EXCEPTIONS
        1. CO-CLIENTS
          1. Restatement §75The Privilege Of Co–Clients
            1. (1) If two or more persons are jointly represented by the same lawyer in a matter, a communication of either co-client that otherwise qualifies as privileged under §§ 6872 and relates to matters of common interest is privileged as against third persons, and any co-client may invoke the privilege, unless it has been waived by the client who made the communication.
            2. (2) Unless the co-clients have agreed otherwise, a communication described in Subsection (1) is not privileged as between the co-clients in a subsequent adverse proceeding between them.
              1. Brennan’s, Inc. v. Brennan’s Restaurants, Inc., 590 F.2d 168 (5th Cir. 1979)As between joint clients, there can be no confidences.
          2. LAYWER MAY SHARE WITH ONE CLIENT COMMUNICATIONS FROM ANOTHER – WITHOUT WAIVING TO THIRD PARTY
          3. EITHER PARTY MAY ASSERT PRIVILEDGED AGAINST THIRD PARTY
        2. JOINT CLIENTS WHO END UP LITIGATING AGAINST EACH OTHER
          1. EACH CLIENT MAY WAIVE PRIVILEGE WITH REST OF HIS OR HER OWN COMMUNICATION WITH COUNSEL BUT NOT WITH RESPECT TO ANOTHER CLIENT’S COMMUNICATION
          2. Teleglobe
          3. Diverging interests of joint clients.The Restatement’s conflicts rules provide that when a joint attorney sees the co-clients’ interests diverging to an unacceptable degree, the proper course is to end the joint representation. Where the attorney does not do so and continues to represent the clients, the communications are privileged against each other notwithstanding the lawyer’s misconduct. Eureka Inv. Corp. v. Chicago Title Ins. Co. (DC 1984).
          4. Example – A agreeing to testify against B when they were joint clients. In their litigation against each other there is no privilege
          5. THIS IS NOT THE SAME AS CONFIDENTIALITY RULE

 

    1. WAIVER
      1. Privileged communication and by virtue of some act, the privilege is waived. This is not an exception. You only worry about waiver if privilege applies.
      2. Only client can waive
      3. Inadvertent Disclosure – Unintentional/Accidental Waiver
        1. Accidental disclosure is not a waiver unless following waiver you are indifferent. Might be waiver.
        2. Model Rule 4.4(b)RESPECT FOR RIGHTS OF THIRD PERSONS
          1. (b) A lawyer who receives a document relating to the representation of the lawyer’s client and knows or reasonably should know that the document was inadvertently sent shall promptly notify the sender.
          2. If you have a document which appears to be privileged you should tell the other side.
          3. EARLY NOTICE IS GOOD. The Fact that you have a document is a discoverable fact! You would have to produce the stolen messages during discovery.
          4. This does not say as soon as you know – stop reading it. It just says tell the other side.
          5. California says stop reading.
        3. Rico v. Mitsubishi Motors, Inc., 42 Cal. 4th 807 (2007) – Technically work product case (since expert discoverable) but analysis still applies
          1. Facts
            1. He left the room and somehow during deposition opposing counsel acquired the notes. Opposing counsel used it and distributed it to his co-counsel, even though it was obvious that it was privileged.
            2. Johnson claims – court reporter inadvertently handed it to me as an exhibit.
          2. Holding
            1. Not a client’s waiver – attorney’s mistake.
            2. The court disqualified counsel as a violation of the work product rule.
            3. An attorney in these circumstances may not read a document any more closely than is necessary to ascertain that it is privileged.
            4. Once it is apparent that the content is privileged, counsel must immediately notify opposing counsel and try to resolve the situation. Here, the court disqualifies counsel.
        4. FEDERAL Rule 502 – Attorney-Client Privilege and Work Product; Limitations on Waiver
          1. (b) Inadvertent disclosure When made in a Federal proceeding or to a Federal office or agency, the disclosure does not operate as a waiver in a Federal or State proceeding if:
            1. (1) the disclosure is inadvertent;
            2. (2) the holder of the privilege or protection took reasonable steps to prevent disclosure; and
            3. (3) the holder promptly took reasonable steps to rectify the error, including (if applicable) following Federal Rule of Civil Procedure 26(b)(5)(B).

 

        1. LAWYERS DEMANDS DOCUMENTS BACK – 4.4 does not say how you should react
          1. You wont be disciplined for not returning but you should. The next step would be a court order to return it. Unless it falls within crime fraud, fight but if not, return it so you save the court the time and trouble.
          2. Claw back provision – if you sent notice asking for material to be returned – FRE 502 codified this claw back procedure and will trump state law.
            1. FRE 502
              1. Even if found, inadvertent waiver does not extend beyond the communication disclosed.
              2. Only if fairness requires
              3. Does not waive privilege in either federal or state proceedings if
                1. The disclosure was unintentional
                2. The disclosing party took reasonable steps to guard against disclosure and
                3. The disclosing party promptly took reasonable steps to rectify the error

 

        1. Metadata and Inadvertent Disclosure
          1. Metadata refers to information about a document that is contained in the document but is not part of the final contents of the document. Ex) track changes and who has looked at it.
          2. Obligation to scrub the document of meta data. The duty of confidentiality requires lawyers to scrub such data before distributing the document.
          3. If you get a file are you able to see if people failed to scrub? Opinion vary
            1. Some jurisdictions find this as trying to obtain privilege data
            2. Other jurisdiction say this is fair game if they are stupid enough to leave it in
          4. Currently, if a lawyer fails to scrub a document, there is no prohibition under the Model Rules on using inadvertently produced metadata. The opinion 06-442 believes Rule 4.4(b) forecloses such a conclusion. Other courts reach a different conclusion.

 

      1. Deliberate Waiver
        1. Restatement §§78 – Agreement, Disclaimer, Or Failure To Object
          1. The attorney-client privilege is waived if the client, the client’s lawyer, or another authorized agent of the client:
            1. (1) agrees to waive the privilege;
            2. (2) disclaims protection of the privilege and
              1. (a) another person reasonably relies on the disclaimer to that person’s detriment; or
              2. (b) reasons of judicial administration require that the client not be permitted to revoke the disclaimer; or
            3. (3) in a proceeding before a tribunal, fails to object properly to an attempt by another person to give or exact testimony or other evidence of a privileged communication.
        2. Restatement §§79Subsequent Disclosure
          1. The attorney-client privilege is waived if the client, the client’s lawyer, or another authorized agent of the client voluntarily discloses the communication in a nonprivileged communication.
        3. Restatement §§80Putting Assistance Or A Communication In Issue
          1. (1) The attorney-client privilege is waived for any relevant communication if the client asserts as to a material issue in a proceeding that
            1. (a) the client acted upon the advice of a lawyer or that the advice was otherwise relevant to the legal significance of the client’s conduct; o
            2. (b) a lawyer’s assistance was ineffective, negligent, or otherwise wrongful.
          2. (2) The attorney-client privilege is waived for a recorded communication if a witness:
            1. (a) employs the communication to aid the witness while testifying; o
            2. (b) employed the communication in preparing to testify, and the tribunal finds that disclosure is required in the interests of justice.
        4. (a) Scope of waiver.When the disclosure is made in a Federal proceeding or to a Federal office or agency and waives the attorney-client privilege or work-product protection, the waiver extends to an undisclosed communication or information in a Federal or State proceeding only if:
          1. (1) the waiver is intentional
          2. (2) the disclosed and undisclosed communications or information concern the same subject matter; an
          3. (3) they ought in fairness to be considered together.  

 

        1. In re Von Bulow, 828 F.2d 94 (2d Cir. 1987) – Partial Waiver
          1. Facts: Von Bulow charged for assaulting his wife. Dershowitz had him acquitted. Then the attorney wrote a book about the first trial. Wife’s friends filed a civil action and wrote to petition saying they would consider publication of the book as waiver of the attorney client privilege. The book was released and counsel moved to compel discovery of certain discussions between the attorney and client with respect to the communications related in the book.
          2. Holding:
            1. Only a client can waive the attorney client privilege, but he can do so impliedly and can also do so by giving his attorney implied authority to waive the privilege.
            2. Here the clientconsented to the attorney’s disclosure in publishing the book and so he waived the attorney client privilege.
            3. DEFAULT – Subject matter
              1. You cannot disclosure only the good parts of the privileges conversation. All information relating to a common subject matter must be disclosed once mentioned.
              2. Where part of a conversation or document is disclosed, the remainder must be disclosed as well. Entire subject matter is disclosed, not just certain aspects of subject matter.
            4. However, the extrajudicial disclosure of an attorney client communication, one not subsequently used by the client in a judicial proceeding to his adversary’s prejudice, does not waive the privilege as to the undisclosed portions of the communication. So the parts not disclosed in the book do not need to be produced. In extrajudicial disclosures, there is also no subject matter waiver (related matters not disclosed remain confidential).

 

      1. Selective Waiver
        1. Majority view – if you disclose to one people, you cannot limit who else will find out.
        2. This attempt usually arises when a large company created an internal investigation – tried to turn over material to some governmental entity but not for a civil litigation which might later ensure. However, once waived to one person, waived to all people.
        3. Westinghouse Electric Corrporation v. Republic of the Philippines (3d Cir. 1991). The court held that the client in disclosing letters to a government agency had waived the privilege. The court distinguished between selective and partial waiver. Selective waiver permits the client who has disclosed privileged communications to one party to continue asserting the privilege against other parties. Partial waiver permits a client who has disclosed a portion of privileged communications to continue asserting the privilege as to the remaining portion of the same communications. Courts diverge on what they allow. Most courts agree that partial waiver raises fairness concerns to the opposing party; but they do not agree as to selective waiver.

 

      1. EXCEPTION TO WAIVERS
        1. Disclosure within another privileged relationshipThe general rule is that disclosure of a privileged communication within the scope of some other privilege is not waiver.
          1. Ex) Telling your doctor or husband what you told you lawyer.
        2. Common Interest” Exception to Waiver (Drunk defense privilege)
          1. Restatement §76The Privilege In Common–Interest Arrangements
          2. (1) If two or more clients with a common interest in a litigated or nonlitigated matter are represented by separate lawyers and they agree to exchange information concerning the matter, a communication of any such client that otherwise qualifies as privileged under §§ 6872 that relates to the matter is privileged as against third persons. Any such client may invoke the privilege, unless it has been waived by the client who made the communication.
          3. Involves many lawyers and many C’s
          4. The common interest doctrine provides an exception to the general rule that a client waives the attorney-client privilege by communicating previously privileged information to a third party. The doctrine permits represented parties who share a common legal interest to exchange privileged information in a confidential manner for the purpose of obtaining legal advice without waiving the attorney-client privilege.
          5. United States of America v Douglas Stepney – This is an Exception to Waiver – not joint defense privieldge.
            1. Facts
              1. 30 plus D’s case. Federal dealing drug causes of action.
              2. Create a joint defense agreement
              3. Client 2 agrees to break and deal and go against C1. C2 goes to trial and L1 has to cross C2
              4. L1 said I need to withdraw; I have a conflict of interest. I cant cross C2 to best reflect my C1’s interest because he has confidential info that belonged to C2 and owes him a duty now. (This can piss off a judge!!!)
            2. Holding
              1. Joint defense agreement allow you the duty of confidentiality but you cannot create attorney client privilege nor duty of loyalty between L1 and C2.
              2. Attorney may use information about C2 to impeach on cross because not duty of loyalty, care, etc.
            3. Prisoner’s dilemma – what if one client decides to cooperate? More common on criminal side
              1. The best situation is where they cooperate.
                Cooperate Defect
                Cooperate 1, 1 – neither confess 7, 0 – person who doesn’t confess gets 7
                Defect 0, 7 – person who doesn’t confess gets 7 5, 5 – both confess

                 

              2. Solutions: how do you cooperate with each other. Trust and information (not sitting in two different cells. Counsel can enter into a joint defense agreement. This term applies when several parties have a common legal interest and its purpose is to create an exception to the rule that disclosure of confidential communications waives the attorney-client privilege with respect to (at least) such communications.

 

  1. CHAPTER 5 – WORK-PRODUCT DOCTRINE
    1. Not about confidentiality. A lot of work product is based on notion that other people can find it out if they want to.
    2. Fed. R. Civ. P. 26(b)(3)
      1. A party may obtain discovery of documents and tangible things otherwise discoverable and prepared in anticipation of litigation or for trial by or for another party or by or for that other party’s representative ONLY UPON A SHOWING that the party seeking discovery has substantial need of the materials in the preparation of the party’s case and that the party is unable without undue hardship to obtain the substantial equivalent of the materials by other means. 
      2. In ordering discovery of such materials when required showing has been made, the court shall protect against disclosure of the mental impressions, conclusions, opinions, or legal theories of an attorney/ other rep of a party concerning litigation.
    3. The Basic Doctrine
      1. The work product of an attorney is material prepared in anticipation of litigation, including but not limited to statements taken from witnesses, notes taken at meetings with witnesses or other people knowledgeable about matter in dispute, and memos summarizing legal research.
      2. Hickman v. Taylor, 329 U.S. 495 (1947)
        1. Fact – transcription of third party statements
        2. Opinion – more than copy of conversation. Notes of conversations include a filter which create the work product
        3. Production of this material will be compelled only upon a showing of substantial need and that the party is unable without undue hardship to obtain the substantial equivalent of materials by other means
      3. Upjohn Co. v. United States, 449 U.S. 383 (1981) – Discovery of mental impressions may be possible but there must be a very strong showing of need and unavailability (much stronger than for discovery of witness statements and summaries).  In fact, the Court indicates that there are very few, if any, circumstances where mental impressions can be discoverable. 

 

    1. Scope
      1. Usually covers everything you produce working for a client.California – MENTAL IMPRESSION IN GENERAL
        1. Opinion and fact
        2. Opinion work product received heightened protection
      2. FEDERAL – ONLY IF PREPARED IN ANTICIPATION OF LITIGATION
        1. United States v. Adlman, 134 F.3d 1194 (2d Cir. 1998) – Where a document was created because of anticipated litigation, and would not have been prepared in substantially similar form but for the prospect of litigation, it falls within Rule 26(b)(3)
        2. Primarily to assist in litigation – 5th
        3. Where a document is created in anticipation of litigation or analyzing the likely outcome of that litigation, it does not lose protection under the work product doctrine merely because it is created in order to assist with a business decision.
        4. United States v Textron, Inc. – Does not apply if they are done merely because required by law.

 

    1. Exceptions
      1. Crime fraud – FEDERAL NOT CALIFORNIA (only in government investigations – no civil side).
        1. Restatement §§ 93 – Client Crime Or FraudWork-product immunity does not apply to materials prepared when a client consults a lawyer for the purpose, later accomplished, of obtaining assistance to engage in a crime or fraud or to aid a third person to do so or uses the materials for such a purpose
      2. Self-defense
      3. NOT client’s criminal or fraudulent activity or conduct

 

    1. Waiver
      1. If the worked product is place at issue or disclosed by testifying expert
      2. Common interest exception to waiver – whether parties collaborate on work product ech contributing party must agree to waive protection for a waiver to be effective.
      3. Control by lawyer.
      4. Waiver By Disclosure to Testifying Experts or When Work Product is At Issue – WP in forming her opinion, the WP must be disclosed under the general rule requiring disclosure of all materials an expert considers in forming an opinion.

 

 

    1. Difficulty of joint client situations
      1. If you have three clients – or if you have a common interest arrangement – common issue, having expert to opine on it, study, field test, etc. cost you a 1/3 to produce work product. In a situation where 1 of the 3 peels off and settles. In privilege only waive privilege of my communication but not in regards to yours
        1. Rule in California – you need unanimous consent to waive work product protection
        2. Rule in federal – maybe the same

 

    1. Confidentiality and Work Product – limited disclosure of work product material does not waive the protection of the doctrine unless the disclosure is likely to reveal work product material to an opponent. Not as cncern with confidentiality as the attorney-client privieldue is.

 

    1. Some Differences Between Federal and California Law
      1. Crime fraud exception
        1. Fed – yes
        2. California no
      2. Material prepared in anticipation
        1. Fed – yes
        2. California – no

 

 

Duty of Confidentiality Privilege Work Product Doctrine
Covers All information relating to representation not generally known Confidential communications between attorney and client for the purpose of providing legal services is privileged and non-disclosable unless ordered under the rules of evidence which would be rare because the privilege protects it and can  only be waived by the client. Mental impressions of counsel; other work done on behalf of client in preparation for litigation
Applies Applies at all times Rules of evidence apply Material is prepared in anticipation of litigation
Effect Forbids voluntary disclosure (but not disclosure required by law) or use of information that harms client or benefits lawyer outside the practice of law Prevents the kinds of disclosure evidence rules could otherwise compel Requires party seeking disclosure to demonstrate substantial need for disclosure and that disclosure would not work unde hardship on producing party.Standard for producing opinion work product is higher—close to absolute protection
Controlled by Client Client Lawyer (except in dispute with client)
Exceptions Prevent death or serious bodily injury;Prevent or rectify serious financial harm in which the lawyers’ services have been used;Self-defense (respond to allegation of representation-related misconduct);Seek advice about compliance with rules;Disclosure required by law Communications in furtherance of a crime or fraud;Joint Clients;

Common Interest Exception to Waiver

Crime-fraud(FED NOT CALIFORNIA); Self-defense

 

 

  1. CHAPTER 6 – REQUIREMENTS OF AND RELATING TO THE DUTY OF CARE
    1. Two manifestations
      1. Liability and Cause of Action – Malpractice (civil)
        1. Duty – act reasonable (what a reasonable prudent lawyer would do in context – based on community, etc.
        2. Breach
        3. Cause in fact
        4. Proximate cause
        5. Harm
      2. Disciplinary side – Negligence
    2. Civil malpractice
      1. DUTY AND BREACH
        1. Model Rules 1.1COMPETENCE: A lawyer shall provide competent representation to a client. Competent representation requires the legal knowledge, skill, thoroughness and preparation reasonably necessary for the representation.
        2. Model Rules 1.2(c)(c) A lawyer may limit the scope of the representation if the limitation is reasonable under the circumstances and the client gives informed consent.
        3. Model Rules 1.3DILIGENCEA lawyer shall act with reasonable diligence and promptness in representing a client.
        4. Model Rules 1.4 COMMUNICATION
          1. (a) A lawyer shall:
            1. (1) promptly inform the client of any decision or circumstance with respect to which the client’s informed consent, as defined in Rule 1.0(e), is required by these Rules;
            2. (2) reasonably consult with the client about the means by which the client’s objectives are to be accomplished;
            3. (3) keep the client reasonably informed about the status of the matter;
            4. (4) promptly comply with reasonable requests for information; an
            5. (5) consult with the client about any relevant limitation on the lawyer’s conduct when the lawyer knows that the client expects assistance not permitted by the Rules of Professional Conduct or other law.
          2. (b) A lawyer shall explain a matter to the extent reasonably necessary to permit the client to make informed decisions regarding the representation.
        5. DUTY TO SEEK ADVISE
          1. The Chicken and the Egg: A Duty to Seek Supervision
            1. Competence includes exercising good judgment – rests of experience. How do you get experience before you are put in a situation where you need to act competently
          2. Beverly Hills Concepts, Inc. v. Schatz & Schatz, Ribikoff and Klotkin, 717 A.2s 274 (1998) – the duty of care probably requires you to be more competent than you really are. You have a duty to act in a way you are incapable of acting. You need to act as the reasonable lawyer who can actually do this stuff.
            1. Under Rule 1.1, if an attorney is not competent, they must seek supervision to provide competent representation. However, not every instance of professional negligence results in a breach of fiduciary duty—professional negligence implicates the duty of care, while the breach of a fiduciary duty implicates the duty of loyalty. Because the attorney in this case did not represent that she had superior knowledge, her negligence did not rise to a breach of fiduciary duty.
            2. Proceed at you own risk. If you make a mistake, it is no defense that you expected you boss to look out for you. You re liable for malpractice on your own.
              1. If he said its ok
                1. Model rules – this would might a good defense for discipline
                2. Not really a defense to malpractice. Unless she put something in because he TOLD HER TO. Not just that it was “ok”
            3. Solution: combine hard work with framing – do you homework first then frame your request in terms of the senior lawyer’s expertise, not you ignorance

 

        1. YOU MUST ACT COMPETENTLY IF YOU HAVE A DUTY – EVEN A DUTY YOU HAVE WITHOUT KNOWING IT.
          1. Nichols v. Keller, 15 Cal. App. 4th 1672 (1993)
            1. Facts
              1. Fulfler met with plaintiff, had him sign a worker’s compensation form and executed the form as “applicant’s attorney”. Atty said he would represent him in the workers compensation proceeding.
              2. After meeting with another atty plaintiff realized that a third party claim should have been brought and that the plaintiff might have a claim against Keller and Fulfer for failing to inform him of this fact. Brought action about attorneys
            2. Holding
              1. Liability can exist if an attorney failed to advise. Not only should an attorney furnish advice when requested, but he or she should also volunteer opinions when necessary to further the client’s objectives. The attorney need not advise and caution of every possible alternative, but only of those that may result in adverse consequences not considered.
              2. Attorneys believed they were just personal injury lawyers. BUT Client did not see these attorneys are merely worker’s compensation lawyers! He wanted all advice relevant to the accident.
              3. THE FACT THAT YOU HAVE RETAINER THAT SAYS YOU ARE DOING WORKER COMPS ANGLE – THIS IS NOT THE END OF YOUR DUTY.
              4. The lawyer must describe, even given incomplete facts by the client, the array of legal remedies available, alert the client as to any apparent legal problems, and if appropriate, indicate limitations of the retention of counsel and the need for other counsel. If the lawyer wishes to limit his representation, he must make this limitation very clear to his client.
              5. Even if an attorney wants to just cover one area of law, unless they explicitly explain this to client and client reasonably believe the attorney is going to advise on all relevant topics, failing to do so may be a breach of care.Job of the lawyers – when dealing with someone that involves jobs they don’t want to do- they must tell that person they are not going to cover those topics.

 

          1. What You Say and What They Hear I
            1. When you as a lawyer have information a client does not, especially information about your own practice in relation to the facts the client brings you, there is a risk of miscommunication.
            2. The lawyer has the burden to make clear to the client that you will only represent him to a limited extent and that he might have other claims you will not pursue but which he might wish to look into.
            3. The burden of clear communication is on the lawyer. The lawyer must make sure the client’s lack of information does not harm them, at least when it is easy for the lawyer to remedy the lack.

 

          1. Exercise: Seeing Yourself As Others See You pick A and BB
            1. Thinking about how other people see you is fundamental to taking care of yourself. It is hard to think about how other people see you.
            2. Your strategy space is constrained by other people
            3. Even though payer 2 wants 73 – player 1 will never pick row C.

 

        1. The Relationship Between Disciplinary Rules and the Duty of Care
          1. There is no cause of action for violating a disciplinary rule.
          2. However, the cause of action for legal malpractice requires a plaintiff to show that an attorney owed the plaintiff a duty of care and breached it.
          3. Disciplinary rules are relevant to defining the scope and content of duties of care. (They are also relevant to defining the scope and content of fiduciary obligations when the cause of action is for breach of fiduciary duty).
          4. Mirabito v. Liccardo (Cal. App. 1992)The rules of professional conduct, statutes, and general principles relating to other fidcuairy relationships, all help define the duty component of the fiduciary duty which an attorney owes to his client. The court therefore did not err in permitting testimony regarding the rules or by instructing the jury with them. There is no independent cause of action for a breach of a disciplinary rule, rather the breach of duty can be measured by the violation of the rules
          5. Hizey v. Carpenter (Wash. 1992) says that a jury in lawyer malpractice action cannot be informed of rules of professional conduct either by instruction or by expert testimony).

 

      1. CAUSASTION AND DAMAGES
        1. It is P’s burden – that but for lawyer breach, P would have been better off = Trial within Trial
        2. Must show – but for the lawyer’s negligence, the plaintiff would have been better off. This may result in a trial within a trial – plaintiff must show that she would have prevailed in the underlying case. The test is the same in transactional work. The crucial causation inquiry is what would have happened if the defendant attorney had not been negligent. Courts require that it be shown that the loss suffered was in fact caused by the alleged attorney malpractice.
        3. THIS IS HARD FOR CLIENT TO PROVE AND MAY REQUIRE A TRIAL WITHIN A TRIAL TO SHOW THAT ATTY’S BREACH OF CARE RESULTED IN HER LOOSING HER CASE
        4. Viner v. Sweet, 135 Cal. Rptr. 2d 629 (2003) – BUT FOR TEST WOULD HAVE WON
          1. In a client’s action against an attorney for legal malpractice, the client must prove, among other things, that the attorney’s negligent acts or omissions caused the client to suffer some financial harm or loss.
          2. When the alleged malpractice offered in the performance of transactional work, the client must prove this causation element according to the “but for” test, meaning that the harm or loss would not have occurred without the attorney’s malpractice.
          3. This applies is client can show would have won or could have settled rather than case being thrown out!
        5. GIVES ATTY OPPORTUNITY TO TRASH CLIENT’S CASE TOO IN AN ATTEMPT TO SHOW NO CAUSATION

 

      1. LIMITATIONS AND TOLLING – SECOND SILVER BULLET – HARD ON PLAINTIFF
        1. Code of Civil Procedure §340.6(a) – Statute of limitations = 1 year from when person found out or 4 years from date wrongful act occur, whenever comes first in California– although varies from state to state (although in California, if you find out and are trying to fix the error, the limitations period is not running)
        2. Clock stops if representation continues! – Tolled during the period you continue to represent someone. So if you make a mistake and try to fix it, drag out representation, SOL is tolling the whole time.
          1. In no event shall the time for commencement of legal action exceed four years except that the period shall be tolled during the time that any of the following exist:
            1. (1) The plaintiff has not sustained actual injury;
            2. (2) The attorney continues to represent the plaintiff regarding the specific subject matter in which the alleged wrongful act or omission occurred;
            3. (3) The attorney willfully conceals the facts constituting the wrongful act or omission when such facts are known to the attorney, except that this subdivision shall toll only the four-year limitation; and
            4. (4) The plaintiff is under a legal or physical disability which restricts the plaintiff’s ability to commence legal action.
          2. Fritz v. Ehrman (Cal. 2006).Ehrman kept representing fritz until april of 03, under the statute, the period is tolled while the attorney continues to represent the plaintiff concerning the subject matter from which the cause of action occurred. On deferred interest, court says injury did not even happen until it actually became due.
          3. This might not apply to other causes besides duty of careWelch v. Erkstine & Tully (Cal). It did not apply to a claim for breach of fiduciary duty.

 

      1. Different between breach of duty of care and breach of loyalty
        1. BREACH OF DUTY OF CARE (MALPRACTICE)
          1. Legal malpractice – proof of duty + breach + proximate cause + harm
            1. Rule of professional conduct help to define the duty competent under a cause for malpractice. However, some courts Ex) Hizey v Carpenter (Wash. 1992) had held that jury in lawyer malpractice actions cannot be informed of rules of professional conduct.
          2. Burden – lawyer bears burden to show fairness
          3. Plaintiff was establish standard but no expert testimony required if breach is obvious to a lay person
          4. No punitive damages
          5. In criminal – requires proof of innocence
        2. BREACH OF FIDUCIARY DUTY – LOYATLY
          1. Burden – party bringing the claim bears the burden
          2. Requires expert
          3. Yes, punitive damages

 

      1. A Duty to Inform On Yourself? YOU HAVE A DUTY TO BLOW THE WHISTLE ON YOURSELF – IF YOU MESS UP, YOU HAVE TO TELL THE CLIENT.
        1. Accord Tallon v. Comm. (NY 1982). An attorney has a professional duty to promptly notify his client of his failure to act and of the possible claim his client may thus have against him. After making such disclosure, withdrawing from the case and advising his former client of his right to retain other counsel, an attorney may then settle any claim that his former client has against him.

 

      1. Note on fee disgorgement
        1. Discretion for a client to sue in equity to return a fee. Sometimes court order this even in the absence of damages.
        2. Restatement §37Partial Or Complete Forfeiture Of A Lawyer’s Compensation
          1. A lawyer engaging in clear and serious violation of duty to a client may be required to forfeit some or all of the lawyer’s compensation for the matter. Considerations relevant to the question of forfeiture include the gravity and timing of the violation, its willfulness, its effect on the value of the lawyer’s work for the client, any other threatened or actual harm to the client, and the adequacy of other remedies
          2. Does not require that the breach of duty have been intentional in order to support a disgorgement remedy. The willfulness of the breach is a relevant factor, but not a condition.
        3. Disgorgement should be analyzed in relation to the facts of a particular case. Where a lawyer deliberately advances his self interest either at the client’s expense or using the client’s information, standard principles of agency law hold that the lawyer must account to the client for the profits earned from the relevant conduct.

 

    1. CRIMINAL MALPRACTICE – INNOCENCE RULE
      1. Different from civil cause of action for breach of duty of care
      2. In criminal context has an element of pleading and proof that the persons is factually innocent of the crime. If you are guilty it does not matter how bad the lawyer was, you cannot recover
      3. Ex) A’s boss gets in trouble with exchange commission. Lawyer represents A and boss and cuts plea deal for A. A agrees to pay 250k an do suspended time. Attorney failed to tell A that government would not prosecute if she would testify against Boss (because he is also representing boss).
        1. However – since she pleaded guilty, she is not innocent and has no cause of action!
      4. Justifications for this rule
        1. Criminals should not be able to profit from their crimes – TX CASE
        2. The need for collateral review of criminal is less than the need for collateral review of civil because of the right to counsel – 5th amendment. We have a constitutional level of scrutiny, and this is not a state law element and therefore all states are bound by it. – POSNER
        3. The relevant considerations are:
          1. The court should not permit a guilty defendant to profit from his or her own wrong.
          2. To allow guilty defendants to shift their punishment to their former attorneys would undermine the criminal justice system.
          3. A defendant’s own criminal act remains the ultimate source of his predicament irrespective of counsel’s subsequent negligence.
          4. A guilty defendant who is convicted or given a longer sentence as a result of counsel’s incompetence can obtain post conviction relief on that basis, in contrast, a civil matter lost through an attorney’s negligence is lost forever.
      5. The actual innocence rule is the majority rule.
        1. Wiley v. County of San Diego (Cal 1998)A criminal defendant who files a malpractice claim against her criminal defense counsel must plead and prove, as an element of the malpractice claim, that she was exonerated of the criminal convention.
        2. Rantz v. Kaufman (Colo. 2005) The Colorado Supreme Court departed from this rule.

 

      1. EXCEPTIONS
        1. IN MOST STATES A PLAITIFF ALLEGING MALPRACTICE AGAINST A CRIMINAL LAWYER MUST PLEAD AND PROVE THAT HE OR SHE WAS ACTUALLY INNOCENT OF THE CRIME ALLEGED
        2. The actual innocence rule applies to disputes that focus on competent representation, not to suits that focus on some other duty of the lawyer or right of the client.
        3. EXCEPTION IF YOU ARE CLAIMING YOU STOLE MY MONEY, RETURN MY MONEY.
          1. Winniczek v. Nagelberg, 394 F.3d 505 (7th Cir. 2005)
            1. Facts
              1. D hires him – 150k in fees – 20k in costs – after the last check clears he says, I am not going to try your case because you made exculpatory statements when you were arrested.
              2. Winniczek is found guilty then sues lawyer for fraud and malpractice and also, negligence
            2. Holding
              1. Usually since D is found guilty – no claim for malpractice due to innocence rule and therefore cannot say you did a bad job representing me
              2. However, in this case, the claim is that the lawyer stole from him! This is collateral to the representation and therefore not barred by the innocent rule. The recovery of overcharge is not barred by the actual innocence rule since it is a claim for breach of contract. A judgment for the client in a fee dispute is not inconsistent with a judgment for the people in a criminal case.
                1. You messed up – barred by innocent
                2. You stole my money – not barred by innocent
          1. Shemaria v. Brooks (Cal. 2006) Held that the actual innocence rule was not a bar to a suit arising out of a criminal defense lawyer’s alleged failure to return the unearned portion of the retainer. The court also held that the actual innocence rule did not bar a claim for negligent failure to seek the return of seized property, even though the claim related to the lawyer’s competence. The court said in determining whether the actual innocence rule applies, the court must look at the policy considerations underlying the actual innocence rule.

 

        1. EXCEPTON IF YOU ARE CLAIMING UNLAWFUL PUNISHMENT
          1. Powell v. Associated Counsel for the Accused (Wash 2005) – OVER SERVING TIME
            1. Plead guilty to crime with max 1 year. He was erroneously sentences to 2.5 years in prison and his attorney failed to catch the mistake.
            2. Court said that Powell’s situation is closer to that of an innocent person wrongfully convicted than of a guilty person attempting to take advantage of his own wrongdoing. Powell does not argue against serving the proper amount of time.

 

    1. The Constitutional Standard of Care In Criminal Cases: The Ineffective Assistance of Counsel Doctrine
      1. APPLIES ONLY IN CRIMINAL CASES.
      2. THIS DOES NOT IN ITSELF REQUIRE INNOCENCE.
      3. Requires state-appointed counsel in certain criminal cases.
      4. Court has held that the right to counsel requires lawyers who meet a certain level of competence, judged in part by prevailing professional standards and in part by whether the lawyer’s conduct affected the result of a proceeding.
      5. Defendant’s who wish to challenge convictions often contend their counsel failed to provide effective representation (ineffective assistance of counsel).

 

      1. Strickland v. Washington, 466 U.S. 668 (1984) – CAPITAL CASE *Def on exam
          1. Facts
            1. Client confessed to gruesome crimes during 3 days period. Options
            2. Attorney becomes despondent about the case. It is difficult as a lawyer to be given a hopeless case and told to live with it for two years.
            3. Counsel says he should invoke his right to an advisory jury – D waived his right to a jury against the advice of counsel.
            4. Lawyers fails to do a lot of investigation
              1. Lawyer did not seek character or psychiatric evaluation or present this evidence at trial because he thought it could not overcome the confessions.
              2. Counsel also excluded client’s rap sheet. Counsel argued that client should be spared death because he confessed and he committed the crimes under extremely stressful circumstances.
            5. TC – sentences to death and he appeals. Client claimed that counsel rendered ineffective assistance at the sentencing proceeding.
          2. Holding
            1. In this case, the conduct of counsel was reasonable. And even if it was unreasonable, the respondent suffered insufficient prejudice to warrant setting aside his death sentence. The omitted evidence would not have changed the conclusion.
          3. Rule
            1. The 6th Amendment right to counsel exists in order to protect the fundamental right to a fair trial. The right to counsel is the right to effective assistance of counsel.
            2. The benchmark for judging any claim of ineffectiveness must be whether counsel’s conduct so undermined the proper functioning of the adversarial process that the trial cannot be relied on as having produced a just result.
            3. The presumption is that the attorney’s assistance was effective. The defendant must rebut this presumption.
            4. Defendant must show that counsel’s representation fell below the objective standard of reasonableness. The defendant must show that there is a reasonable probability that, but for counsel’s unprofessional errors, the result of the proceeding would have been different. A reasonable probability is a probability sufficient to undermine confidence in the outcome. This requires D show
              1. Counsel made errors so serious that counsel was not functioning as the counsel required by the Sixth Amendment.
              2. Deficient performance prejudiced the defense. This requires showing that counsel’s errors were so serious as to deprive the defendant of a fair trial, a trial whose result is reliable.
            5. A convicted defendant making a claim of ineffective assistance must identify the acts or omissions of counsel that are alleged not to have been the result of reasonable professional judgment. The court must then determine whether, in light of all the circumstances, the identified acts or omissions were outside the wide range of professionally competent assistance.
          4. JUSTICE MARSHALL DISSENTING: Marshall thinks the reasonable standard is too difficult to apply consistently. Also he thinks the prejudice standard is wrong since it is difficult to tell if the outcome would have been different after the fact.

 

      1. Strickland and later cases are less willing to second guess judgment calls than the failure to investigate (such as if the failure itself was based on a strategic judgment).
        1. Schriro v. Landigan (US 2007) reversed an appellate court finding that counsel had been ineffective. In this case, the defendant refused to allow his lawyer to present mitigating evidence and later claimed that the lawyer was ineffective for failing to show that evidence. The court found counsel was not ineffective because the defendant interfered with counsel’s proffer to the court regarding what such evidence would show.
        2. Ex) Florida v. Nixon (US 2004) A defendant confessed and there was evidence. His public defender urged him to concede guilt at trial (he pleaded not guilty) to try to make his penalty phase arguments more credible. Nixon was unresponsive and neither approved nor disapproved this strategy. After Nixon was convicted and sentenced to death, new counsel argued at that trial counsel was ineffective because he did not get express consent to the concession strategy. The court disagreed. It found counsel had adequately consulted the client and in light of his unresponsiveness, counsel cannot be deemed ineffective for attempting to impress the jury with his candor and unwillingness to engage in a useless charade.

 

      1. However, Bad judgments sometimes amount to ineffective assistance of counsel
        1. BAD JUDGEMENT = INEFFECTIVE COUNSEL – In Miller v. Anderson (7th Cir. 2001) the defendant convicted of murder. Prosecution presented expert testifying that hair on victim was defendants. Counsel chose to only cross-examine that expert and not present one of his own. In post-conviction proceedings a different lawyer got another expert who testified against the other expert.
          1. Most questionable was that he called a psychologist to testify that the defendant was incapabale of that sort of violaence, even though he had previously been convicted for very violent crimes; this destroyed the case. The lawyer could not articulate a coherent reason for putting the psychologist on the stand. If no reason can be given for a tactic, the label “tactic” will not prevent it from being used as evidence of ineffective assistance of counsel. The minimally competent lawyer would not have done this. The lawyers errors here were therefore in the aggregate, prejudicial.
        2. SLEEPING = INEFFECTIVE COUNSEL– In Burdine v. Johnson (5th Cir. 2001) the court held that, where counsel slept in court while evidence of guilt was introduced against his client, the client had been denied counsel at a critical stage of trial and could be presumed under Strickland.
        3. DISTRUCT = INEFFIECTIVE COUNSEL – In Plumlee v. Del Papa (9th Cir. 2005) the court found that denial of counsel where the defendant distrusted the public defenders appointed to represent him, and indeed the public defender’s office generally, and the court found that this distrust was reasonable. The court said that the conflict generated by the distrust was so severe that the defendant had been effectively denied counsel. The court emphasized that the defendant had a reasonable basis for his distrust, and had not manufactured the conflict for strategic purposes.

 

    1. Special Obligations of Prosecutors
      1. LAWYER IS PERMITTED TO ALLOW A FACT FINDER TO DECIDE WHETHER A CLAIM OR DEFENSE IS TRUST EVEN IF THE LAWYER PERSONALLY KNOWNS OF EVIDENCE THAT CONTRADICTS OF MAY OVERWHELM IT.
      2. LAWYER HAVE NO OBLIGATION TO MAKE SURE HIS OR HER OPPONENTS TAKE ADVANTAGE OF PROSECURAL DEVISES THAT MAY BENEFIT THE OPPONENT.
      3. Prosecution’s obligation is to seek justice – not bring charged that are not true. Even if jury might believe it.
      4. Criminal defense attorney – put government to its proof – tell any story the facts might support even if the defense attorney knows it is true. It is client’s call whether to plea so if they don’t, criminal defense must try to raise reasonable doubt.
      5. Model Rule 3.8 Responsibilities Of A Prosecutor
        1. Prosecutor may not bring chargers he know are not supported by probable cause
        2. A prosecutor must make reasonable efforts to ensure an accused person has been advised of right to counsel and reasonable opportunity to get counsel
        3. Much not seem from an unrepresented person a waiver of important rights
        4. Must timely disclose all information the prosecutor know which tends to show accused is not guilty of mitigate the level of guilty
        5. Refrain from making extrajudicial comments that have a substantial likelihood of heightening public condemnation and exercise reasonable care to ensure that persons associated with prosecutor make no such comments
        6. A prosecutor who knows of new, credible, material evidence creating a reasonable likelihood that convicted D did not omit offense for which he was convicted must disclose evidence to the court and D
        7. A prosecutor who knows of clear and convincing evidence showing a D in prosecutor’s jurisdiction was wrongly convicted must seek to remedy that conviction
        8. The prosecutor in a criminal case shall:

 

 

  1. CHAPTER 7 – LIABILITY TO NON-CLIENTS
    1. DUTIES TO PARTIES RELATING TO CLIENT
      1. CLIENT TO THIRD PARTY
          1. If client is a fiduciary for 3P
          2. In certain circumstances lawyer can owe duty to 3P when client owe duty to 3P. If lawyer knows client is harming 3P, lawyer needs to stop it.
          3. Action by L is needed to prevent or rectify C’s breach of fiduciary duty which is also a
            1. crime or fraud or
            2. in which L assisted
            3. and 3P cannot protect itself
      2. LAWYER TO THIRD PARTY
        1. Restatement § 51– Three ways to owe duty
          1. Duty created by invitation – option letter from lawyer to company – becomes responsible
            1. Rule 2.3 Evaluation for Use by Third Persons
              1. (a) A lawyer may provide an evaluation of a matter affecting a client for the use of someone other than the client if the lawyer reasonably believes that making the evaluation is compatible with other aspects of the lawyer’s relationship with the client.
              2. (b) When the lawyer knows or reasonably should know that the evaluation is likely to affect the client’s interests materially and adversely, the lawyer shall not provide the evaluation unless the client gives informed consent.
              3. (c) Except as disclosure is required in connection with a report of an evaluation, information relating to the evaluation is otherwise protected by Rule 1.6.
            2. Ex) All in the Family – Graycas, Inc. v Proud, 7TH CIR 1987 – A owns farm and needs to borrow money. A asks his brother to write an opinion letter saying that A does not have any liens. When A defaults, lender sues brother for negligent misrepresentation. Insurance can sue because of reliance
          2. Client intention – ex) third party beneficiary
          3. Client status.

 

    1. MISREPRESENTATION
      1. Restatement 98 – a lawyer may not knowingly make false statements of material fact or law to nonclients or fail to disclose information where required by law.
      2. Silent with duty to disclose may violate
      3. Even if not duty – if you undertake to speak about something, you have a duty to provide complete and non-misleading information with respect that that subject.
      4. MR 4.1 TRUTHFULNESS IN STATEMENTS TO OTHERS
        1. In the course of representing a client a lawyer shall not knowingly:
          1. (a) STATEMENT- make a false statement of material fact or law to a third person; or
          2. (b) SILENCE – fail to disclose a material fact when disclosure is necessary to avoid assisting a criminal or fraudulent act by a client, unless disclosure is prohibited by Rule 1.6.
        2. Prohibiting fraud. Lawyer are not allowed in the course of representing a client, from knowingly making a false statement of material la w or fact or failing to disclose a material fact when disclosure is needed to avoid assisting criminal or fraudulent conduct
        3. Affirmative duty to disclose to client
        4. Silence, absent a duty to third party is not misleading
        5. You can create a duty to tell the truth 4.1(b) – you may create a duty that would not other side exists by undertaking the speech when you don’t have to. When you say party truth, you are required to finish it. If you make a statement, you are treated as a person making a statement and the standard fraud analysis applies to you: that means if you disclose, you must speak the whole truth.
        6. 4.1(b) tells you you have a duty to disclose material facts when disclosure is necessary to avoid assisting a criminal or fraudulent act by a client unless disclosure is prohibited by 1.6.
          1. 1.6(2)(3) – can disclose to prohibit physical harm.
        7. Noisy withdraw –– technique to withdrawal and disaffirm statements without saying no more – allowed by 4.1

 

      1. Cicone v. URS Corp., 183 Cal. App. 3d 194 (1986) —Liable to third parties, including adversary for misrepresentation and fraud. You generally don’t owe a duty to third parties, but you CAN create a duty by telling third party partial truths. Restatement 98 – Lawyer can be held responsible from communicating with a non-client on behalf of a client and fail to disclosure information required by law or partial truth.
        1. Facts
          1. Lawyer for buyer tells seller and seller’s atty that the buyer balance sheet is only based on best knowledge.
          2. Seller and sellers knowledge says they will accept on basis that you know nothing to contradict them. In reality seller and seller’s lawyer is treating them as a guarantee
          3. Deal falls through. Seller sues buyer – buyer sues his lawyer and law sues seller’s lawyer based on guarantee. Canady had said you were not going to treat this as a guarantee but you did!
        2. Holding
          1. Everyone owes everyone a duty not to defraud.
          2. It is fraudulent misrep because Canady said would “deem guarantee to best of seller’s knowledge,” yet enforced the warranty clause in K. Promise without the intention of performance is a misrepresentation of fact
          3. Don’t get to lie in practice of law; doesn’t matter if guy suing you is opposing L.

 

      1. A Duty to Correct Your Opponent’s Mistake? Yes – and allowed within 1.6 to prevent client from accomplishing fraud with you services.
        1. If you make a mistake to third party of behalf of client – you must correct within parameters of R. 1.6. Since 1.6(b)(2) allows disclosure to prevent the client from accomplishing a fraud using your services, disclosure is permitted.
        2. If you notice a mistake or omission – you must notify the other side. By not disclosing you might raise “a serious question violation of the duty” under Rule 1.2(d) not to assist a client in fraudulent conduct. Also 4.1(b) requires disclosure (within confined of Rule 1.6) to avoid assisting a client in committing a fraudulent act.
        3. Rule 8.4 © forbids lawyers from dishonest or fraudulent conduct.
        4. What do you do if client tells you not to say anything? 1.16(a) 0 you much withdraw if continue representation would lead to violation of the rules. Fraud can occur without anyone suffering damages or relying on the misrepresentation of failure to inform under 1.1.

 

    1. Secondary liability: aiding and abetting and conspiracy
      1. 3RD PARTIES WHO FEEL WRONGED BY CLIENT MAY ACCUSE LAWYER OF ASSISTING THE CLIENT IN UNLAWFUL CONDUCT.
      2. LAWYERS ARE PRIVILEDGED FROM AIDING AND ABETTING TORT LIABILITY WHEN THEY ARE ADVISING A CLIENT, BUT ONLY IF THAT ADVICE IS WITHIN THE SCOPE OF THE ATTY-CLIENT RELATIONSHIP. (CIVIL LIABILITY HAS A PRIVILEGE) IF YOU CONSCIOUSLY AVOID ACQUIRING KNOWLEDGE, YOU ARE CHARGED WITH THE KNOWLEDGE THAT A REAOSNABLE INQUIRY WOULD HAVE GIVEN YOU. (CRIMINAL LIABILITY HAS NO PRIVILEGE TO COMMIT A CRIME)
      3. Usually fall between advising and unlawful conspiring – Reynolds v. Schrock, 142 P.3d 1062 (Ore. 2006)
        1. Facts
          1. Physician has affair with patient (schrock) they buy property together. They decide they will divest themselves of one piece of property and they will send the second property to Schrock.
          2. Patient asks lawyer whether she can sell property or must she wait for it to appreciate and lawyer says she can sell. However, patient has fiduciary duties to physician and violated them by selling the property.
          3. Patient sues lawyer because lawyer’s advice triggered the breach of fiduciary duties.’
          4. Restatement of torts – 876 – For harm resulting to a third person from the tortuous conduct of another, one is subject to liability if he
            1. a – does a tortuous act in concert with the other or pursuant to a common signed with him or
            2. b. knows that the other’s conduct constitutes a breach of duty and give substantial assistance of encouragement to the other so to conduct himself.
            3. c. Gives substantial assistances
        2. Holding – EXCEPTION IF A LAWYER IS ACTING ON BEHALF OF A CLIENT AND WITHIN THE SCOPE OF THE LAWYER-CLIENT RELATIONSHIP.
          1. TORT LIABILITY FOR HELPING CAUSE HARM – BREACH OF FIDUCIARY DUTY– UNLESS ACTING WITHIN SCOPE OF LAWYERING FUNCTIONS. Lawyers will usually not be liable for correctly advising what their client may do under the law.
          2. *Privilege extends to act done in the course or scope of representative. If this had been outside of the lawyering, he could be punished.
          3. The court determines this is within the scope of lawyering and is therefore privileged. L not liable to physician because was acting w/in scope of lawyering, thus no tort aiding/abetting liability.
          4. Here is attny construing a document, Ct thinks is w/in lawyering, thus privilege.
          5. If outside capacity as L, then tort liability here, because seems to be substantial assistance to make patient breach a FD (seems like joint venture situation).

 

    1. Advising or assisting clients in unlawful activity
      1. Rule 1.2(d)d) A lawyer shall not counsel a client to engage, or assist a client, in conduct that the lawyer knows is criminal or fraudulent, but a lawyer may discuss the legal consequences of any proposed course of conduct with a client and may counsel or assist a client to make a good faith effort to determine the validity, scope, meaning or application of the law.
        1. CANNOT COUNSEL OR ASSIST A CLIENT IN COMMITING A CRIME OR FRAUD
        2. You should not cave into what the client wants to do.
      2. United States v. Sarantos, 455 F.2d 877 (2d Cir. 1972)
        1. Facts
          1. Lawyer is helping people set up marriages for the purpose of obtaining immigrant document. Lawyer says couples just have to fill out form and they will get a green card. Sometimes they execute divorce papers simultaneously with the immigration papers.
        2. Holding
          1. Ostrich instruction – D knew without direct evidence of knowledge. If jury finds that D consciously disregarded facts that would have given him the knowledge, then you are charged with knowing facts that you AVOID knowing. If you see a red flag and you avoid it and stick your head in the sand, you will be charged with knowledge of the red flag as if you had it completely. It is sufficient to demonstrate guilt beyond a reasonable doubt on the mens rea element of a crime. If you stick your head in the sand, the only difference between knowing that and the fact you avoid it is that you wont know it and you will be treated as if you do and you wont be able to do anything about it.
          2. In other words, ostriches bury their heads in the sand so they will not hear or see bad things. They deliberately avoid acquiring unpleasant knowledge. The ostrich instruction is designed for cases in which there is evidence that the defendant, knowing or strongly suspecting that he is involved in shady dealings, takes steps (either mental or physical) to make sure that he does not acquire full or exact knowledge of the nature and extent of those dealings. A deliberate effort to avoid guilty knowledge is all the guilty knowledge the law requires to convict.
          3. In this case aiding and abetting and knowledge because he intentionally avoided truth.Conscious efforts to avoid knowledge are the same as knowledge.
      3. YOU MAY NOT ADVISE CLIENTS TO BREAK THE LAW OR ASSIST THEM IN DOING SO, BUT YOU MAY TEST THE LEGALITY OF LAWS OR COURT ORDERS.
        1. Matter of Scionti, 630 N.E. 2d 1358 (Ind. 2001)
          1. Facts:
            1. Gary and wife divorced. Gary had custody every other weekend. Wife charged with child molestation.
            2. Client will always ask what should I do, not what the law is.
            3. Lawyer tells husband to keep child against custody agreement and court order.
            4. The court found husband in contempt and he was incarcerated.
          2. Holding
            1. The court found that the lawyer counseled his client to engage in conduct that he knew was criminal, thereby violating 1.2(d). Comment to 1.2(d) says: “A lawyer may counsel a client to make a good faith effort to determine the validity, scope, meaning or application of the law.”
            2. Lawyer also violated 8.4(d) by engaging in conduct that is prejudicial to the administration of justice.
              1. Comment to 8.4(d) provides: A lawyer may refuse to comply with an obligation imposed by law on a good faith belief that no valid obligation exists.
            3. L gets public reprimand for counseling C to break law (not return child).  C also liable for own conduct even if due to L’s bad advice. L gets off easily because no bad motivation to profit; was caring about C’s well being.  If bad intent, probably disbarred.
        2. In re Evans (Ind. 2001), the attorney was disciplined for filing a tax return he knew to be false. The client misstated the profit from a real estate transaction in which the lawyer was involved.
        3. Attorney Grievance Comm’n of Maryland v. Culver (Md. 2004), Counsel represented a client who fell behind in paying fees. The lawyer advised the client to open credit card accounts, draw down cash advances to pay the fees, and then declare bankruptcy (in which the lawyer offered to represent him).

 

    1. Some Problems In Advising Clients: Tax Shelters and “Torture Memos” – CLIENTS WANTS TO OBEY THE LAW AND KNOW HOW TO DO IT, THEY DON’T WANT LEGAL BS
      1. *Everything in this country becomes a legal question – CIA has created a sense of legalism
      2. Severe mental pain or suffering is not a term of act just because it is in a statute. It means what is says. Bordering on bad faith to act like it is some obscure legal term
      3. You are not supposed to look for furthest defensible spot just because client ask you too.
      4. Model Rule 2.1 – distinguishes between incompetent performance and poor judgment
  1. CHAPTER 8 – ASSUMING DUTIES TO PERSPECTIVE CLIENTS
    1. Relationship between lawyer and clients is based on asset. Under Restatement §15 “an agency relationship exists only if there has been a manifestation by the principal (client) to the agent (lawyer) that the agent may act on his account and consent by the agent to act.
      1. Lawyer assumes duties to clients through acts that give clients reason to believe lawyer agrees to assume duty.
      2. Does not rest of contract law so can be created without consideration. Payment of fees is neither a necessary nor sufficient condition to create duties running from a lawyer to a client.

 

    1. Duty of Loyalty (Confidentiality) – PERSPECTIVE CLIENTS
      1. Confidences received from prospective clients who don’t hire you
        1. MR 1.18 = Duties to Prospective Client
          1. (a) A person who discusses with a lawyer the possibility of forming a client-lawyer relationship with respect to a matter is a prospective client.
          2. (b) Even when no client-lawyer relationship ensues, a lawyer who has had discussions with a prospective client shall not use or reveal information learned in the consultation, except as Rule 1.9 would permit with respect to information of a former client.
          3. (c) A lawyer subject to paragraph (b) shall not represent a client with interests materially adverse to those of a prospective client in the same or a substantially related matter if the lawyer received information from the prospective client that could be significantly harmful to that person in the matter, except as provided in paragraph (d). If a lawyer is disqualified from representation under this paragraph, no lawyer in a firm with which that lawyer is associated may knowingly undertake or continue representation in such a matter, except as provided in paragraph (d).
          4. (d) When the lawyer has received disqualifying information as defined in paragraph (c), representation is permissible if:
            1. (1) both the affected client and the prospective client have given informed consent, confirmed in writing, or:
            2. (2) the lawyer who received the information took reasonable measures to avoid exposure to more disqualifying information than was reasonably necessary to determine whether to represent the prospective client; and
              1. (i) the disqualified lawyer is timely screened from any participation in the matter and is apportioned no part of the fee therefrom; and
              2. (ii) written notice is promptly given to the prospective client.

 

        1. When client’s shop around for lawyer – “prosective client” under MR 1.18 and therefore counsel is prohibited from revealing his/her information or using it except as permitted by Rule 1.9
          1. Consent, general known, etc.
          2. Rule 1.18 is easier on lawyers than Rule 1.9, because Rule 1.18(c) provides for disqualification only when a conflict is likely to be serious, and Rule 1.18(d) allows a firm to oppose the potential client if the lawyer who obtained the information can be screened effectively
          3. Rule 1.8(d) allows a firm to represent a client adverse to the potential client if the layer who obtained the information from the potential client can be screen effectively.
        2. ABSENT SOME ACTION ON LAWYER PART, THE DFAULT ASSUMPTION IN SUCH CASES IS THAT THE CLIENT WILL EXPECT TO KEEP CONFIDENTIAL WHAT IS SAID DURING INTERVIEW – unless consent or written waiver.
        3. Bridge Products, Inc. v. Quantum Chem. Corp. (Ill. 1990)
          1. Facts
            1. Bridge Products had to change lawyers while it was in litigation against Quantum.
            2. Bridge interviewed four firms. Bridge interviewed Sidley and had an initial meeting and discussed several aspects of the case including the opponents theory of the case.
            3. Bridge ended up hiring Katten and Quantum ultimately hired Sidley.
            4. Bridge moved to disqualify Sidley and the courts granted the motion.
          2. Holding
            1. A part seeking to establish an implied AC relationship much show
              1. He submitted confidential information to a lawyer and
              2. That he did so “with reasonable believe that lawyer was acting as party’s attorney
            2. Then analyze if substantially related and if firm’s efforts to screen preclude disqualification
            3. Though there was no formal attorney client relationship, there was an implicit one derived from the nature of the initial interview
            4. The confidential information was disclosed to Sidley by Bridge. Bridge must merely raise an evidentiary inference that actual confidences were disclosed which it did here. The fact that Bridge interviewed with other firms does not destroy the confidentiality.
            5. Focus is on what client believes at the time. Lawyer had the burden of making clear to Bridge that the initial meeting was purely preliminary and that confidences would not be protected, but it did not. So it is the lawyer that must pay for the confusion by being deemed part of an implicit professional relationship and all the ethical responsibilities arising therefrom.

 

      1. Internet Communications *reduction of transaction costs*
        1. Barton v. United States District Court, 410 F.3d 1104 (9th Cir. 2005)
          1. Facts
            1. A law firm posted a questionnaire online seeking information from users of Paxil. It appeared that they were trolling for potential clients for a class action for those who used the drug but they explicitly disclaimed the formation of any attorney client relationship on the form.
            2. DC ordered plaintiff’s to produce the answers to the questionnaire and plaintiff’s moved the production order vacated and confidential.
          2. Holding
            1. Prospective clients’ communications with a view to obtaining legal services are plainly covered by the attorney-client privilege under CA law, regardless of whether they have retained the lawyer, and regardless whether they ever retain the lawyer. Can be priviledged even if the relationship does not ensue.
            2. The privilege does not apply where the lawyer has specifically stated that he would not represent the individual and in no way wanted to be involved in the dispute, but the law firm did not do that in this case, it just made it clear that it did not represent the submitter yet.
            3. Where the communication between a lawyer and possible client proceeds beyond initial or peripheral contacts to acquisition by a lawyer of information that would be confidential were there to be representation, the privilege applies.
            4. The disclaimer does not disclaim the purpose of securing legal advice. The form would lead a person to think the firm would include him in the class action mentioned at the beginning of the form.
            5. The court therefore does not allow the disclosure of the questionnaires.
            6. PRIVILEDGE WOULD NOT APPLY WHERE THE LAWYER HAS SPECIFICALLY STATED THAT HE WOULD NOT REPRESENT THE INDIVIDUAL AND IN NO WAY WANTED TO BE INVOLVED IN DISPUTE – MUST USE PLAIN-LANGUAGE REFERENCE.
        2. Corpac opinion 2005-168 – hypothetical in which wife submitted confidential information to the firm through its website. The website included terms like “I agree that I am not forming an atty-client relationship by submitting this question. I also understand that I am not forming a confidential relationship.” The opinion concluded that the firms page invited the wife to consult with the firm. Even assuming the disclaimer prevented formation of atty-client privilege, the terms did not prevent a duty of confidentiality from arising on the facts because the law firms disclosure to wife were no adequate to defeat her reasonable belief that she was consulting law firm for the purpose of retaining law firm. An atty-client relationship is not a prerequisite to a lawyer assuming a duty of confidentiality in such a situation.
          1. HAD LAW FIRM WRITTEN ITS AGREEMENT WITH WIFE WITH A PLAIN-LANGUAGE REFERENCE THAT HER SUBMISSION WOULD LACK CONFIDEINTALITY, THEN THAT WOULD HAVE DEFEATED A REASONABLE EXPECATION OF CONFIDENTLIATY.

 

      1. Confidences From Parties Related to Clients
        1. The duty of confidentiality extends to person – even non-clients – who provide confidences to you under circumstances justifying them in believe you will keep the information confidential.
        2. Presumed to have confidences about people you have worked for. You should make a list of these people. The list goes further – any body about whom or from whom you have confidential information whether they are a client or not, if they are justified in thinking you will keep the information confidential.
        3. Westinghouse Elec. Corp. v. Kerr-McGee Corp., 580 F.2d 1311 (7th Cir. 1978)
          1. Facts
            1. Large law firm representing API – law firm does opinion letter after talking to all individual companies in oil industry.
            2. Westinghouse feels contract are commercially unreasonable based on law firm’s option letter.
            3. Westinghouse wants same large law firm to pursue claim against API relating to anti-trust and hired law firm
            4. However, law firms letter for API saying there is a lot of competition.
            5. API moves to disqualify K&E. Does not matter that different branches are doing the cases.
          2. Holding – Disqualifying confidences can arise from non-clients.

            1. The fact that the two contrary undertaking by the firm occurred contemporaneously, with both involving substantial steaks and substantially related to each other, outbalances the client’s interest in continuing with its chosen attorney.
            2. There is a conflict with Westinghosue and API so either disqualify lawyer or dismiss the API members that gave confidential information.
            3. Although the companies themselves did not become clients – a relationship was formed when the companies disclosed confidential information, reasonably believes that the law firm would keep it confidential. There is no need for an express agreement of representation. This is a conflict based on confidential information.
    1. Duty of Care – PERSPECTIVE CLIENTS – Bad because more than disqualification as punishment
      1. YOU OWE A DUTY OF CARE IF AND TO THE EXTENT YOU UNDERTAKE TO ADVISE PEOPLE ABOUT THEIR MATTERS. THIS IS TRUE EVEN IF YOU DON’T ACCEPT A CASE.
      2. Togstad v. Veseley, Otto, Miller & Keith, 291 N.W.2d 686 (1980)
        1. Facts:
          1. Evidence of medical malpractice. Husband paralyzed on right side of body when he was admitted to hospital for aneurism and doctor put clamp on vessel to his brain.
          2. Wife consulted Miller about medical malpractice and Miller told her he didn’t think she had a case, so she did not consult any other attorneys and the SOL expired.
        2. Holding: Breach of Care – Should have advised her to get second opinion and on the SOL
          1. The lawyer must hear/read himself the way the audience does. You owe a duty of care to the extent that you give advice. By absolutely clear and unambiguous to avoid potential problems.
            1. He said “our firm is not interested in your case.”
            2. She heard “you do not have a case”
          2. Breach of Care – there was a mistake and that mistake cause harm = malpractice. There is no requirement of written agreement or payments for duty of care to attach
          3. Togstad went to Miller for advice, he told her there wasn’t a case, and she relied on the advice in failing to pursue a claim for medical malpractice. He did not qualify his opinion by urging her to seek advice from another attorney and he was negligent in not checking his medical records which an ordinary prudent attorney would have done.
      3. Flatt v. Superior Court, 9 Cal. 4th 275 (1995)
        1. Facts:
          1. Daniel went to see Flatt to see if he had a legal malpractice claim against a lawyer who represented him in his divorce. Flatt told him he definitely had a case.
          2. Later Flatt discovered his firm had previously represented the D lawyer and told Daniel the firm could not represent him
            1. This is the wrong sequence – should have done a background check before!
          3. Flatt did not warn him about the statute of limitations.
          4. Daniel sued Flatt claiming she breached a duty of care to him by not warning of the limitations period.
        2. Holding: No duty to Daniel because duty of loyalty to previous atty, doctor.
          1. The SC held that there was an attorney-client relationship.
          2. Based on the duty to their previous client (the lawyer) Flat had no duty to give advice to Daniel about the statute of limitations because to do so would have run counter to her duty of loyalty to the attorney. She also did not have a duty to tell Daniel to seek another attorney promptly for the same reason (although if there had been no duty of loyalty, she would have had to do so).
          3. Correct in doctrine and analysis because she had a duty to the previous client – not Daniel.
          4. PROF DOES NOT LIKE THIS OPINION
          5. FN – but if anyone got hurt – this might change!
        3. Dissent: They argue that the duty of loyalty to the lawyer did not absolve Flatt of the duty of care to Daniel. The client did not create the conflict the lawyer did, by letting P tell him the information before running a conflicts check. Therefore D owed P a duty to tell him to promptly find representation and about the SOL.
        4. CORRECT ORDER
          1. Pre – Get information and run a conflicts check – MR 1.7
          2. Representation – Act competently
          3. Post – duty of confidentiality continues – MR 1.9
      4. Delso v, Trustees for the Retirement Plan for the Hourly Employees of Merck & Co., Inc. (NJ 2007). – An attorney client relationship begins with a non-lawyer’s reliance on the professional skills of an attorney, who, in turn, knows of this reliance and accepts responsibility for it. Held that an attorney who agreed to research and “ghostwrite” owed duty of care.

 

    1. Limitations on the Ability to Refuse to Assume Duties
      1. THE LAW DOES NOT ALLOW LAWYERS TO DENY SERVICES BASED SOLELY ON A POTENTIAL CLIENT’S GENDER. LAW OFFICES ARE “PUBLIC PLACES” AND SOLICIT THE BUSINESS OF CLIENTS SO ARE THEREFORE COVERED UNDER ANTI-DiSCRIMINATION LAWS.
        1. Stropinicky v. Nathanson, 19 M.D.L.R. 39 (1997)
          1. Facts
            1. Attorney Nathanson told Stropinicky that she did not represent men in divorce proceedings. She said that the issues that arise in representing women are different than those that arise in the representation of men and that she sought to devote her expertise to eliminating gender bias in the court system. She also said she feels a personal commitment to women.
            2. She said in proceedings not involving controversies between men and women, she has no problem representing men.
          2. Holding: Unlawful Disrimination
            1. The court concludes that the law does not allow an attorney to deny service based solely on a potential client’s gender. An attorney or law office holding itself out as open to the public may not reject a potential client solely on the basis of gender or some other protected class.
            2. Other notes: The court would have allowed her to reject the client if she had concluded that the issues raised by the divorce were not consistent with her specialty and area of interest, instead of just rejecting him because he was a man.
      2. Wishnatsky v. Rovner, 433 F.3d 608 (8th Cir. 2006) – public clinic cannot exclude persons from the program solely on the basis of their viewpoint. It would be a violation of the client’s constitutional right to free speech.

 

    1. Advertising and Solicitation
      1. Advertising
        1. Model Rule 7.1 A lawyer shall not make a false or misleading communication about the lawyer or the lawyer’s ser vices. A communication is false or misleading if it contains a material misrepresentation of fact or law, or omits a fact necessary to make the statement considered as a whole not materially misleading.
        2. Model Rule 7.2: – Permits advertising but not payment by a lawyer for an endorsement
        3. In Re RMJ (US 1982). The court held that advertisements in bad taste that were not misleading could not be limited.

 

      1. Solicitation
        1. Model Rule 7.3:a lawyer shall not solicit professional employment from a prospective client when a significant motive is pecuniary gain UNLESS the person contacted is another lawyer or someone with a close relationship or prior professional relationship with the garget of the solicitation
        2. Ohralik v. Ohio State Bar Association (US 1978). In this case, a lawyer was sanctioned for an in-person solicitation of two victims of an accident, one in a hospital bed. The Supreme Court held that Ohio could ban in person solicitation to prevent coercion or overreaching.
        3. In re Primus (US 1978). Women in South Carolina sterilized as a condition for receiving further welfare. Primus said the women might have a suit and the ACLU agreed to pay Primus for her services as an attorney. Williams decided to sue and Primus wrote a letter to Williams informing her of the ACLU’s offer. The supreme court held that Williams’s letter was not a violation. It was not solicitation for pecuniary gain and she offered her services for free. Her actions were undertaken to express personal political beliefs and to advance the civil liberties objectives of the ACLU rather than to derive financial gain.
        4. Shapero v. Kentucky Bar Association (US 1988). The Court held that KY could not prevent a lawyer from sending solicitation letters to targeted homeowners facing foreclosure proceedings. Brennan’s opinion said: “the relevant inquiry is not whether there exist potential clients whose condition makes them susceptible to undue influence, but whether the mode of communication poses a serious danger that lawyers will exploit any such susceptibility.
        5. Ibanez v. Florida Department of Business and Professional Regulation (US 1994). The Court held that FL could not discipline an attorney who was also a CPA and certified planner and who listed these certifications on her business cards, letterhead, and telephone listings.
        6. Peel v. Attorney Disciplinary Commission of Illinois (US 1990). Held that Ill. could not discipline an attorney for proclaiming that he was “certified as a civil trial specialist by the National Board of Trial Advocacy.” In this case and Ibanez, the court noted that there was no evidence that the statements were misleading and rejected the argument that such statements generally may be regulated as inherently misleading.
      2. States can regulate solicitation more closely than advertising
        1. Regulating commercial speech: Board of Trustees of the State University of NY v. Fox, regulations of commercial speech are permissible if the speech at issue is: (i) protected speech, which means it concerns lawful conduct and is not misleading; (ii) the government interest in regulating speech is substantial; (iii) the regulation directly advances the government’s interest; and (iv) there is a reaosnable fit between the scope of the regulation and the scope of the interest. The aim is to prevent fraud or overreaching.
        2. Zauderer v. Office of Disciplinary Counsel (US 1985). Court rejected a state’s claim that the dignity of the profession is a substantial state interest that could justify advertising restrictions.

 

    1. On (Not) Advancing Clients Money
      1. Model Rule 1.8(e) – You cannot lend clients money before they retain you. If they hire you, then you can leand them money as long as its in writing and they repay you. CAN LOAN AFTER BEING HIRED.
        1. Advancing client’s litigation cost = contingency = OK
        2. Advancing living expenses = maintenance = BANNED.
        3. Reasoning – So lawyers do not have to participate in auctions to obtain clients.
      2. Oklahoma Bar Ass’n v. Smolen, 17 P.3d 456 (2000)
        1. Facts:
          1. Workers compensation case. Client’s house burns down and is going to move and will need a new lawyer.Attorney Smolen loaned client money for living expenses so that she would not have to move and could thus hired him.
        2. Holding – Disciplined – MR 1.8(e) forbids advances for living expenses. This prevents clients selecting lawyers for improper factors. Court found that lawyer violated 1.8(e).

 

      1. Alternative Funding Mechanisms
        1. In some jurisdictions, lawyer can direct clients to third parties who fund litigation through a cash advance. The advances do not have to be repaid unless P receives money from the case.
        2. ABA Formal Opinion 431 – 2004 –criminal defense counsel may post bond for a client or arrange for it to be posted so long as the personal interest implicated by doing so presents no risk of limiting the lawyer’s ability to represent

 

  1. CHAPTER 9 – TERMINATION DUTIES – REQUIRES CLEAR, UNQUIVOCAL AND TANGIBLE TERMINATION OF DUTIES AND AC RELATIONSHIP
    1. Model Rule 1.16(A) – (D)DECLINING OR TERMINATING REPRESENTATION
      1. ((a) Except as stated in paragraph (c), a lawyer shall not represent a client or, where representation has commenced, shall withdraw from the representation of a client if
        1. (1) the representation will result in violation of the rules of professional conduct or other law;
        2. (2) the lawyer’s physical or mental condition materially impairs the lawyer’s ability to represent the client; or
        3. (3) the lawyer is discharged – fired
      2. (b) Except as stated in paragraph (c), a lawyer MAY WITHDRAW from representing a client if:
        1. (1) withdrawal can be accomplished without material adverse effect on the interests of the client;
        2. (2) the client persists in a course of action involving the lawyer’s services that the lawyer reasonably believes is criminal or fraudulent;
        3. (3) the client has used the lawyer’s services to perpetrate a crime or fraud;
        4. (4) the client insists upon taking action that the lawyer considers repugnant or with which the lawyer has a fundamental disagreement;
        5. (5) the client fails substantially to fulfill an obligation to the lawyer regarding the lawyer’s services and has been given reasonable warning that the lawyer will withdraw unless the obligation is fulfilled;
        6. (6) the representation will result in an unreasonable financial burden on the lawyer or has been rendered unreasonably difficult by the client; or
        7. (7) other good cause for withdrawal exists.
        8. WILL NEED TO GET PERMISSION OF TRIBUNAL AND MUST NOT VIOLATE MR 1.6. JUST SAY “IRRECONCILABLE DIFFERENCES”
      3. (c) A lawyer must comply with applicable law requiring notice to or permission of a tribunal when terminating a representation. When ordered to do so by a tribunal, a lawyer shall continue representation notwithstanding good cause for terminating the representation.
      4. (d) Upon termination of representation, a lawyer shall take steps to the extent reasonably practicable to protect a client’s interests, such as giving reasonable notice to the client, allowing time for employment of other counsel, surrendering papers and property to which the client is entitled and refunding any advance payment of fee or expense that has not been earned or incurred. The lawyer may retain papers relating to the client to the extent permitted by other law.
    2. Restatement §31-33
      1. A client may terminate the duties of care and loyalty at will
      2. A lawyer may terminate them if
        1. The lawyer can terminate the relationship without material harm to client or
        2. There is good cause to do so, so long as the notion of good cause includes a good reason for termination.
          1. Ex) nonpamyment of fees or fundamenal disagreement over objectives
        3. Where the tribunals’s consent to termination is necessary, the tribunal gives it
      3. A lawyer must if any of the conditions in Rule 1.16(a) exist
      4. Subject to some exceptions, the duty of confidentiality survives termination.
    3. Three important points. (1) it is easier to stay out of trouble than to get out of trouble, so make sure you want to create duties before you do; (2) when you want to get out, make sure everyone knows it; and (3) make sure the client is no worse off for your leaving than the client would have been had you stayed. There must be a cealr point at which the attyc-client relationship is over!
      1. Hanlin v. Mitchelson, 794 F.2d 834 (2d Cir. 1986)
        1. Facts
          1. Client is required to pay 25k in advance. She is angry and sends letter saying you did not do a good job for me, I want my money back.
        2. Holding
          1. A client’s malpractice suit against an attorney is enough to indicate that the client has terminated the relationship.
          2. Short of instituting a suit, a client may question an attorney’s tactics and consult with another attorney without terminating the attorney client relationship.
          3. You need a clear and unequivocal and tangible communication terminating the duty. You should do as you think best.
          4. The termination of the relationship here was not so clear as to render Hanlin’s proposed amendment frivolous and, therefore, no proper basis appears for denying leave to amend. Even if the relationship had been terminated, the termination would not end the malpractice inquiry; questions would still remain about Mitchelson’s handling of the termination. To withdraw an attorney must give a client a clear and unambiguous notice of the attorney’s intent to withdraw. No such notice was given here to Hanlin.
          5. A lawyer shall not withdraw from employment until he has taken reasonable steps to avoid foreseeable prejudice to the rights of the client.

 

      1. Whiting v. Lacara, 187 F.3d 317 (2d Cir. 1999) –
        1. Facts
          1. Client is a cop who was fired and is suing for retaliatory discharge, seeking 9.9 million dollars, and hires Lacara as his attorney.
          2. Whiting was a bad client and Lacara filed a motion to withdraw as counsel, which was denied and Lacara appealed.
            1. Denied because Assumed Risk = Choose your clients with care.
            2. Managing Expectations = You have to manage your client’s expectations so they don’t hurt themselves by insisting on things the system is not set up to provide
        2. Holding
          1. Usually, motions of withdrawal will be denied if prosecution of the suit is likely to be disrupted by the withdrawal of counsel.
          2. But there are some instances in which an attorney representing P in a civil case might have to withdraw even at the cost of significant interference with the trial court’s management of its calendar.
            1. Ex: If P brings the suit merely for harassing or maliciously injuring D
          3. Court allowed This withdrawal because Whiting wanted to dictate (impermissible) legal strategies to Lacara and to sue him if those strategies were not followed.Placed Lacara in so impossible a situation that he must be permitted to withdraw
          4. 1.16(b) Except as stated in paragraph (c), a lawyer may withdraw from representing a client if:
            1. (1) withdrawal can be accomplished without material adverse effect on the interests of the client;
            2. (4) the client insists upon taking action that the lawyer considers repugnant or with which the lawyer has a fundamental disagreement;
            3. (6) the representation will result in an unreasonable financial burden on the lawyer or has been rendered unreasonably difficult by the client; or

 

      1. Haines v. Liggett Group, Inc., 814 F. Supp. 414 (D. N.J. 1993) – NOT USUALLY FOR UNREASONABLE FINANCIAL BURDEN, ESPECIALLY IF IT WILL HURT CLIENT
        1. Facts: Law firm Bud Larner sought permission to withdraw from cases for cigarette related health claims because litigation against the cigarette industry had become an unreasonable financial burden for the firm. They urge the court to look at the Cipollone case so that they can gauge how much the case will cost.
        2. Holding:
          1. Allowing withdrawal is up to the court’s discretion, even when a lawyer shows good cause.
          2. While BL has demonstrated it has expended significant resources, it has not demonstrated sufficient basis to permit withdrawal absent the consent of the client.
          3. Also, a court will refuse to permit withdrawal where withdrawal would impair a client’s ability to find substitute counsel or maintain the action, even where representation becomes unprofitable for the client’s lawyers. Even if counsel could be found, it would cause severe prejudice to the client’s interests. Also, in this case there was a contingency fee agreement, just like a client cannot break the agreement because the litigation was very easy, an attorney also cannot withdraw, even if the litigation is expensive, he is bound by the agreement. The court therefore refuses to allow withdrawal.

 

      1. There May Be Places To Get Justice In This World; This Is Not One of Them
        1. It is common for clients to want to use lawsuits to get back at people they feel have hurt them. They see their problem as victimization rather than as particular causes of action and they want satisfaction –justice, as they see it – for their pain.
        2. But the law does not offer vindication as such. Courtrooms are for resolving factual and legal disputes. They are not soapboxes for damning one’s enemies.

 

    1. Voluntary Withdrawal and Contingency Fee – LAWYER MAY WITHDRAW IF CLIENT DOES NOT PAY
      1. A lawyer may withdraw from a matter if the client doesn’t pay his bill.
      2. When appearing before a tribunal, however, a lawyer may only withdraw if the client consents.
      3. Without strict formalism – you might be allowed to say you have not been paid.
      4. Usually when a lawyer has taken the case on a contingent fee bases – consideration of fairness and risk allocation tend to favor client. Having contracted with client on contingency free basis, the lawyer cannot now walk away from the contract because the case may not generate the return expected at the outset.
      5. The basic rules of how withdrawal affect contingent fee cases
        1. Withdrawal will eliminate your entitlement to a fee unless withdrawal is justified.
        2. Justified if withdrawal is compelled by need to adhere to a mandatory ethical rule
        3. Permissive withdrawal or withdrawal because counsel thinks the case is merit-less never counts as justified. Nor does the client’s rejection of a settlement commended by counsel.
      6. Falco
        1. Recovery for services in quantum meruit is allowed only when the atty has justifiable cause for withdrawing. An atty who voluntarily abandons a case without good cause will be denied compensation.
        2. If counsel negotiated a settlement the client initially rejected and then accepted a substantially similar settlement after counsel withdrew, the client should be required to make restitution to the atty under a theory of unjust enrichment.

 

    1. IN HOUSE COUNSEL – WITHDRAWL
      1. General Dynamics Corp. v. Superior Court, 7 Cal. 4th 1164 (1994)
        1. Facts
          1. In house counsel is fired saying they lost faith in him, but it was really because in house counsel would not cover up crimes of company.
          2. Fired lawyer sues for wrongful termination
          3. Company moves to dismiss on claim that company should have both the right and the power at any time to discharge his attorney with or without cause.
        2. Holding – If you fire in house counsel without good cause – will likely have to pay money
          1. General rule does not mean that the “absolute” right of the client to discharge unilaterally his attorney permits all clients to terminate the attorney-client relationship under all circumstances without consequence
          2. While the “unfettered” right of the client to discharge his attorney at any time is an important value that should be upheld in most cases, the legal consequences of such an act will vary, depending on the strength of competing interests that are present in a particular case.
          3. Example: for in-house counsel, because unlike the law firm partner, who typically possesses a significant measure of economic independence and professional distance derived from a multiple client base, the economic fate of in-house attorneys is tied directly to a single employer, at whose sufferance they serve. Therefore, in-house counsel may not be terminated without good cause.
          4. A terminated in-house lawyer may bring a tort cause of action if (and only if) “some statute or ethical rule, such as the statutory exceptions to the attorney client privilege…specifically permits the attorney to depart from the usual requirement of confidentiality with respect to the client-employer and engage in the ‘nonfiduciary conduct’ for which he was terminated.”
          5. If the conduct was merely ethically permissible but not required, the inquiry is different and two questions must be resolved:
            1. (1) Whether the employer’s conduct is of the kind that would give rise to a retaliatory discharge action by a non attorney employee; and
            2. (2) the court must determine whether some statute or ethical rule, such as the statutory exceptions to the attorney client privilege permits the attorney to depart from the usual requirement of confidentiality with respect to the employer and engage in the nonfiduciary conduct for which he was terminated.

 

      1. Fox Searchlight Picture – In-house counsel, in a suit for wrongful termination, may disclosure employer-client confidences to the extent that they are relevant in the preparation and prosecution of the wrongful termination suit.

 

    1. The Ethical Significance of Client Diversification – The more diversified your client base, the easier it is to walk away from a client that wants you to violate a rule, and the more likely you are to do it.
        1. *In house attorneys are unusually vulnerable to coercion from their clients because they are undiversified. If income comes from only 1 source, you are more likely to listen to that one source’s commands.
        2. Diversification means you will never “pick up the papers”

 

 

  1. CHAPTER 10 – CONFLICT OF INTEREST
    1. CONCURRECT CLIENT CONFLICTS – Contemporaneous Representation
      1. INTRODUCTION
        1. You may not represent one client in a matter which that client is directly adverse to another current client, even if the two representations are completely unrelated UNLESS INFORMED CONSENT
        2. Should you choose to do business with a current client you will need to
          1. Make sure they know you did not act for them in the transaction
          2. Advise them to get another lawyer regarding the transaction and give them a chance to do it and
          3. Be prepared to demonstrate the substantive fairness of the trasncation
        3. Restatement § 121 – Unless all affected clients/other persons consent, a lawyer may not represent a client if there is a conflict of interest (substantial risk that the representation would be materially and adversely affected by the lawyer’s own interests or lawyer’s duties to another client/person)
        4. Restatement § 122even if conflict of interest, a lawyer may represent if each affected client or former client gives informed consent to the representation (informed consent = client must have reasonably adequate information about the material risks of such representation) unless:
          1. the representation is prohibited by law;
          2. one client will assert a claim against the other in the same litigation; or
          3. in the circumstances, it is not reasonably likely that the lawyer will be able to provide adequate representation to one or more of the clients
        5. Restatement § 128 – Unless all clients consent, a lawyer in civil litigation may not:
          1. represent 2 or more clients in a matter if substantial risk that representation of one client would be materially/adversely affected y the lawyer’s duties to another client in the matter; or
          2. represent one client to assert or defend a claim against or brought by another client currently represented by the lawyer, even if the matters are not related.
        6. MR 1.7 – lawyer shall not represent a client if the representation involves a concurrent conflict of interest. A concurrent conflict of interest exists if
          1. the representation of one client will be directly adverse to another client; OR
          2. there is a significant risk that the representation of one or more clients will be materially limited by the lawyer’s responsibilities to another client, a former client or a third person or by a personal interest of the lawyer
          3. Lawyer may not simultaneously represent directly adverse interests, even in unrelated matters. Directly adverse usually means parties are across the table from each other
          4. Lawyers may also not represent a client if their ability to do so is materially limited by an obligation to another current or former client, third person, or personal interest.
          5. EVEN IF THE SIMULTANEOUS REPRESENTATIONS HAVE NOTHING IN COMMON AND THERE IS NO RISK THAT CONFIDENCES TO WHICH COUNSEL IS A PARTY IS THE ONE CASE HAVE ANY RELATION TO THE OTHER MATTER – DISQUALIFICATION MAY NEVERTHELESS BE REQUIRED.
          6. The main value at stake in simultaneous representation = loyalty

 

      1. MEANING OF CURRENT CONFLICT – 1.7(a)
        1. Frequent but intermittent Representation – When you represents a client commonly (over and over) but not continuously if you have a pattern of representation over and over as extended period of time and you make no indication you are only representing them in sepciail discrete cases, courts will view you as having ongoing, continuous atty-client relationship as “outside general counsel” and they will be considered current clients = automatic DQ. To prevent, must inform clint that each case is a separate new representation and that the reltionship ends at the conclusion of each one.

 

      1. MEANING OF DIRECTLY ADVERSE
        1. Topics do not need to be directly related
        2. Direct adverse – when parties are across the table from eachother in negotiations or across the courtroom in litigation. ISSUE OF LOYALTY
        3. Does not apply to economic adversity such as competition in the marketplace
        4. Current Course of Confidential, Non Atty-Clieent Relationship Information – When a firm represents a client and has some interest not arising from an atty-client relationship but which is adverse to the client, such as when representing a client whose adverse party is bank at which a partner at the firm is on the Board of Trustees = automatic DQ (personal conflicts of lawyer are same as conflicts by concurrent clients).
        5. Issues of funds paying for lawsuit from the funds of another client– North Star Hotels Corp. v. Mid-City Hotel Assocs,118 F.R.D. 109 (D. Mn 1987)
          1. Facts
            1. Conflicted connection meant that if lawyer won C1’s case, that could take money from C2’s company finances.
            2. Mid-City hotels (defendant) moved to disqualify Faegre from representing Northstar claiming the representation created a conflict of interest with two of Faegre’s other clients (that the representation of Northstar was directly adverse to the interests of its two other clients).
          2. Holding
            1. Disqualified.
            2. In essence flowing through of liability – concern that another client of attorneys will end up paying/reduce revenue of another client. Fegre is attempting to put money in the pocket of one of his clients and this may come from the pocket of one of its other clients.
            3. Rule of table – directly adverse test/across the table + whether a judgment could control in a pass through effect which could create economic harm
            4. Because Harry Johnson is a large partner in both of the 2 partnerships represented by the firm, either way the case came out would have a serious impact on his assets. Therefore, the firm’s representation of North Star in this action is “directly adverse” to the financial interests of its two partnership clients within the meaning of Model Rule 1.7(a).
            5. Rule: A law firm simply should not seek to compensate one client from the pockets of another

 

        1. Positional” Conflicts – Legal conflicts relating to some legal rule – Rule of law rather than legal or economic interest. Usually NOT direct adversity.
          1. You may take inconsistent legal position in different tribunals at different times for different C’s Usually deemed to be representing your client’s position – not your own position. It is usually ok to choose to take irreconcilable doctrines at the same time. In practice the client will be angry because of a concern for a bad judge within judges
          2. Positional conflict. A positional conflict is a conflict arising from different positions on some legal question, rather than different legal or economic interests. The general rule is that positional conflicts do not create direct adversity for purposes of Rule 1.7(a). MR 1.7(a), comment 24 says that ordinarily a lawyer can take inconsistent legal positions at different times on behalf of different clients.
          3. However, positional conflicts can be real conflicts if there is a significant risk that a lawyer’s action on behalf of one client will materially limit the lawyer’s effectiveness in representing another client in a different case; for example, when a decision favoring one client will create a precedent likely to seriously weaken the position taken on behalf of the other client
          4. WHETHER LAWYER CAN EFFECTIVLEY ARGUE BOTH SIDES OF THE SAME LEGAL QUESTION WITHOUT COMPROMISING THE INTEREST OF ONE CLIENT OR THE OTHER.
          5. Williams v. Delaware (2002). This case is an example of a true positional conflict. A firm represented two clients both appealing their sentences for convictions that led to capital sentences. The lawyer would have argued the exact opposite theory in each of the cases. The success of one would create unfavorable precedent for the other.
            1. Holding: In determining whether a positional conflict requires a lawyer’s disqualification, the question is whether the lawyer can effectively argue both sides of the same legal question without compromising the interests of one client or the other.
            2. Decision: In this case the lawyer was disqualified due to the positional conflict.

 

      1. Truck Ins. Exchange v. Fireman’s Fund Ins. Co., 6 Cal. App. 4th 1050 (1992) – CANNOT DROP FIRST CLIENT WHO CAUSES CONFLICT.
        1. Facts
          1. Firm took a big personal injury case. Found a conflict and tried to get personal injury clients to consent. When they would not consent, they dropped the previous client’s wrongful termination case.
          2. The other side moved to disqualify
          3. Firm argued that because of the withdrawal the adverse party is a former client and so the court must do a “substantial relationship” test.
        2. Holding
          1. A firm may not play “hot potato” and drop clients in order to take up better ones who are adverse to them. Here, there was concurrent representation, and adversity, so therefore automatic disqualification is appropriate.
          2. First Client First Rule: The 1st client you represent is owed superior duty of loyalty, therefore a 2nd client cannot disqualify an attorney based on the first representation
          3. Even if the simultaneous representations have nothing in common, and there is no risk that confidences to which counsel is a party in the one case have any relation to the other matter, disqualification may nevertheless be required.
          4. Rule: Disqualification for simultaneous representation is a per se “automatic one”
          5. Reasoning: A client who learns that his/her lawyer is also representing a litigation adversary, even with respect to a matter wholly unrelated to the one for which counsel was retained, cannot be expected to sustain the level of confidence and trust in counsel that is one of the foundations of the professional relationship.

 

      1. NOT DIRECTLY ADVERSE BUT SIGNIFICANT RISK THAT THE REPRESENTATION OF ONE OR MORE CLIENTS WILL BE MATERIALLY LIMITED. LIMITATION ON COUNSEL’S ABILITY TO REPRESENT ZEALOUSLY.
        1. Introduction
          1. MR 1.7(a)(2)CONFLICT OF INTEREST: CURRENT CLIENTS – . A concurrent conflict of interest exists if (2) there is a significant risk that the representation of one or more clients will be materially limited by the lawyer’s responsibilities to another client, a former client or a third person or by a personal interest of the lawyer.
          2. Even if not directly adverse – lawyer still has a conflict if there is a significant risk that the lawyer’s ability to represent a client would be materially limited by a lawyer’s duty to another or former client, third person, or personal interest.
          3. California does not have this at this time – amendment pending before the Supreme Court.
          4. If you removed rule – courts have the inherent power to exclude/disqualify you due to conflicts. Even in California the result is the same

 

        1. Fiandaca v. Cunningham, 827 F.2d 825 (1st Cir. 1987
          1. Facts:
            1. Firm represents both women prisoner’s and mentally challenged patients who both want use to State Schools.
          2. Holding
            1. The combination of clients and circumstances placed NHLA in the untenable position of being simultaneously obligated to represent vigorously the interests of two conflicting clients.
            2. No counsel could have properly performed the role of “advocate” for both plaintiffs, regardless of its good faith or high intentions and therefore disqualifications is appropriate.
            3. Therefore, in accordance with rule 1.7(a)(2), the court disqualified NHLA. Conflict within remedy raised what an attorney was willing to argue = material limitations.

 

        1. Personal interests of prosecutors. Criminal defendants sometimes argue that prosecutors have personal interests that threaten to deny the defendant a fair trial.
          1. Haraguchi v. Superior Court (Cal. 2008)
            1. Prosecutor wrote a novel about deciding whether to prosecute a rape case involving an intoxicated victim. The book was published at the time that the prosecutor was set to proceed on a similar case. The defendant moved to recuse the prosecutor arguing that her personal interest to see the book succeed deprived the defendant of a fair trial.
            2. The court found that it was only a coincidence and would not deprive defendant of a fair trial.
            3. Note: Severeal cases have shown that lower court ruling on disqualification are gien great deference and are reviewed ONLY for abuse of direction.
          2. Hollywood v. Superior Court (Cal. 2008)
            1. Prosecutor cooperated with director on film about defendant that was still at large. Defendant was found and moved to recuse prosecutor. The court denied the motion saying the prosecutor had no financial interest in the movie by the time of trial.

 

      1. REMEDIES FOR CONCURRENT CONFLICTS
        1. Usually disqualification
          1. What happens with what you were going to charge the client? Maybe give money back
          2. What happens with work you are working on while work is disqualified – some courts might order that the lawyer cant transfer to client or new attorney. This increases the
        2. However, if does not violate 4 factors egregiously, disqualification might not be required. Disqualification is discretionary – a judges’ determination.
        3. Courts should analyze
          1. Party’s right to counsel of choice
          2. An attorney’s interest in representing a client
          3. Financial burden on a client of replacing DQ’s counsel
          4. Any tactical abuse underlying the DC order.
        4. Research Corp. Tech., Inc. v. Hewlett-Packard Co., 936 F. Supp. 697 (D. Az 1996)
          1. Facts:
            1. Atty represents a client in an unrelated tax matter. Then atty represents HP against this client in a patent case.
          2. Holding:
            1. Rule violation (Direct adversity between current clients) – However, disqualification is discretionary and judge decides not to disqualify McDermott.
            2. It looks to four factors: (1) the nature of the ethical violation; (2) the prejudice to the parties, including the extent of the actual or potential delay in the proceedings; (3) the effectiveness of counsel in light of the violations; (4) the public’s perception of the profession.
            3. It concludes that the violation is not so egregious as to require disqualification.
            4. Rare case where court holds that since cases were separate and distinct and “expectation of laytaly was not cavalierly trampled that DQ is warranted as a sanction”

 

      1. Harm vs. Risk II: Civil Liability (or Sanctions) And Concurrent Conflicts
        1. Discipline/disqualifications – requires substantial risk that lawyer’s representation of the client would be materially and adversely affected by the lawyers own interest or by the lawyers duties to another current client, a former client or third party. Harm is not requires for disqualification
        2. Civil Tort Liability for Breach of Fiduciary – Requires actual HARM
          1. Used as a narrative of betrayal in a malpractice case for causation

 

      1. Standing and Interlocutory Appeal
        1. A party that has never had an attorney-client relationship with a particular lawyer CANNOT prevail on a motion to disqualify the lawyer. A standing requirement is implicit in disqualification motions. Some sort of confidential or fiduciary relationship must have existed before a party is entitled to prevail on a motion to disqualify. THEREFORE THE OTHER SIDE CANNOT RAISE THIS ISSUE ABOUT YOU!
        2. Either a client or a disqualified lawyer has standing to appeal.
        3. Federal court requires – federal appellate court must have jurisdiction over “final decisions” on the direct court.

 

    1. FORMER CLIENT CONFLICTS – 1.9
      1. You must not
        1. Act adversely to them in a matter substantially related to the prior representation. Meaning if the lawyer could have obtained confidential information in the first representation that would have been relevant in the second.
          1. Under the substantial relationship test you are conclusively presumed to know things a reasonably competent lawyer would have learned in performing the work you did; it does not matter whether you actually learned such information. – objective, not subjective
          2. Whether a reasonably competent lawyer/court would recognize that the types of information available in the first representation would be pertinent in the second
          3. It is irrelevant whether he actually obtained such information and used it against the former client.
          4. UNLESS THE CLIENT CONSENTS
          5. NO ISSUE IF UNRELATED MATTER.
        2. Use or disclose information learned in representation in a manner adverse to the form client unless the information has become generally known
      2. THE OTHER SIDE CANNOT RAISE THIS ISSUE ABOUT YOU!
      3. Purpose: protect client confidences. We don’t want to require client’s to disclose confidences in order for them to protect them. The main value at stake when you act adversely to a former client is confidentiality

 

      1. Model Rule 1.9DUTIES TO FORMER CLIENTS
        1. (a) A lawyer who has formerly represented a client in a matter shall not thereafter represent another person in the same or a substantially related matter in which that person’s interests are materially adverse to the interests of the former client unless the former client gives informed consent, confirmed in writing.
        2. (b) A lawyer shall not knowingly represent a person in the same or a substantially related matter in which a firm with which the lawyer formerly was associated had previously represented a client
          1. (1) whose interests are materially adverse to that person; and
          2. (2) about whom the lawyer had acquired information protected by Rules 1.6 and 1.9(c) that is material to the matter; unless the former client gives informed consent, confirmed in writing.
        3. (c) A lawyer who has formerly represented a client in a matter or whose present or former firm has formerly represented a client in a matter shall not thereafter:
          1. (1) use information relating to the representation to the disadvantage of the former client except as these Rules would permit or require with respect to a client, or when the information has become generally known; or
          2. (2) reveal information relating to the representation except as these Rules would permit or require with respect to a client.
      2. Restatement §132 – unless both the affect present and former clients consent, a lawyer who has represented a client in a matter may not represent another client in the same or a substantially related matter in which the interests of the former client are materially adverse. The current matter is substantially related to the earlier matter if
        1. The current matter involves the work performed for the former client or
        2. There is substantial risk that representation of the present client will involve the use of information acquired in the course of representing the form client unless the information has becomes generally known.

 

      1. CASE EXAMPLES
        1. Analytica, Inc. v. NPD Research, Inc., 708 F.2d 1263 (7th Cir. 1983)
          1. Fact:
            1. Lawyer at firm received confidential information to grant and value stock of NPD to employee
            2. He later sought to represent analytica, NPD’s competitor in a suit for anticompetitive behavior.
          2. Holding:
            1. In this case, the information the firm obtained was highly pertinent in the subsequent representation and therefore they had to be disqualified.
            2. If lawyer could have obtained confidential information in the first representation that would have been relevant in the second, it is irrelevant whether he actually obtained such information and the lawyer must be disqualified.
            3. Even if the party had NOT RETAINED counsel, but had disclosed confidential information to them, they have a right not to see that firm reappear on the other side of the litigation to which the data might be highly pertinent.

 

        1. Maritrans, GP, Inc. v. Pepper, Hamilton (Pa. 1992) – Same industry but not in a matter directly adverse to their old client – Case by case basis – Same information covers this substantial relationship. Elements and defenses are not the only thing you look at.
          1. Facts:
            1. Pepper represented Maritrans for many years on different matters. Then it began to represent some of its competitors in labor negotiations
          2. Rule: The court will decide these cases on a case by case basis and one important factor is the extent to which the fiduciary was involved in its former client’s affairs. There is not a blanket rule preventing counsel from representing the economic competitor of a former client in matters in which the former client is not also a party to a lawsuit.
          3. Holding: Here there was a substantial relationship between Pepper’s former representation and their current representation so the injunctive relief was justified.

 

        1. Johnson v Superior Court Lawyer who formerly represented husband and wife in personal injury case Could later represent wife in divorce proceeding against husband.

 

      1. Subsequent Conflicts and Confidences Obtained Other Than Through Representation
        1. Oaks Management Corp. v. Superior Court (Cal. 2006
          1. Facts: Defendant borrowed money from a partnership in which the lawyer who represented plaintiff was a partner. Defendant argued that the partnership had acquired important information that would help the plaintiff in the litigation.
          2. Holding: The significant question is whether there exists a genuine likelihood that the status or misconduct of the attorney in question will affect the outcome of the proceedings before the court. In this case, the plaintiff already had access to the information and so the information revealed to the partnership would not have a significant impact.
          3. In CA, an appearance of impropriety is not sufficient ground for disqualification of an attorney.

 

    1. CURRENT AND FORMER CLIENT CONFLICTS IN CRIMINAL COURT
      1. UNLIKE IN CIVIL, PROSECUTOR CAN RIASE CONFLICT OF INTEREST ARGUMENT ON PART OF DEFENSE AND HAS STANDING.
      2. ACTIVE CONFLICT INEFFECTIVE COUNSEL UNDER STRICKLAND
      3. Counsel laboring under an actual conflict of interest that adversely affect their performance are presumed ineffective – prejudice need not be shown
      4. Courts may decline to allow defendants to waive conflict. Right to the counsel of your choice < effective counsel
      5. Wheat v. United States, 486 U.S. 153 (1988)
        1. Facts. 
          1. Attorney goes to trial with client A and wins. Client B then wants him to represent him but attorney intends to put client B on as witness. Client B is willing to waive conflict.
        2. Holding
          1. While the right to select and be represented by one’s preferred attorney is comprehended by the Sixth Amendment, the essential aim of the Amendment is to guarantee an effective advocate for each criminal defendant rather than to ensure that a defendant will inexorably be represented by the lawyer whom he prefers
          2. Trial courts “must be allowed substantial latitude in refusing waivers of conflicts of interest not only in those rare cases where an actual conflict may be demonstrated before trial, but in the more common cases where a potential for conflict exists which may or may not burgeon into an actual conflict as the trial progresses.”
          3. Sixth Amendment right to choose one’s own counsel is often circumscribed by the right to effective assistance of counsel. Right to choose own counsel < right to effective counsel.
          4. Particularly, as in this case, it is at the discretion of the District Court to determine whether certain counsel could offer effective representation and, should the court decide it cannot, the court has the right to abrogate the defendant’s wishes.
      6. Conflicts of Interest and the Strickland Standard of Ineffective Counsel
        1. Cuyler v Sullivan – 1980
          1. Decided before Strickland. Sulivant was covicted and his attorneys defended two other men for the same trial. He claimed his attorneys fulled their punches at trial to help the otwo other mean and dnied him effective assistance of counsel.
          2. Holding – trial judges are not obligated to inquire into whether counsel jointly repreenting defedants had a conflict of interest and court will not presume prejudice except in the presumably rare case where D raises the issue and court refuses to consider it. Since he did not raise is at trial he must prove an actual conflict of interest adversely affected his lawyer’s performance.
          3. This standards was adopted in Strickland
        2. Mickens v Taylor – 2002
          1. Lawyer represented D on trial for murdering of the lawyer’s other clients. Conflict but counsel did not raise this conflict. Sullivan test applied and D counsel show that conflict of interest adversely effected his counsel’s performance.

 

    1. Imputation of Knowledge and Screening
      1. You are presumed to know what a competent lawyer would know about the matters you work on regardless of what your actual knowledge is. There is an objectively defined set of information that attached to every piece of work you do. This tends to be information that someone acting competently would have acquired. This information is then imputed to everyone working at your firm.
      2. GENERAL RULE 1.10
        1. A lawyer tainted by a conflict taints the new firm. If a lawyer personally worked for a client at the former firm on a matter substantially related to a matter the new firm wants to take on, the new firm will be disqualified.
        2. Rule 1.10 – allows rebuttable presumption that attorney for a firm does not disclose information that would affect the firm’s representation in a case if screening is done
        3. The train of conflict generally run from lawyer to firm but not firms to lawyer. Although lawyers are presumed to know what their firm know, this presumption is rebuttable, so a lawyer need only submit a declaration averring that he had no contract with the substantially related matter while at this old firm
        4. Model Rules 1.10 IMPUTATION OF CONFLICTS OF INTEREST: GENERAL RULE
          1. a) While lawyers are associated in a firm, none of them shall knowingly represent a client when any one of them practicing alone would be prohibited from doing so by Rules 1.7 or 1.9, unless
            1. (1) the prohibition is based on a personal interest of the disqualified lawyer and does not present a significant risk of materially limiting the representation of the client by the remaining lawyers in the firm; or
            2. (2) the prohibition is based upon Rule 1.9(a) or (b) and arises out of the disqualified lawyer’s association with a prior firm, and
              1. (i) the disqualified lawyer is timely screened from any participation in the matter and is apportioned no part of the fee therefrom;
              2. (ii) written notice is promptly given to any affected former client to enable the former client to ascertain compliance with the provisions of this Rule, which shall include a description of the screening procedures employed; a statement of the firm’s and of the screened lawyer’s compliance with these Rules; a statement that review may be available before a tribunal; and an agreement by the firm to respond promptly to any written inquiries or objections by the former client about the screening procedures; and
              3. (iii) certifications of compliance with these Rules and with the screening procedures are provided to the former client by the screened lawyer and by a partner of the firm, at reasonable intervals upon the former client’s written request and upon termination of the screening procedures.
          2. (b) When a lawyer has terminated an association with a firm, the firm is not prohibited from thereafter representing a person with interests materially adverse to those of a client represented by the formerly associated lawyer and not currently represented by the firm, unless:
            1. (1) the matter is the same or substantially related to that in which the formerly associated lawyer represented the client; and
            2. (2) any lawyer remaining in the firm has information protected by Rules 1.6 and 1.9(c) that is material to the matter.
          3. (c) A disqualification prescribed by this rule may be waived by the affected client under the conditions stated in Rule 1.7.
          4. (d) The disqualification of lawyers associated in a firm with former or current government lawyers is governed by Rule 1.11.

 

      1. Model rule 1.11 and CA allow screening for government lawyers (prosecutors) moving to private practice and the rules allow for screening in private practice (not CA yet).
        1. If the new firm screens the lawyer from the matter, meaning that the lawyer does not work on the matter and is prevented from learning about or discussing it with other lawyers at the firm, the firm can defeat a DQ presumptions.
        2. Model Rules 1.1SPECIAL CONFLICTS OF INTEREST FOR FORMER AND CURRENT GOVERNMENTOFFICERS AND EMPLOYEES
          1. (a) Except as law may otherwise expressly permit, a lawyer who has formerly served as a public officer or employee of the government:
            1. (1) is subject to Rule 1.9(c); and
            2. (2) shall not otherwise represent a client in connection with a matter in which the lawyer participated personally and substantially as a public officer or employee, unless the appropriate government agency gives its informed consent, confirmed in writing, to the representation.
          2. (b) When a lawyer is disqualified from representation under paragraph (a), no lawyer in a firm with which that lawyer is associated may knowingly undertake or continue representation in such a matter unless:
            1. (1) the disqualified lawyer is timely screened from any participation in the matter and is apportioned no part of the fee therefrom; and
            2. (2) written notice is promptly given to the appropriate government agency to enable it to ascertain compliance with the provisions of this rule.
          3. (d) Except as law may otherwise expressly permit, a lawyer currently serving as a public officer or employee:
            1. (1) is subject to Rules 1.7 and 1.9; and
            2. (2) shall not:
              1. (i) participate in a matter in which the lawyer participated personally and substantially while in private practice or nongovernmental employment, unless the appropriate government agency gives its informed consent, confirmed in writing; or
              2. (ii) negotiate for private employment with any person who is involved as a party or as lawyer for a party in a matter in which the lawyer is participating personally and substantially, except that a lawyer serving as a law clerk to a judge, other adjudicative officer or arbitrator may negotiate for private employment as permitted by Rule 1.12(b) and subject to the conditions stated in Rule 1.12(b).

 

      1. SCREENING – The model rules do not adopt a general screening provision. However, MR 1.11 allows for screening for government attorneys who move to private practice, so long as the firm promptly notifies the government agency in question. Failure to notify can forfeit the protection screening procedures might otherwise provide. US v. Philip Morris, Inc. (DDC (2204).
        1. In re Complex Asbestos Litigation (Cal. 1991). Screening also works to prevent disqualification where a nonlawyer employee has confidential information.

 

 

 

      1. CHART – presumed to know what a competent lawyer would have known – actual knowledge trumps
        1. A’S OLD PARTNER B– A has info – B leaves– could represent that subject matter in new firm if
          1. They did not actually receive inform about this
          2. Under penalty of perjury filed an affidavit that they did not received this information
          3. Then imputation is shed and B and B’s new law firm is ok. Under MR 1.9 (a)
          4. Adams v. Aerojet-General Corp., 86 Cal. App. 4th 1324 (2001) –You don’t imput information from firm to lawyer once the lawyer has left the firm. Rebuttable presumptions under 1.9 (b)
            1. Facts:
              1. Attorney worked at firm A who represented client but attorney did not work on the case and knew nothing about it.
              2. Attorney left firm A and started his own firm that ended up representing a Plaintiff against prior firm’s client.
            2. Holding
              1. There is no indication of lawyers personal involvement with D at prior firm nor any direct evidence that he was exposed to client secrets during the time his former firm rendered services to D.
              2. DQ is inappropriate without further showing that he had access to the confidential; information
              3. Disqualification should not be ordered where there is no reasonable probability the firm switching attorney had access to confidential information while at his or her former firm that is related to the current representation
              4. Focus should not be on substnaital relationship between new and old firm work but rather attorney’s responsibilities as partner at old firm placed him in a position where he was reasonably likely to have obtained confidential information relting to the current case.

 

        1. A’S NEW FIRM – A has info – A leaves – can new firm represent an old client of A’s at his previous firm
          1. Until 2 years ago- when A moves to new firm, every lawyer in new firm is deemed to know what you know about the subject or case
          2. Today, new firm is presumptively tainted. Might be able to rebut by screening MR 1.10(a)(2)
          3. A firm that has acquired a new lawyer to rebut the presumption of share confidences by implementing an effective screen. This requires 1. The lawyer not be allocated a part of the fee from the matter creating the conflict, former client must be notified and given a discription of the screenig procedures and screening firm must respond rumple to queries from former client.
          4. Kirk v. First Am. Title Ins. Co., 183 Cal. App. 4th 776 (2010)
            1. Facts
              1. Gary is an associate at A and is working at Firemans Funds. One day receives a call from P’s lawyer who has class action pending against Fireman. Asks Gary to be an expert and Gary turns down the phone call.
              2. Gary then moves to firm B where conflict with client from Firm A arises.
            2. Holding
              1. New firm must adequately screen the new atty from the others at the firm representing the adverse party. Build a wall for the information and atty with the information!
              2. Elements of an effective screen
                1. Once the moving party in a motion for DQ has established atty is tainted
                2. Burden then shifts to new law firm to establish that the tactical effect of formal screen has been achieved. The showing must satisfy the trial court that the tainetd atty has not had and will not have any involvement with the litigation or communication with attys that would support a reasonable inference that the information has been use or disclosed.
                3. Screen mst be timely imposed
                4. Not sufficient just to produce declarations stating that confidential information was not conveyed or that that the atty did not work on this case. – there have to be preventative measures to guarantee that information will no be conveyed.
                5. Usually require atty to not have supervisory powers over the attys in the litigation and visa-vera.
                6. 1.10(a)(2) – Written notice to the former client must be promptly given

 

        1. A’S OLD FIRM – If A was the only lawyer who knew the info at the previous firm or all the lawyers who knew left with A – firm X is saying we don’t have anyone here left at the firm with that info. Yes if
          1. Affidavit that no one at the firm has information may rebut imputation under MR 1.10(b)

 

 

        1. Keep a list of matters and make it clear
          1. Do matter list straight – don’t play with adjectives.
          2. List all the cases you work on and have confidential information about.
          3. Lennartson v. Anoka-Hennepin Indep. School Dist. No. 11, 662 N.W.2d 125 (2003)
            1. Facts:
              1. Lennartson sued employer school district for sexual harassment. One of the lawyers at the firm representing her had taken one deposition for her case. WHAT A REASONABLE LAWYER WOULD HAVE OBTAINED.
              2. Later, that lawyer moved to another firm that represented the school district.
              3. The firm implemented screening procedures. Lawyer claims that she did “isolated work on the case” Firm agrees with this statement and minimiztation. This seems to be an obvious lie. However they kept her.
              4. However, the firm still needed to be disqualified under the disjunctive rule because of the information received, even though screens had been implemented.
            2. Holding: The court determines that 1.10(b) should be read conjunctively. Whenever one lawyer in a firm has a conflict, the conflict will be imputed and the entire firm will be disqualified unless the information communicated to the lawyer is unlikely to be significant in the same or substantially related matter, appropriate screening is implemented, and notice is given to all affected clients.

 

 

        1. Conflicts and the “Common Interest” Exception to Waiver of the Attorney-Client Privilege
          1. In Re Gabapentin Patent Litigation – A JDA reveals a clear intent that any voluntarily-shared information would remain confidential and be protected by the AC priviledge. Therefore, before of the relationship between each co-D and the lawyers, the lawyers must obtained consent from EVERY D IN THE JDA before they may defeat a motion to disqualify even with screening
          2. Essex Chemical Corp v Hartford Accident and Indemnity Company – came to a different conclusion
            1. There must be a strong factual finding before an attorney is DQ’s under a double imputation rule.
            2. Cannot automatically impute knowledge from firm one to individual lawyer to all defense counsel in JDA

 

        1. Note on Information Transmitted to Non-Lawyers: The Expert Witness ProblemWhat happens if one party retains an expert as a consultant, whose identity is not disclosed, and who is then contacted by the opposing party?
          1. Shandralina G. v. Homonchuck (Cal. 2007)
            1. Facts: Shandralina brought a malpractice action. Defense lawyers contacted expert Dr. Landers to consult with them. Later, one of Plaintiff’s attorneys retained Landers and sent him dome of plaintiff’s medical records. Defense counsel moved to disqualify Plaintiff’s lawyerss
            2. Holding:
              1. The court of appeals denied the motion because there was no legal impediment to Doctor’s ability to obtain evidence from the expert on the content of the conversation to satisfy the burden of proof.
              2. It held that a party seeking disqualification must show that the non-attorney whose conduct is the basis for the motion actually obtained confidential information.
              3. The party seeking disqualification will bear the burden of proving that the expert both received the information and disclosed it to the other side.
              4. Once the moving party shows this, the rule depends on whether the moving party is able to discover from the expert the contents of her communications with the lawyers for the opposing party. If the party seeking disqualification can discover from the expert what information the expert conveyed to the other side, then no presumption that such information was conveyed will attach.
              5. If the expert has severed all ties with the moving party, however, and refuses to communicate with them, then the court presumes that the expert conveyed confidential information to the other side and the burden shifts the the party whose disqualification is sought to rebut the presumption. (Note that unlike the substantial relationship test for attorney disqualification, this presumption is rebuttable).

 

    1. NON – CLIENT INFORMATION AND AFFILIATED ENTITIES
      1. Morrison Knudson Corporation v Hancock, Rotert, and Bunshoft – When an atty has received information from one affiliate which is substantially related to a claim against another affiliated, the attorney should ordinarily be disqualified from advancing the claim against the other.
      2. Faughn v Perez
        1. Facts – A represented a woman in medical malpractice against hotel. Years ago the parent corporation had retained A to defend other hospitals it own (not the D hospital in this case) in five childbirth malpractice cases.
        2. Holding – D had not established a substantial relationship between A’s previous representation and his suit against the hospital. There was evidence that suggested individual hospitals played some role in managing their defenses and the current hospital was in a different region with different policies that previous hospitals.

 

    1. CLIENT CONSENT
      1. Does not relieve a lawyer from loyalty and care
      2. Under 1.7(b)(4) clients may consent to allow their lawyer or a formal lawyer to undertake a matter the conflict rules otherwise would bar from taking so long as the consent is informed and in writing.
        1. Under 1.7 – consent to concurrent conflicts may be sought only if lawyer reasonably believes he could fufill his obligation to both clients.
        2. HOWEVER – 1.7(B)(3) – parties opposing each other in litigating cannot consent to same lawyer. No consent is the conflict is present!
      3. 1.9(a) – A lawyer who has formerly represented a client in a matter shall not thereafter represent another person in the same or a substantially related matter in which that person’s interests are materially adverse to the interests of the former client UNLESS the former client gives informed consent, confirmed in writing
      4. For multiple clients or representing entity and employees may want to obtained an advanced conflict waiver
        1. If a conflict comes up you all will waive them. Common when there is a company and employee or agent of the company.
        2. Say – 1. There isn’t a current conflict. 2. There isn’t one reasonably foreseeable 3. But if one comes up, the duty of loytaly will be affected and there will be no atty-client priv as between the waivers.
        3. The waiver asks the client to consent to the future representation of the other party and that they will not seek to disqualify from representing the other party and that they encourage them to seek independent counsel regarding the importances of consent.
        4. Should include
          1. A recitation of any current adversity
          2. Aa statement as to whether the waiver will allow the lawyer to be adverse to the client in that matter, explicit that this means that we will sue you and free to Use what you tell us even though absent this agreement we would be bound by duty of confidentiality.
        5. It’s Not Just A Letter, It’s An Exhibit
          1. Assume that everything you do and say will become publicly known. This applies also for conflict waivers.
          2. Tips for writing exhibit letters and conflict waivers:
            1. 1. Use simple words.
            2. 2. Be clear.
            3. 3. Lay it all out.
            4. 4. Don’t minimize or hide downside.
            5. 5. Don’t advise the client on your request

 

      1. Restatement §122Client Consent To A Conflict Of Interest
        1. (1) A lawyer may represent a client notwithstanding a conflict of interest prohibited by § 121 if each affected client or former client gives informed consent to the lawyer’s representation. Informed consent requires that the client or former client have reasonably adequate information about the material risks of such representation to that client or former client.
        2. (2) Notwithstanding the informed consent of each affected client or former client, a lawyer may not represent a client if:
          1. (a) the representation is prohibited by law;
          2. (b) one client will assert a claim against the other in the same litigation; or
          3. (c) in the circumstances, it is not reasonably likely that the lawyer will be able to provide adequate representation to one or more of the clients.
      2. Klemm v. Superior Court, 75 Cal. App. 3d 893 (1977) – Parameters of Consent
        1. Facts: Attorney represented both husband and wife in dissolution of marriage. On good enough terms that they ask the lawyer to represent both of them
        2. Holding:
          1. A lawyer may jointly represent two parties if the conflict between them is merely potential and there is full disclosure and informed consent of both clients.
          2. If the conflict is present and not just potential, then there can be no joint representation.
          3. This case was a case of potential conflict so the lawyer was allowed to represent the two parties if it was found that they gave informed consent. The only conflict that existed was between the county on the one hand and the husband and wife on the other as to whether the husband was obligated to pay the child support that the wife had waived.
          4. No conflict – just paper work for divorce

 

      1. Image Technical Servs, Inc. v. Eastman Kodak Co., 820 F. Supp. 1212 (N.D. CA 1993) – How to obtain consent
        1. Facts:
          1. Coudert, a law firm, represented Eastman Kodak on several international matters.
          2. It then accepted representation of ISOs before the supreme court in a brief contrary to the interests of Kodak.
          3. Coudert failed to obtain informed written consent from Kodak before representing the ISOs.
        2. Holding:
          1. Coudert needed to obtain INFORMED CONSENT, meaning that he would explain to the client the nature of the conflict and then obtain their written consent.
          2. It is not sufficient that both parties be informed of the fact that lawyer is undertaking to represent both of them. He must explain to them the nature of the conflict of interest in such detail so that they can understand the reasons why it may be desirable for each to have indepdent counse with unidivided loyalty to the interests of each of them.
          3. Here the conflict was mentioned in passing instead of explaining why the interests were contrary.
          4. Therefore, Coudert failed to obtain informed consent and feel short of the undivided loyalty it owed to its client. Two conditions need to be met: each client must consent to the multiple representation after full disclosure; and (2) it must be obvious that the attorney can adequately represent the interests of each client.

 

      1. Zador Corp., N.V. v. C.K. Kwan, 31 Cal. App. 4th 1285 (1995) –ADVANCED CONFLICT WAIVER – How to obtained consent to a future conflict that was not certain at the time consent was given
        1. Facts:
          1. Heller had Kwan sign a consent letter explaining that there would be no privilege in the representation, that if the interests between Zador and Kwan became adverse that Heller would represent Zador, and encouraging him to seek independent counsel.
          2. The documents showed that Kwan might have received money from the sale and Heller said that Kwan should retain independent counsel and that per the agreement, Heller would continue to represent Zador.
          3. Wilson represented Kwan and told Heller that if Zador sued Kwan, it would seek to disqualify Heller. Kwan admitted that he had profited from the property deal. Zador refused to indemnify Kwan. Heller named Kwan as a cross-defendant.
          4. Kwan moved to disqualify Heller on the ground that there was a substantial relationship between Heller’s prior representation of Kwan and the current litigation.
        2. Holding:
          1. In cases of joint representation where C represents A and B and then B sues A who is still represented by C, the substantial relationship between the former representation and subsequent action is inherent and a substantial relationship will always exist and therefore should not be the determinative factor in deciding whether to disqualify an attorney.
          2. Rather, these cases turn on the scope of the client’s consent.
          3. Here the consent was sufficient and Heller was not disqualified from representing Zador.
          4. Here Kwan signed a waiver and agreed that Heller would represent Zador in the event of a conflict. He also knew he had the right to obtain separate counsel. Kwan also agreed that he would consent to continued representation of the company notwithstanding any adversity that may develop. Kwan then reaffirmed his consent once the conflict emerged. Not every possible consequence of a conflict must be disclosed for consent to be valid.
        3. Was the reaffirmation of consent necessary? Visa USA, Inc. v. First Data Corporation (CA 2003). A second waiver is only necessary if the attorney believes that the first waiver was insufficiently informed. There is no case law requiring a second disclosure in all circumstances for an advance waiver to be valid.

 

      1. Waivers Involving Entities and Entity Constituents
        1. Under MR 1.13 counsel for an entity represents the entity itself and not its representatives.
        2. In some cases, counsel may represent both, but may need to obtain a waiver.
        3. Conflict waivers in such situations must be signed by a representative of the entity other than the representative who will be jointly represented. (MR 1.13(g)).

 

    1. TRANSCATION WITH CLIENTS/GIFT FROM CLIENTS AND HOW THAT LEADS TO CONFLICTS
      1. Dangerous to do business with clients or to receive substantial gifts.
      2. IN A TRANSACTION
        1. You bear the burden of showing the substantive fairness. Fairness is hard to show.
          1. Voidable transactions. As Beery suggests, if a lawyer engages in a transaction with a client, the transaction is voidable by the client unless the attorney demonstrates that the client was fully informed regarding all material aspects of the transaction, and willingly entered into it, and that the transaction was substantively fair to the client.
          2. To show that tty-client transaction was substantively fair to the client
            1. Take an arm-length market transaction as baseline then
            2. Compare that transaction to the one between lawyer and client.
        2. Model Rule 1.8(a) –CONFLICT OF INTEREST: CURRENT CLIENTS: SPECIFIC RULES
          1. (a) Forbids lawyers from doing a deal or acquiring an interest (suc as a lien) adverse to a client UNLESS
            1. The client is advised in writing that he or she may wish to consult an independent lawyer regarding the transaction
            2. Given reasonable time to do so
            3. Client consents in a sign writing that sets for the essential terms of the transaction and lawyer’s role in it – including whether the lawyer represents the client in the transaction
            4. This applies even if the business transaction is unrelated of the representation.
            5. Generally does not apply to transactions in good of services the client offers to general public and in which lawyer trades on that basis.
        3. Beery v. State Bar, 43 Cal. 3d 802 (1987)
          1. Attorney represented client and client’s business in several cases. Later on, attorney offered client a private investment deal which in negotiations about drafting a will for client. Business transaction failed and client brought suit for money back. Client claimed he thought atty was acting as attorney, not a business partner. Lawyer claims no lawyer-client relationship at time of deal.
          2. Holding
            1. It the lawyers burden to explicitly end the atty-client relationship and atty did not in this case
            2. Reasonable for client to believe he was acting in the capacity of lawyer.
            3. In a lawyer-client business transaction, there is always a rebuttable presumption that the deal was unfair due to the atty’s great influence over client. Thus, attorney must show
              1. That he disclosed to client ALL information against himself as he would against a 3rd partying transacting with client
              2. Term of the agreement were fair and just
              3. Client acted with full knowledge of all acts and fully disclosed their effects.
            4. Transactions between an attorney and client are always strictly scrutinized against the attorney. There is a fiduciary relationship which means that the parties are not on equal terms.
            5. In this case, it was not an arm’s length business deal, material facts were concealed, the attorney willfully violated a disciplinary rule and the attorney should be disciplined.
        4. Fletcher v. Davis (Cal. 2004)
          1. An attorney can obtain a charging lien on some property to secure his fee in the event that the client fires him and then prevails. However, the attorney must obtain informed written consent (and tell the client to seek the advice of independent counsel) for the charging lien because it is an adverse interest to the client because in the event that there is a dispute over the lien or its existence, the attorney would have the ability to detain the property from being distributed to the client.
          2. Other note: Note that this rule is limited to cases in which counsel charges an hourly fee and does not apply to contingent fee arrangements.
        5. Sugarman v. State Bar (Cal. 1990)
          1. Facts: Attorney represented both company and its president. He had not been paid and refused to do more work. The president said he would loan the money on behalf of the company to the attorney and then the attorney would pay the president back. The attorney agreed and they executed a formal note. The loan went through but the attorney later declared bankruptcy and listed the president as a creditor. The court held that: all dealings between an attorney and his clients that are beneficial to an attorney will be closely scrutinized with the utmost strictness for any unfairness.
        6. Fergus v. Songer (Cal. 2007). Songer hired Fergus to enforce a judgment against the owner of a hotel. Fergus worked on a contingency fee basis. Songer wanted to buy the hotel so Fergus advanced him the money in exchange for raiding his percentage of the contingency fee. Fergus and his wife used their home as credit and advanced the money to Songer, they became 50% partners. Songer refused to pay the contingent fee claiming the initial fee was voidable. The court agreed but said that Fergus’s wife had a separate claim against Songer because she was not an attorney and was not bound by the ethical rules.

 

      1. GIFT
        1. Model Rule 1.8(c) Forbids lawyer from soliciting a substantial gift or preparing a document (such as a will) conveying such a gift, unless you are related to client
        2. When a client wants to give the lawyer a gift, everything rests on the procedure by which the gift was memorialized and whether the attorney can rebut the presumption that the gift was the product of undue influence, and then, if necessary, defeat whatever evidence a plaintiff might introduce to establish such influence.
        3. If an atty is involved with drafting a persons will and atty end up being contained in the will, they must have clear and convincing evidence to rebut to presumption of undue influence once it has been raise – i.e. related or friends for a long time.
          1. Franciscan Sisters Healthcare Corp v. Dean (Ill. 1983). Example of when an attorney was able to rebut the presumption that a bequest by his client was void. The woman gave the attorney money in a will. They had known each other for over 20 years socially. The attorney had her speak to another attorney alone. The court therefore found there was no undue influence. The undue influence must be of such a nature as to destroy the testator’s freedom concerning the disposition of his estate and render his will that of another. This was not present in this case and the presumption was rebutted because the attorney met the “clear and convincing evidence” standard.

 

      1. Cal. R. Prof. Conduct 3-300Avoiding Interests Adverse to a Client – A member shall not enter into a business transaction with a client; or knowingly acquire an ownership, possessory, security, or other pecuniary interest adverse to a client, unless each of the following requirements has been satisfied:
          1. (A) The transaction or acquisition and its terms are fair and reasonable to the client and are fully disclosed and transmitted in writing to the client in a manner which should reasonably have been understood by the client; and
          2. (B) The client is advised in writing that the client may seek the advice of an independent lawyer of the client’s choice and is given a reasonable opportunity to seek that advice; and
          3. (C) The client thereafter consents in writing to the terms of the transaction or the terms of the acquisition.

 

    1. INSURANCE CONFLICTS
      1. Introduction
        1. Insurance companies commonly hire lawyers to represent an insured who has been involved in an accident
        2. Does lawyer represent the insured or both insured and insurance company
        3. Is insured at risk from a lawyer who is paid by and long term relationship with the insurance company?
          1. Model Rule 1.8(f) – allows a lawyer to accept payment from a non-client is the client gives informed consent, the lawyer’s independent judgment is not compromised and the client’s confidences are maintained.
        4. Conflict with insurer pays to defend an insured while reserving the right to claim later than the insurance policy did not cover the relevant matter.
          1. Rule 1.7(a) – “Unless a client gives informed consent, a lawyer shall not represent a client if the representation involves a “concurrent conflict of interest.” A concurrent conflict of interest exists if there is a significant risk that the representation of one or more clients will be materially limited by the lawyer’s responsibilities to another client, a former client, or a third person or by a personal interest of the lawyer.”
      2. Purdy v. Pacific Automobile Insurance Company, 157 Cal. App. 3d 59 (1984)
        1. Facts:
          1. Attorney represents insurer and insured. Client wants attorney to settle for 100k because jury verdict would be must higher. Counsel refuses to settle hoping to show contributory negligence of other driver and no liability. counsel looses and insured bring suit for bad faith filutre to settle – legal malpractice.
        2. Holding:
          1. STUNNING CONFLICT between insurers interest and Purdy’s interest.
          2. The attonrye’s primary duty has been said to be further the best interest of the INSURED. Therefore when a conflict develops between insurer and the insured, the insurer cannot compel the insured to surrender control of the litigation and must, if necessary, secure independent counsel for the insured.
          3. He was not able to represent both people too. If no advanced waiver, needs to get out of both. If advanced waiver, can keep representing the insurance company. Roberts should be replaced by two other people.
          4. Duty to defend vs. indemnify. The duty to defend is broader than the duty to indemnify.
          5. In this case malpractice case loosing because of causation
        3. All jurisdictions agree that the insured is a client of the lawyer. But the insurer has client like characteristics too and will generally select and pay for the lawyer. Jurisdictions differ on whether the insurance company is a client in this situation.
          1. Default -Insured-only representation, subject to modification by the parties.
      3. Mixed Action – both potentially covered and non-covered actions – the insurer has a duty to defend the action in its entirely. As to the claims that are at least potentially covered, the insurer may NOT seek reimbursement for defense costs but can for non-covered action costs.
      4. Note on the Dual Representation Problem and Cumis Counsel – IF DEFENSE AFFECTS WHETHER CLAIM IS COVERED BY INSRUANCE.
        1. REQUIRES INDEPENDENT COUNSEL.
        2. What happens when there is a conflict between the insurer and insured such that control of the defense of the action might affect whether a claim was covered? Cumis counsel.
        3. San Diego Navy Fed. Credit Union v. Cumis Ins. Society (Cal. 1984).
          1. When it is unclear whether claims are covered by the policy it is standard for an insurer to defend under a reservation of rights.
          2. A lawyer in this situation must explain to the insured and insurer the full implications of joint representation in situations where the insurer has a reservation of rights. If the insured does not give informed consent to the joint representation, counsel must cease to defend both.
          3. In the absence of consent, the insurer must pay reasonable cost for hiring independent counsel by the insured. Rules governing whether an insured has a right to Cumis counsel are codified in Cal. Civ. Code §2860.

 

    1. The Prohibition On Trying A Case You (Or Another Lawyer From Your Firm) Will Testify In
      1. MR 3.7 – forbid lawyer from trying a case in which they are likely to be a necessary witness unless your expected testimony is uncontested (such as authenticating a document nobody questions), or if disqualifying you would work a hardship on your client.
      2. You may try the case if another lawyer in your firm is likely to testifying UNLESS the testimony creates some kind of conflict of interest under 1.7 and 1.9 which inputs to you under 1.10(a)
        1. These are construed strictly – Murray v Metropolitan Life Insurance
          1. Law firm can be disqualified by imputation only if the movant proves by clear and convincing evidence that
            1. The witness will provide testimony prejudicial to the client and
            2. The integrity of the judicial system will suffer as a result

 

  1. CHAPTER 11 – RELATIONS WITH THIRD PARTIES ON BEHALF OF CLIENTS
    1. REPRESENTED PERSONS
      1. Model Rules 4.2COMMUNICATION WITH PERSON REPRESENTED BY COUNSEL – In representing a client, a lawyer shall not communicate about the subject of the representation with a person the lawyer knows to be represented by another lawyer in the matter, UNLESS the lawyer has the consent of the other lawyer or is authorized to do so by law or a court order.
          1. Also, the attorney must have actual knowledge that they are represented by counsel to violate the rule

 

      1. OVER VIEW
        1. PURPOSE TO SUPRESS INFORMAL DISCOVERY
        2. Cannot communicate without the consent of their lawyer or authorize by law or court order – even if they contact you
        3. UNLESS PART OF A GOOD FAITH UNDER COVER INVESTIGATION.
        4. Charging with a crime unnecessary. A client need not be charged with a crime for 4.2 to apply. Legitimate investigative processes may go forward even if the target of the investigation is represented by counsel, but when the process goes beyond legitimate investigation, it is not authorized by law. (State v. Miller (Minn. 1999)).
        5. Communication. Cases come out differently on what constitutes a communication for purposes of this rule. Hill v. Shell Oil Co. (Ill. 2002) (videotaping operations of gas stations not communications); Midwest Motor Sports v. Arctic Cat Sales, Inc (8th Cir. 2003) (investigator speaking to salesperson at showroom of plaintiff (terminated dealer) was a communication); Microsoft Corp. v. Alcatel Bus. Systems (Del. 2007) (technician who installed telecommunication system that allegedly infringed their client’s patents was questioned and since he was represented, the lawyers were sanctioned).
        6. Representing one self or unbundled representation
          1. A party representing himself but to whom limited representation has been provided is considered unrepresented for purposes of 4.2, unless the lawyer who wants to communicate with that party has knowledge to the contrary.
        7. Indirect contact and scripting. A lawyer cannot do indirectly what he could not do himself. So a lawyer cannot script his client’s contact with the opposing party. The contact must originate and be directed by the client.
          1. Your client can call them. But you cannot write out script or do anything indirectly which you yourself cannot do.
          2. Scripting a letter is a violation.
          3. Difficult to know how to act – it is difficult to prove and therefore you are not supposed to make an accusation. This subset is ignored by practice

 

        1. Matter of Howes, 940 P.2d 159 (N.M. 1997) – EVEN LISTENING COUNTS.
          1. Facts:
            1. Defendant contacts police to talk about murders with which he is charged. D is arrested and has an urge to talk.
            2. Respondent represented the US and told the detective to listen to defendant’s statements but not initiate any contact. Detective worked to US. They know he is going to be represented! He asked no questions, only listened.
            3. The public defender was never told.
          2. Holding
            1. 5.2 – a junior lawyer can rely on the reasonable resolution by the person supervising them.
            2. California does not have this rule.
            3. This was not a reasonable resolution
            4. Respondent still violated the rule by listening to the defendant. One can communicate simply by indicating a willingness to listen.
            5. Nor were the communications authorized since they went beyond the bounds of investigation because defendant was already being held with probable cause.

 

      1. COMPANIES
        1. When you represent an entity, you don’t represent the constituents on an individual basis unless you undertake to do for. But for 4.2 purposes, you are deemed to represent some set of entity employees. – See Snider.
        2. When employee does not want representation. When an employee comes forward saying she has a conflict with the corporation, the prosecutor should tell the employee to seek her own representation. United States v. Talao (9th Cir. 2000).
        3. Comment of 4.2 – in the case of a represented organization or company, the rule prohibits communications with the constituent of the organization who supervises, directs or regularly consults with the organization’s lawyers concerning the matter or has authority to obligate the organization with respect to the matter or whose act or admission in connection with the matter may be imputed to the organization for the purpose of criminal or civil liability.
          1. Jorgensen v. Taco Bell Corp (Cal. 1996). Where an attorney hires an investigator to interview an employee before any suit is even filed who later turns out to be represented by counsel, there is no violation since there was no matter in which the alleged harasser, in this case, could be represented.
          2. Inorganic Coatings, Inc. v. Falberg – where contact occurred but the decision to file suit had clearly been made = disqualification
          3. Snider v. Superior Court, 113 Cal. App. 4th 1187 (2003) – For 4.2 purposes, you are deemed to represent some set of entity employees.
            1. Facts:
              1. Snider left Quantum and formed Gardenia which Quantum complained was a competing business in which Snider used confidential information from Quantum.
              2. Snider’s attorney spoke to two Quantum employees (sales manager and director of production) before trial and Quantum’s attorney moved to disqualify them.
            2. Holding
              1. There are five tests to determine who the attorney can contact: the control group test, the party-opponent test, and the management speaking agent test, case by case balancing test, and the alter ego test.
                1. Broad test – as soon as entity has a lawyer, lawyer represents everyone in the entity. But this isn’t followed anywhere
                2. Party opponent jurisdictions – anyone who could make a statement binding on the entity counts as a binding
                3. Management speaking agent test
                4. Alter ego
                5. Control group
              2. An attorney cannot communicate with a party he knows to be represented by counsel upon the subject of the controversy without the consent of that counsel.
              3. However, attorneys can contact nonmanagement personnel so long as the communication does not involve the employee’s act or failure to act in connection with the matter which may bind the corporation, be imputed to it.
              4. The court in this case concludes that managing agent refers to those with substantial discretionary authority over decisions that determine organizational policy; the two employees did not fit this description; nor could their statements constitute admissions of the organization.
          4. DOES NOT COVER FORMER EMPLOYEES. YOU MAY INTERVIEW FORMER EMPLOYEES WIHTOUT ASKING PERMISSION FROM COUSEL FOR THE ENTITY.
            1. But under 4.4, the attorney must still respect the rights of those third parties, so if there is privileged information, the attorney must not ask for it (since the privilege belongs to the entity and not the employee).
            2. Siebert & Co., Inc. v. Intuit Inc. (2007)
              1. In this case the attorneys advised the former employee of their interest, they asked him not to answer any questions that would disclose privileged information, and also he stated he understood this, and no information was disclosed.
              2. Lower court disqualified but appeals court reversed. No disqualification.
              3. Attorneys may interview former employees who posess priviledges of work-product information, as long as they take reasonable steps not to acquired such informations
                1. Inform former employees not to disclose any priviledge/confidential information
                2. Decline to answer any questions that might reasonably lead to disclosure of such information
              4. Even with that level of disualification – laower court disqualified the firm
              5. There was therefore no basis for disqualification.

 

      1. THE NO CONTACT RULE AND UNDERCOVER INVESTIGATIONS
        1. Indirect acts by attorneys. Model rule 8.4(a) prohibits lawyers from using third parties to do things the rules prohibit lawyers from doing themselves. Combined with 4.2, that rule presents a problem for lawyers that use investigators. Most cases find a way around this literal approach..
          1. Usually you don’t know when the person is represented so no issue
        2. If you are doing only what members of the general public do and asking the target only to do what he normally does in his course of business OK. Although no support for this in the text of the rule.
        3. In at least three types of cases, courts tend to allow lawyers to use investigators to obtained evidence without admit tin that they are investigating legal claims.
          1. Civil rights cases – lawyers may employ “testers”
          2. Government lawyers who work with undercover law enforcement officials to gather evidence of a crime
            1. United States v. Hammad (2d Cir. 1988)
              1. Applied Rule 4.2 to prosecutors who participate in pre-indictment investigations.
              2. Prosecutor drafted a fake subpoena for person to talk to D.
              3. Holding – violated the rule – created a pretense that might help the informant elicit admissions from the represented suspect.
              4. Trial courts have discretion to suppress evidence obtained through violations of the no-consent rule. But to date no reported federal case has suppressed evidence as a sanction for a biolation of the no-contact rule.
        4. The No-Contact Rule and Class Actions –uncertified class members are not represtned by counsel for purposes of the no=contact rule. However courts will pay attention to circumstances of particular case and might be willing to grant orders that would limit or preclude certain kinds of contacts.

 

    1. UNREPRESENTED PARTIES
      1. You may communicate with unrepresented people BUT
        1. You may NOT imply you are disinterested. YOU CANNOT IMPLY TO THEM THAT YOU HAVE NO ANGLE. YOU ALWAYS HAVE AN ANGLE. and
        2. IF YOU KNOW OR SHOULD KNOW THAT UNREPRESENTED CLIENT THINKS YOU ARE ON THEIR SIDE OR NO SIDE – YOU NEED TO MAKE REASONABLE EFFECT TO CORRECT MISUNDERSTAND. If they are confused about your role, you must clarify it.
      2. Model Rule 4.3 – Dealing with Unrepresented Parties
        1. In dealing on behalf of a client with a person who is not represented by counsel, a lawyer shall not state or imply that the lawyer is disinterested. When the lawyer knows or reasonably should know that the unrepresented person misunderstands the lawyer’s role in the matter, the lawyer shall make reasonable efforts to correct the misunderstanding. The lawyer shall not give legal advice to an unrepresented person, other than the advice to secure counsel, if the lawyer knows or reasonably should know that the interests of such a person are or have a reasonable possibility of being in conflict with the interests of the client
      3. Restatement §103Dealings With An Unrepresented Nonclient
        1. In the course of representing a client and dealing with a nonclient who is not represented by a lawyer
          1. (1) the lawyer may not mislead the nonclient, to the prejudice of the nonclient, concerning the identity and interests of the person the lawyer represents; and
          2. (2) when the lawyer knows or reasonably should know that the unrepresented nonclient misunderstands the lawyer’s role in the matter, the lawyer must make reasonable efforts to correct the misunderstanding when failure to do so would materially prejudice the nonclient.
      4. Stating A Position, Stating A Fact, Or Giving Advice?
        1. Stating A Position – may implicate 4.1 but generally no issues at all.
        2. Stating A Fact – implicates fraud and 4.1. it implicates reliance interest
        3. Giving Advice – implicated duties of care and malpractice

 

      1. Hopkins v. Troutner, 4 P.3d 557 (Idaho 2000) – DO NOT GIVE ADVISE OTHER THAN TO SECURE COUNSEL IF THE LAWYER KNOWS AND SHOULD KNOW INTERESTS OF THAT PERSON HAVE A REASONABLE POSSIBILITY OF BEING IN CONVLICT WITH INTEREST OF CLIENTS.
        1. Facts:
          1. Defendant’s attorney contacts unrepresented plaintiff to settle the case. Plaintiff asks D’s attorney what he thinks the case is worth and he says three to four thousand.
            1. This was his honest opinion. No 4.1 issue, no deceit
          2. The plaintiff agrees to settle for 5,500.
          3. The plaintiff has settler’s remorse and then seeks release from the settlement due to overreaching of the attorney.
        2. Holding: WRONGFULLY GAVE ADVICE
          1. The court holds that under MR 4.3, an attorney should not give advice to an unrepresented person, other than to obtain counsel. Attorney gave plaintiff advice and he relied on that advice and therefore they find there was a violation and release plaintiff from the agreement.
          2. Words are deeds. When you are talking about 4.3 what you say matters a lot. Important distinction is taking a position, starting a fact or giving advice.
            1. If Troutner had said – In my opinion this is worth 3 or 4k – no rule violation
          3. Lawyer is not going to get reprimanded over this but it makes the point

 

    1. CANNOT INDUCE OR ASSIST WITNESS IN EVADING OR IGNORING REQUEST FOR RELEVANT INFORMATION.
      1. Model Rules 3.4(f) A lawyer cannot request a person other than a client to refrain from voluntarily giving relevant information to another party UNLESS
        1. (1) the person is a relative or an employee or other agent of a client; and
        2. (2) the lawyer reasonably believes that the person’s interests will not be adversely affected by refraining from giving such information.
      2. Restatement 116(3): a lawyer may not unlawfully induce or assist a prospective witness to evade or ignore proceed obliging the witness to appear to testify
      3. Kensington Int’l Ltd v. Republic of Congo, 2007 WL 2456993 (S.D.N.Y.)
        1. Facts
          1. Congo tried to depose Mbemba to determine if he knows where the Congo’s money is stashed. They want to depose him in NY to be convenient for everyone.
          2. Cleary (Lawfirm), contacted Mbemba directly and persuaded him not to go to a deposition (even though he was unrepresented by counsel) it was illegitimate.
          3. Then, the firm tried to hide their influence in persuading him not to go to the deposition by saying that he had contacted them saying he would not go without a lawyer.
        2. Holding
          1. Not allowed to dissuade an unreprestned party from offering evidence. However, may send a signal so you are not overtly saying anyting.
          2. Cleary therefore interfered with a post-judgment discovery process and sanctioned under 3.4(f) which prevents a lawyer from instructing that a person other than a client refrain from voluntarily giving relevant information to another party.
          3. All of this is discoverable because no representation.
          4. Client’s perception was that he was basically told not to go.

 

    1. Receiving Evidence – DOCUMENTS
      1. You may not get evidence by violating the rights of third party
      2. Documents are property
        1. They don’t belong to the individual employees
        2. If you get them and you shouldn’t, don’t read them and give them back
      3. Model Rule 4.4 –RESPECT FOR THE RIGHT OF THIRD PERSONS
        1. (a) In representing a client, a lawyer shall not use methods of obtaining evidence that violate the legal (rights created by law – rather than contract like non-disclosure agreement) rights of such a person.
          1. CA – is planning to eliminate legal requirement so that someone could be in trouble for ignoring Non-disclosure agreement
        2. (b) A lawyer who receives a document relating to the representation of the lawyer’s client and knows or reasonably should know that the document was inadvertently sent shall promptly notify the sender.
          1. 4.4 does not say you cant read the paper. But you should definitely notify the puntative owner that you have them.
      4. Pillsbury, Madison & Sutro v. Schectman, 55 Cal.App.4th 1279 (1997)
        1. Facts:
          1. The facts here involve the wrongful possession of confidential documents for use in anticipated litigation against PM&S by Schechtman and his law offices. PM&S claims the documents were wrongfully removed.
        2. Holding:
          1. Self-help discovery is not allowed if it is a violation of ownership or privacy interests.
          2. Schectman was ordered to return the documents.
          3. Will probably obtain these again once there is discovery.

 

      1. Carpenters Health and Welfare Fund v. Coca Cola Company (GA 2008).
        1. Facts
          1. Coke employees who were fired took documents with then, marked confidential and offered these to plaintiff’s counsel
          2. D – coke – moved for plaintiff’s counsel to be disqualified.
        2. Holding
          1. Under 4.4(a) a lawyer may not get evidence by violating the rights of third party
          2. You cannot do indirectly what you are prohibited from doing directly
          3. Therefore, lawyer cannot be involved in the subsequent review of evidence obtained improperly by the client
          4. Restatement 102 – an investigating lawyer may not seek to obtain information when he reasonably should know that the witness in question owes to a third party a duty of confidentiality that is imposed by law
          5. 3.4 – a lawyer may not ethically offer a witness money contingent on the contents of witness’ testimony

 

    1. Embarrassing or burdening others
      1. You may not do things to embarrass, delay or burden a third party
      2. 4.4(a) In representing a client, a lawyer shall not use means (methods, tactics) that have no substantial purpose other than to embarrass, delay, or burden a third person
      3. Idaho State Bar v. Warrick, 44 P.3d 1141 (2002)
        1. Facts:
          1. Warrick the prosecutor was at the jail where defendant was being held.
          2. He wrote waste of sperm and scumbag next to his name on the board.
          3. Complaints were filed against Warrick under 4.4(a).
        2. Holding: An attorney cannot act under 4.4(a) when it is obvious that the action would harass or maliciously injure another. Warrick’s actions had no other purpose than to embarrass the inmate and engender bias in the local officers. Warrick therefore violated 4.4(a).

 

 

 

  1. CHAPTER 12 – RELATIONSHIP WITH YOUR FIRMS
    1. Rules
      1. Model Rules 8.3REPORTING PROFESSIONAL MISCONDUCT
        1. (a) A lawyer who knows that another lawyer has committed a violation of the Rules of Professional Conduct that raises a substantial question as to that lawyer’s honesty, trustworthiness or fitness as a lawyer in other respects, shall inform the appropriate professional authority.
        2. (b) A lawyer who knows that a judge has committed a violation of applicable rules of judicial conduct that raises a substantial question as to the judge’s fitness for office shall inform the appropriate authority.
        3. (c) This Rule does not require disclosure of information otherwise protected by Rule 1.6 or information gained by a lawyer or judge while participating in an approved lawyers assistance program.
      2. Model Rule 8.4: It if professional misconduct for a lawyer to violate or attempt to violate the Rule of PC, knowingly assist or indict another to do so or do so through the acts of another.
      3. Model Rule 8.4 (c) – it is professional misconduct for a lawyer to engage in conduct involving dishonesty, fraud, deceit or misrepresentation.

 

    1. Fiduciary Obligations to a Firm
      1. You are an agent of your firm or partnership as well as an agent of your clients.
      2. Therefore you owe the firm the standard duties or loyalty and care agents owe any principles
      3. You may be disciplined for violating these duties, although discipline in such cases if pless common than discipline for violating cuties to client.
      4. In the Matter of Curran 509 N.W. 2d 429 (Wis. 1994)
        1. Facts: Curran an attorney funneled funds from law partnership to his personal account. When the firm dissolved, current and partners settle things and he pays them back.
        2. Holding: Violation and suspended from practice for 2 years.
          1. Lawyers in a partnership have the same duty to one another—loyalty, disclosure, accounting—as persons in a non-lawyer partnership.
          2. The misappropriation of funds from the law partnership is just as serious as misappropriating them from a client. Therefore, he violated the rules of professional conduct. This same rule applies even to associates who are not partners of the firm.
      5. Johnson v. Brewer & Pritchard, P.C. (Tex 2002)
        1. An associate owes a fiduciary duty not to accept a fee or other compensation for referring a matter to a lawyer or other firm other than the associate’s employer without the employer’s consent. Associates in a firm are not allowed to practice law for their own account or engage in employment fo any kind other than their employment with that firm. However, a firm’s legitimate interest in demanding loyalty from its associates should not outweigh competing considerations of the public’s interest in encouraging lawyers to assist those who need legal advice and securing the most appropriate representation, even if it means referring them to another firm without compensation.
        2. Rule: An associate may participate in referring a client or potential client to a lawyer or firm other than his or her employer without violating a fiduciary duty to that employer as long as the associate receives no benefit, compensation, or other gain as a result of the referral. However, an associate owes a fiduciary duty not to accept or agree to accept profit, gain, or any benefit from referring or participating in the referral of a client or potential client to a lawyer or firm other than the associate’s employer.

 

    1. Mobile Lawyers
      1. Common for lawyers to move from one firm to another. Risk of breach of contract, ficuairy duty and/or theft of trade secrets.
      2. Soliciting Former Clients
        1. Restatement §9 – absent an agreement with the firm providing a more permissive rule, a lawyer leaving a firm may solicit firm clients
          1. Prior to leave the first
            1. Only with respect to the firm clients on whose matters the lawyer is actively and substantially working and
            2. Only after the lawyer has adequately and timely informed their firm of the lawyer’s intent to contact firm client for that purposes
          2. After ceasing employement – to the same extend that any othe rnonfirm lawyer could.
        2. Graubard, Mollen, Dannett & Horowitz v. Moskovitz, 629 N.Y.S. 2d 1009 (1995)
          1. Issue: Can departing lawyer solicit clients of the firm?
          2. Holding:
            1. Telling a client that the attorney is leaving and that they are free to retain counsel of their choice is ok; but secretly attempting to lure clients to a new firm is not. Lawyer cannot fraudulently misrepresent to your former firm that you will not impair their clients relationships
            2. There is a breach of fiduciary duty when a withdrawing partner, prior to announcing his resignation, solicits firm clients; second, a contractual requirement that an attorney try to integrate or institutionalize clients into the firm is legally enforceable; and third, a cause of action for fraud is properly stated when a promisor, at the time of making certain representations, lacked any intention to perform them.
        3. Gibbs v Bam
          1. Facts: employee supplied confidential employee information to their new firm, while still partners at the old firm
          2. Holding
            1. No breach when D’s removed thir desk copies of recent correspondence, as they had a good faith belief that they were entitled to do so. Desk filed were duplcate or material mainitna in individual client files and was apparently common practice for departing attorneys.
            2. REACH BY SUPPLYING NEW DIEM WIRH OLD IFIRMS T/E DEPARTMENT AND SALARIES AND OTHER CONFIDENTIAL INFORMATION
            3. You may prepare to complete with you current firm while still employed by then but you cannot actually compete with your current firm until you leave – i.e. soliciting clients from old firm.

 

      1. Soliciting Former Colleagues
        1. Reeves v. Hanlon, 33 Cal. 4th 1140 (2004)
          1. Facts: On the evening of their resignations, defendants personally solicited their firm’s key employees. The firm lost 9 employees. They also contacted at least 40 clients by phone without offering them a choice of counsel.
          2. Holding:
            1. A firm may recover damages for intentional interference with an at-will employment relation under the same standard applicable in CA to recover for intentional interference with prospective economic advantage.
            2. To recover, a plaintiff must plead and prove that the defendant engaged in an independently wrongful act—an act proscribed by some constitutional, statutory, regulatory, common law, or other determinable legal standard—that induced an at-will employee to leave the plaintiff.
            3. Under this standard, D is not subject of liability for intentional interference if the interference consists merely of extending a job offer that induces an employee to terminate his or her at will employment.
            4. Here the two attorneys engaged in unlawful and unethical conduct by mounting a deliberate campaign to disrupt plaintiff’s business. They destroyed computer files and unlawfully solicited the clients.
            5. Note: Reeves treated a client list as a trade secret. It qualified this holding by distinguishing between use of a list to “solicit” clients, which would qualify as misappropriation, and the use of the list merely to announce a change in employment status. This was upheld in Moss, Adams & Co. v. Shilling (Cal. 1986) where two accountants sent announcements to everyone on their former employer’s rolodex with the announcement and address of their new partnership. The court held it did not constitute solicitation.

 

      1. The Trade Secrets Problem
        1. UTSA – violation occurs when an individual misappropriates a former employee’s protected trade secret list – for example bu using it to solicit client.
        2. Client slits qualified as a trade secret it is “derives independent economic value from not being generally known to the public or to other persons who can obtain economic value from it disclosure or use and is subject of efforts that are reasonable under the circumstances to maintain its secrecy
        3. A past employee MAY use a client list merely to announce a change in employe ment statute BUT NOT to “solicit” clients which qualifies as misappropriation (Reeves)

 

      1. Agreements Restricting Practice
        1. Lawyers may not enter into agreements restricting their right to practice law after they leave the firm
        2. A LAWYER SHALL NOT PARTICIPATE IN OFFERING OR MAKING AN AGREEMTN THAT RESTRICTS THE RIGHT OF A LAWYER TO PRACTICE AFTER TERMINATION OF A RELATIONSHIP EXCEPT AN AGREEMENT CONCERNING BENEFITS UPON RETIREMENT
        3. Model Rule 5.6RESTRICTIONS ON RIGHT TO PRACTICE
          1. A lawyer shall not participate in offering or making:
            1. (a) a partnership, shareholders, operating, employment, or other similar type of agreement that restricts the right of a lawyer to practice after termination of the relationship, except an agreement concerning benefits upon retirement; or
            2. (b) an agreement in which a restriction on the lawyer’s right to practice is part of the settlement of a client controversy.
        4. Howard v. Babcock, 6 Cal 4th 409 (1993) – Unusual case which allows a non-competition to be enforced.
          1. An agreement between law partners is enforceable if it requires withdrawing partners to forego certain contractual withdrawal benefits if they compete with their former law form.
          2. An agreement among law partners imposing a reasonable toll on departing partners who compete with the firm is enforceable.
          3. An agreement that assesses a reasonable cost against a partner who chooses to compete with his or her former partners does not restrict the practice of law. Rather, it attaches an economic consequences to a departing partner’s unrestricted choice to pursue a particular kind of practice.
          4. HOWEVER it must have reasonable limitations involving geographic or time period scope
          5. Note: Howard v. Babcock is somewhat unusual in enforcing an agreement that creates a disincentive for lawyers to take on new clients. The following case is more representative
        5. Law offices of Ronald J. Palagi v. Howard (Neb. 2008)
    1. Facts: In this case, there was a clause in an attorney’s employee agreement that if he left the firm and a client decided to have him continue representing them, the attorneys fees generated on such a matter would be paid to the law firm.
    2. Financial disincentive provisions which are anti-competitive and which restrict trade are unenforceable as against public policy and rule 5.6
    3. Under MR 5.6, courts do not enforce any agreement involving the employment of lawyers that appears to have restrictive and thus anticompetitive tendencies. This is so whether the restriction on competition was direct or indirect.

 

      1. Payment on Dissolution
        1. Jewel v. Boxer (Cal. 1984).
          1. This case involved the dissolution of a partnership (JBE) in which 4 people were partners. They did not have a written agreement specifying what would happen to fees on matters pending when JBE was dissolved.
          2. Holding
            1. In the absence of a partnership agreemebnt, any income generate through the winding up of unfinished business I allocated to the former partners according to their respective interests in the partnership
            2. Prevents partners from fighting over the most valuable clients in anticipation that they might retain those cases wshould the partnership dissolve

 

    1. Superior-Subordinate Relations
      1. MR 5.1(a) – requires managing attys to take reasonable steps to assure the firms lawyers adhere to the rules
      2. MR 5.1(b) -requires supervising atty to do the same for lawyers they supervise
      3. MR 5.1 (c) – a supervising advisor is responsible for violations of a junior lawyer is the supervisor ratifies the conduct or is aware when consequences can be avoided but fails to take action
      4. MR 5.2 – makes clear that rules apply to junior lawyers in full notwithstanding relative lack of exzperiance
        1. 5.2 (b) EXCEPTION – IF YOU RELIED ON SENIOR ATTY’S REASONABLE RESOLUTION OF AN ARGUABLE INSSUE.
        2. THIS WILL STILL NOT EXCUSE MALPRACTIE CLAIMS AGAINST YOU – JUST DISCIPLINARY ACTIONS
      5. Kramer v. Nowack, 908 F. Supp. 1281 (E.D. Pa. (1995) – A FIRM CAN SUE AN ASSOCIATE FOR NEGLIGENCE UNLESS SUPERVISING ATTY AFFIRMED AND RATIFIED.
        1. Can a firm sue an associate for negligence? Yes. An attorney can owe a fiduciary duty both to a client and another attorney or his employer under the law of agency. Supervising attorneys may therefore sue subordinate attorneys for their negligence in representing clients under the generally applicable laws of agency.
        2. HOWEVER, not if supervising atty affirmed and ratified.
      6. Matter of Howes, 940 P.2d 159 (N.M. 1997):It is not the case that a subordinate lawyer who is entitled and obligated to follow the advice given to him by his supervisor is excused from his actions. Rule 5.2 is not intended to immunize attorneys from accountability for their misconduct
        1. A lawyer is not relieved of his or her responsibility for a violation of the rule of professionka conduct just because he or she acted at the direction of a supervisor. Each member of the bar is an officer of the court. His or her first duty is not to the client or senior partner but the administration of justice.

 

  1. CHAPTER 13 – SOME ECONOMICS OF PRACTICE
    1. FUNDS
      1. Never take money that you have no earned – make clear how you earned it – charge only reasonable fees
      2. MR 1.15
        1. Cannot intermingle client funds
        2. Attorney must segregate funds advanced by the client from the attorney’s own funds
        3. Unless the attorney provides some benefit/service in exchange for the fee, the attorney has not earned nay fee and usually cannot treat advanced fee as his property
        4. Client trust account – If client owns it – it may not go in your account, must go in client trust account
        5. If lawyer own it (advanced retainer) – it may not go in trust account because of requirement of random auditing. You must have half sentence of how you have earned it!
      3. Reasoning – not to punish attorneys but to protect the public
        1. There is no exception and attorney’s subjective intent to return money is not an excuse. Even where the misappropriate was the product of severe personal or financial hardship to attorney.
        2. Goal is to preserve the confidence of the public in the integrity/trustworthiness of lawyers.
        3. Having erected a barrier to entry – licensing requirement. Profession felt the need to regulating fees within the market
        4. Justification for these rules: lemons effect – one of the concerns of the financial rules. If A does bad things with money, lawyers in general will get a bad reputation. In order to stop the lemons effect, financial misconduct must be punished severely. This is the most severe set of violations.
      4. Matter of Sather
        1. Facts – Attorney accepted a 20k flat fee in advance for filing a civil rights suite. Declared person bankruptcy and suspended from practice. He acknowledges his client was entitled to a partial refund of fees but failed to refund the fees promptly
        2. Holding
          1. If you don’t do the work it is refundable because it is still the clients money.
          2. The court considers the retainer to be advanced, they will construe the ambiguity against you, unless you have a recitation saying the money is yours on receipt AND you explain what you have done to earn it.
          3. Unless the attorney provides some benefit or service in exchange for the fee the atty has not earned any fees and with a possible exception in very limited circumstances, the attorney cannot treat advanced fees are his or her property.

 

Classic/Engagement/True Retainer Security Retainer Advance Payment
Purpose Secure availability (no work required)

Place client’s case @ top of attorney’s priority list

Present attorney from representing adverse partiesProvide lawyer assurance that money will be there when earnedPay lawyer up front for work to be performed later (may be flat fee or partial feeBelongs ToLawyer, on receiptClient; lawyer takes only when earned.

 

If the client opposing the amount, you leave the opposed amount in trust account.Jurisdictions differ; Lawyer in CA but client under SatherDeposit InFirm or lawyer’s account – NOT TRUST ACCOUNT 1.15(b)Trust account only – 1.15 (c)Jurisdictions differ; advisable to deposit in trust account in all jurisdictions

 

      1. The Third Rail: Taking What You Haven’t Earned
        1. Matter of Warhaftig, 594 A.2d 398 (N.J. 1987)
          1. Facts
            1. Lawyer’s wife gets cancer and son needs therapy
            2. D charged with invading trust account funds by withdrawing anticipated fees in advance of real-estate closings. Warhaftig starts taking fees before the deal close. Documents say you get paid when the deals clothes. Fees are taken in advance of closing and take deal out of escrow account.
            3. Puts money back if they don close – no clients looses a dime.
            4. State bar to access and audit his trust account. Even under hideous pressure – he doesn’t double down. He doesn’t perjure himself, he doesn’t drain all the money and split.
          2. Holding – Violated 1.15
            1. Precedent of In Re Wilson folds for disbarment of an attorney who “knowingly misappropriates his client’s funds. Knowing disapprobation under Wilson consists simply of a lawyer taking a client’s money entrusted to him, knowing that is the client’s money and knowing that the client has not authorized the taking
            2. A lawyers subjective intent (whether borrowing to steals) is irrelevant when determine discipline in a misappropriation case
            3. Here D took funds from his trust account before he had any legal right to those monies and without authorization, therefore= disbarment.
            4. Keep over head low so that you don’t end up needing money earlier than it is yours. Difficult to keep overhead low. You need to be able to have flexibility in finances.

 

        1. Cash Flow, Cost Structures, Ethical Problems
          1. Such rules therefore do NOT govern the initial fee negotiations
            1. Disciplinary and fiduciary rules governing transaction between lawyer and clients do NOT apply to negotiations between parties over whether to form an atty-client relationship
            2. Because the fiduciary relation is the product of the relationship, it cannot precede it, at least not unless the lawyer has taken steps to assume such duties before reaching a formal agreement
            3. Agreement reached at the beginning of representation within a reasonable time thereafter are enforceable as made.
          2. Any modification beyond a reasonable time – presumptively fraudulent – voidable at the option of the client unless the lawyer can show that the agreement and the circumstances in which it was made were fair and reasonable to client. Restatement 18(1), 38(1).
            1. Maksym v Loecha
              1. Facts
                1. Case to defend a will, client signed a fee agreement that would pay the atty a percentage of the FMV of the estate and for additionally services would be paid 100.hr on top of percentage.
                2. Client ended up firing atty because she thought he moved to slow and refused to pay the balance of his fee claiming it was “presumptively fraudlent”
              2. Holding
                1. The purpose of the “presumptively fraudulent” rule is protect the reasonable expectations of the person who reposes confidence in an agent who, has been led to believe, will treat his affairs with the same solicitude with which the agent would treats the agent’s own affairs.
                2. The creation of these expecation is not instantaneous, and we may assume therefore that a retainer agreement signed shortly after the firt meeting between the parties is not subject to the rule of presumptive imprpreity.
                3. A different rule would have occurred is agreement is signed months later, after the fidcuairy relationship has flowed without benefit of a written agreement.

 

    1. REASONABILITY OF FEES
      1. MR 1.5
        1. (a) – atty may not charge or collect an unreasonable fee.
          1. CAC – Fees must not be unconscionable
        2. 8 factors to assess reasonability
          1. (1) the time and labor required, the novelty and difficulty of the questions involved, and the skill requisite to perform the legal service properly;
          2. (2) the likelihood, if apparent to the client, that the acceptance of the particular employment will preclude other employment by the lawyer;
          3. (3) the fee customarily charged in the locality for similar legal services;
          4. (4) the amount involved and the results obtained;
          5. (5) the time limitations imposed by the client or by the circumstances;
          6. (6) the nature and length of the professional relationship with the client;
          7. (7) the experience, reputation, and ability of the lawyer or lawyers performing the services; and
          8. (8) whether the fee is fixed or contingent.
        3. (b) The scope of the representation and the basis or rate of the fee and expenses for which the client will be responsible shall be communicated to the client, preferably in writing, before or within a reasonable time after commencing the representation, except when the lawyer will charge a regularly represented client on the same basis or rate. Any changes in the basis or rate of the fee or expenses shall also be communicated to the client.
        4. (c) and (d) – allow contingent fees except in divroce or criminal matters and requires that the fee agreement be written and signed by client and that client informed of how costs will be dealt with under the garement

 

      1. As long as an agreement exists between a client and atty to bill a reasonable rate multipled by number of hour actually worked, the attorney’s fee is within a “safe harbor” and proected from a chlallenge that the fee was clearly excessive.
      2. Matter of Fordham, 423 Mass. 481 (1996) – CANNOT REASONABLY CHARGE YOUR CLIENT FOR THE TIME YOU TOOK TO EDUCATE YOURSELF ON A SUBJECT OF LAW.
        1. Facts
          1. Lawyer agreed to take a DUI case but has to learn how to successfully argue one. He wins by arguing a breathlizer technicality
          2. Attorney charged 50,000 and client refuses to pay
        2. Holding
          1. Fee is unreasonable. Other lawyers – expert witnesses testified that they had never heard of a fee that high for that kind of case. They testify to much lower prices.
          2. Here the attorney had no experience in the area and it took him more time to prepare the case. As a result he had to educate himself but you cant charge client for tuition.
          3. A client cannot be expected to pay for a lawyers time for tasks that if the lawyer had a reasonable amount of experience would just be routine.
          4. The client here did not understand the implications of the lawyer’s lack of experience, even though the lawyer told him about it. The attorney was therefore sanctioned.
          5. A lawyer generally should nto accept employment in any area of the law in which he is not qualified. However, he may accept such employmenet if in good faith he expects to become qualified through study and investigation, as long as his preparetion would not result in unreasobale delay or expense to his client.
          6. Client consent is NOT a defense to unreasonable fee
          7. Time records are not protected, you can be deposed on them.

 

      1. Sanction: ABA Model Standards for Imposing Lawyer Sanctions §7.3 endorses a public reprimand as the appropriate sanction for chanrging a clearly excessive fee.
      2. Ratification: Some cases hold that ratification of a fee is possible with full disclosure if the parties perform the agreement over a long period of time. But Fordham implied that client consent is not a defense to an unreasonable fee.
      3. Ex post or ex ante – An agreement that starts out reasonable but then becomes unreasonable will not be upheld. (Holmes v. Loveless (Wash. 2004).
      4. In the matter of O. Doyle Martin (SC 2007). Falsely attributing work done on one matter to another matter would be unacceptable even for an individual client. Though in that cause there might be no risk of misleading the client, the client might use the false records to mislead others.
      5. Fee Agreement Formalities
        1. Model rule 1.5(b) expresses a preference for written fee agreements but does not require them.
        2. Model rule 1.5(c), however, requires that contingent fee agreements be in writing and be signed by the client.
        3. Similarly, if a fee modification were treated as a transaction with a client, Model rule 1.8(a)(1) would require a writing.

 

      1. Division of Fees Among Lawyers
        1. Model Rule 1.5(e) – THIS ONLY APPLIES TO LAWYERS NOT IN THE SAME FIRM
          1. If it ok to refer a client to another lawyer who can actually represent them. You can divide fees if it is
            1. Equal to what was performed or each lawyer assumes joint responsibility for the representation
            2. Client approves = to the arrangement, including the share each lawyer will receive and the agreement is confirmed in writing
            3. Total fee is reasonable
        2. Cal. R. Prof. Conduct 2-200 – California – client must affirmatively consent in writing to the division of fees
        3. Sexual harassment cost baker and McKenzie 8.3 million dollars. – this was for putting up for an asshole partner. When dealing with people like this, they are liability risks.
        4. Chambers v. Kay, 29 Cal.4th 142 (2002)
          1. Facts – Attorneys had separate law practices but came together to serve as co-counsel in a sexual harassment action. Dispute arises over settlement, attorneys split up but sign/agree to a split of the attorney’s fees without client’s consent.
          2. Holding
            1. No in a firm, partnership or association and therefore client’s consent in writing was requires after a fill disclosure had been made in writing that a division of fees will be made and the terms of such divison.
            2. Since not members of the same firm and no client consent – the lawyer cannot share/split the fees.
            3. Just as a client has a right to know how his or her atty’s fees will be determined, he or she also has a right to know the extent of and the basis for the sharing so such fees by attorneys.
            4. THE INVALIDITY TO FEE DIVISON AGREEMENT DOES NO FORECLOSE RECOVERY.
              1. Look at hourly rate and how much time
              2. The terms of the agreement may not be the basis for calculating the amount of equantum meruit recover because this would grant by the back door of requirty relief that is denied through the front door of contract.
              3. Small slips – (just not doing it in writing) – will result in big losses.

 

      1. (The Prohibition On) Selling Yourself Off
        1. Model Rule 5.6(b)A lawyer shall not participate in offering or making an agreement in which a restriction on the lawyer’s right to practice is part of the settlement of a client controversy.
        2. MR 5.6(b) forbids you from agreeing to restrict your practice as part of a settlement on behalf of your client. This issue arises often in mass tort and class actions
        3. Offering or accepting such a payment is a serious violation of the rule and can lead to disbarment
        4. Cal. R. Prof. Conduct 1-500 – Same as above
        5. Florida Bar v Rodriguez, 959 So. 2d 150 (Fla 2007)
          1. Facts – Firm was paid 6 million on top of settlement to clients in exchange for the firm’s agreement not to pursue future claims against the opposing corporation.
          2. Holding
            1. By doing this the firm became an agent of the corporation and created a conflict of interest
            2. The atty potentiall harmed public by forming an agreement with the corporation not to pursue future claims against it
            3. Clients cannot propose to opposing counsel the possibility of retaining them after the current litigation ends, since it would be substantially similar matters and a conflict of interest for the attorney.
            4. Violation to offer and accept it. This side deal was wrong
            5. Rationale
              1. Designed to prevent the creation of a conflict of interest between lawyer and client which can emerge if lawyers future practice revenue stream is placed at odd with clients interest in getting current settlement.
              2. Concern that litigation can really create.
            6. Firm is suspend
            7. Person who drafted the unlawful engagement agreement – disbarred
            8. People not involved had to disgorge their incremental profits.
            9. Why weren’t due ponte disciplined – weren’t license in Florida.

 

    1. UPL – UNLISCENSED PRACTICE OF LAW
      1. The Economics of Licensing – Barriers to entry – impose costs on you, costs you have to recover for this to be an economically rational endeavor.
      2. Model Rule 5.5Unauthorized Practice of Law; Multijurisdictional Practice of Law
        1. (a) A lawyer shall not practice law in a jurisdiction in violation of the regulation of the legal profession in that jurisdiction, or assist another in doing so.
        2. (b) A lawyer who is not admitted to practice in this jurisdiction shall not:
          1. (1) except as authorized by these Rules or other law, establish an office or other systematic and continuous presence in this jurisdiction for the practice of law; or
          2. (2) hold out to the public or otherwise represent that the lawyer is admitted to practice law in this jurisdiction
        3. (c) A lawyer admitted in another United States jurisdiction, and not disbarred or suspended from practice in any jurisdiction, may provide legal services on a temporary basis in this jurisdiction that:
          1. (1) are undertaken in association with a lawyer who is admitted to practice in this jurisdiction and who actively participates in the matter;
          2. (2) are in or reasonably related to a pending or potential proceeding before a tribunal in this or another jurisdiction, if the lawyer, or a person the lawyer is assisting, is authorized by law or order to appear in such proceeding or reasonably expects to be so authorized

 

      1. Unauthorized Practice Practice of Law Committee v. Parsons Technology, Inc. – LICENSED PRACTICE OF LAW IS ONLY NEEDED IF YOU AR PRACTICING LAW WHICH RAISE THE QUESTION OF WHA DOES AND DOES NOT CONSTITUTE LEGAL PRACTICE.
        1. Issue: Whether the sale of a software that helps fill out legal firms constitutes the unauthorized practice of law?
          1. There is not logical domain of what involves the “practice of law”
          2. Super competitive profits induce entry
        2. Holding:
          1. It is the unauthorized practice of law. Even though the software has disclaimers.
          2. The software goes beyond merely instructing someone on how to fill out a blank form. Practice does not require personal contact or a lawyer client relationship, nor do the services have to be on behalf of a client. (Note this opinion was later vacated when TX modified its statute).

 

      1. Franklin v. Chavis (SC 2007) – The court held that an insurance agent engaged in the unlicensed practice of law when an elderly acquaintance (and former client) of his asked him to help her draft a will. He put in her responses into Quicken. He was practicing (and acting as more than a scrivener).
        1. THE PREPARATION OF LEGAL DOCUMENTS CONSTITUTES THE PRACTICE OF LAW WHEN SUCH PREPARATION INVOLVES THE GIVING OF ADVICE, CONSULTATION, EXPLANATION OR REOMMENDATIONS ON MATTERS IF OAW.
        2. If the person had done nothing more than record verbatim what the decedent says, no UPL.

 

      1. Your Comparative Advantage and the Thesis of This Course
        1. Two things that I have that non-lawyer do not
          1. Ability to apply a large number of different types of knowledge to solve concrete legal problems.
          2. Habit of mind that can select what is relevant and analyze it dynamically and interactively in relating to some problem that needs to be solved.
          3. =legal judgment

 

      1. Note on UPL and Multi-State Practice
        1. UPL regulations require that a person be licensed to practice in the state in which the rule applies.
        2. Birnbrower, Montalbano, Condon & Frank v. Superior Court (Cal. 1998):You cannot practice law in a state in which you are not licensed. Lawyers practicing law while physically located in a state cannot recover under a fee agreement for such work unless the lawyer is either admitted to the bar of that state or falls within an exception (like pro hac vice status).

 

 

  1. CHAPTER 14 – RELATIONS WITH OTHER LAWYERS
    1. Introduction
      1. Notional reason – Need for self-policing rule
        1. Self-regulating profession – we draft the rules that regulate us. The professions has decided we should do self-policing. This includes having a rule that says you have to do it.
        2. In most states – legislature does not have power to regulate practice of law. This is for judiciary.
        3. California is a state that has held legislative regulation is consultation violating.
        4. Two problems with this
          1. Legislature regulated in a number of ways
          2. In California, not for purposes of exam, there is no 8.3 – no reporting requirement.
          3. However, it is misleading to think of 8.3 or absents just in terms of self-policing
          4. There is an underlying problem – BLACK MAIL. And there are independent rules against black mail in California
      2. MR 8.3
        1. There is a requirement under 8.3 that you report to relevant disciplinary – raises a substantial questions as to fitness as to someone else to practice law
        2. Model Rules 8.3REPORTING PROFESSIONAL MISCONDUCT
          1. (a) A lawyer who knowsthat another lawyer has committed a violation of the Rules of Professional Conduct that raises a substantial question as to that lawyer’s honesty, trustworthiness or fitness as a lawyer in other respects, SHALL inform the appropriate professional authority.
          2. (b) A lawyer who knows that a judge has committed a violation of applicable rules of judicial conduct that raises a substantial question as to the judge’s fitness for office shall inform the appropriate authority
          3. (c) This Rule does not require disclosure of information otherwise protected by Rule 1.6 or information gained by a lawyer or judge while participating in an approved lawyers assistance program.
          4. TRUMPED BY 1.6 – if information is a client confidence – duty of confidentiality and there is not exception then you are not to report
          5. 3.3. trumps 1.6 – 1.6 trumps 8.3
          6. DON’T LIE TO COURT (3.3) > CONFIDENTIALITY (1.6) > REPORTING (8.3)
        3. MR 8.4 – professional misconduct to violate or assist/induce others to violate the rules – engage in dishonest/fraudulent conduct – conduct prejudicial to the administration of justice – or knowingly assists a judge of juridical officer in such conduct.

 

    1. Reporting (And Getting In Trouble By Threatening to Report) Misconduct
        1. Basic rules
          1. Never threaten to report, either do it and shut up
            1. Threats look coercive.
            2. Suspension, disbarment or other discipline is warranted for a threat to report
          2. To be required to report, an atty must have sufficient knowledge before him such that a reasonable lawyer would have formed a firm opinion that the other lawyer had broken a rule and the aty is not require to report alleged misconduct.
      1. In re Himmel, 533 N.E. 2d (Ill. 1988)
        1. Facts
          1. Client came to D telling her that previous lawyer did not give her the 23k she is owed.
          2. D possessed unprivileged knowledge of another atty’s conversation of clients fund, yet did not report the misconduct
          3. Attorney used client information (non privileged) that another attorney had violated a rule by misappropriating funds of the client as a bargaining chip to get the client a larger settlement; the lawyer said if the other lawyer paid the settlement, he would not report him for the violation.
        2. Holding
          1. The court said he had a duty to report the attorney
          2. Subject to discipline, even though the client asked him not to report him.
          3. This failure to repor resulted in interference with the investigation into the atty and therefore interfere with the administration of justice. D chose to draft a settle agreement with atty rather than report his misconduct.
          4. Suspension for 1 year.
          5. Blackamil often signals duty to report but desire to gain profit instead. This should take blackmail off the table because Himmel is not legally allowed to promise silence. This makes it so Ferbsberg wont fire Himmel and get another attorney willing to blackmail Casey!
          6. If it had been barred by 1.6 -Hummel would have had good defense! we teach model rules but states may get around them as here where the require privilege, not confidence (not general known).

 

    1. Claims Between Lawyers
      1. Sub agents – AGENT IS LIABLE FOR SUBB-AGENTS BREACHES.
        1. Agent of client who hires a sub-agent (associate).
        2. Agent is liable for own acts to principles. Agent is also liable for sub-agent. Kind of looks for respondent superior however, this is just hiring someone else and obligation has never left me. I have just chosen to discharge by hiring in some help.
        3. Sub agent is liable to agent. The original lawyer then has a claim for indemnity from the subagent
        4. Whalen v. DeGraff, Foy, Conway, Holt-Harris & Mealey, 863 N.Y.S. 2d 100 (2008)
          1. Facts
            1. This involves sub-agent.
            2. Instead of telling client to find Florida counsel (to get co-agency – not responsible for clients) – found counsel himself
            3. Lawyer retained another lawyer to file a claim but the claim was not filed.
          2. Holding
            1. A firm is nor ordinarily liable for the acts or omissions of a lawyer outside the firm who is working with the firm lawyers as co-counsil or in a similar arrangement, as such a lawyer is usually an independent co-agent of the client
            2. However, if a lawyer solicits and obtained another lawyer without the client’s knowledge and the client does not enter into a retainer agreement with the second firm, second firm becomes a sub=agent of original firm and the original firm has duty to pervice the second firm or atty.

 

        1. Musser v. Provencher, 28 Cal. 4th 274 (2002)
          1. Facts
            1. Lawyer calls friend and friend gives wrong advice. As a result lawyer is sued by client. Lawyer never told client about 3rd person giving advise
          2. Holding
            1. These were sub-agency!
            2. Would have been co-agency if A had just told client she needed to obtain another counsel.
            3. While first lawyer can be sued by client.
            4. Lawyer can also sue second lawyer he relied. Lawyer can sue (1) concurrent counsel (2) for indemnity. This is not a duty of loyalty problem, it is duty of care. If you create liability lawyer/lawyer you might create a situation where lawyer will worry about liability to another lawyer and that worry impairs the representation of the client.

 

      1. Co-agents – Co-counsel – chosen with clients knowledge – NOT RESPONSBILITY FOR EACH OTHER
        1. No liable for each other’s torts or malpractice
        2. Beck v. Wecht, 28 Cal. 4th 289 (2002) – Co-Counsel
          1. Facts: One attorney failed to call to accept settlement so clients lost the case for personal injury. Lawyers had a contingent fee arrangement.
          2. Holding
            1. A lawyer may NOT sue co-counsel for breach of fiduciary duty on the theory that co-counsel’s negligence simply cost the first lawyer money
            2. There is no fiduciary duty to protect one another’s prospective interests in a contingency fee.

 

      1. Original attorney cannot sue subsequent atty’s who replace them for breaches of fiduciary duty.

 

 

  1. CHAPTER 15 – ETHICS IN ADVOCACY
    1. Introducing Perjured Testimony
      1. Model Rule 3.3 – CANDOR TOWARD THE TRIBUNAL
        1. (a) A lawyer shall not knowingly:
          1. (1) make a false statement of fact or law to a tribunal or fail to correct a false statement of material fact or law previously made to the tribunal by the lawyer;
            1. HOWEVER YOU MAY ASK A JURY TO MAKE A LOGICAL INFERENCE THAT YOU KNOW TO BE FALSE
          2. (2) fail to disclose to the tribunal legal authority in the controlling jurisdiction known to the lawyer to be directly adverse to the position of the client and not disclosed by opposing counsel; or
          3. (3) offer evidence that the lawyer knows to be false. If a lawyer, the lawyer’s client, or a witness called by the lawyer, has offered material evidence and the lawyer comes to know of its falsity, the lawyer shall take reasonable remedial measures, including, if necessary, disclosure to the tribunal. A lawyer may refuse to offer evidence, other than the testimony of a defendant in a criminal matter, that the lawyer reasonably believes is false.
      2. Restatement §120False Testimony Or Evidence
        1. (1) A lawyer may not:
          1. (a) knowingly counsel or assist a witness to testify falsely or otherwise to offer false evidence;
          2. (b) knowingly make a false statement of fact to the tribunal;
          3. (c) offer testimony or other evidence as to an issue of fact known by the lawyer to be false.
        2. (2) If a lawyer has offered testimony or other evidence as to a material issue of fact and comes to know of its falsity, the lawyer must take reasonable remedial measures and may disclose confidential client information when necessary to take such a measure.
        3. (3) A lawyer may refuse to offer testimony or other evidence that the lawyer reasonably believes is false, even if the lawyer does not know it to be false.
      3. California is 6068(d).

 

 

      1. Chart Over View
        1. No reason to suspect falsity – No problem – REMEDY IF YOU LEARN OF FALSITY 3.3(a)(3)
        2. You know testimony is false
          1. Civil – CANNOT preset testimony. You should informed client of your rule 3.3 duties and urge truthfulness or cooperation in remedying falsity. If relationship then deteriorates, possible withdraw.
          2. Criminal – CANNOT present testimony UNLESS state law differs from Rule 3.3. Some states allow “narrative approach.” You should informed client of your rule 3.3 duties and urge truthfulness or cooperation in remedying falsity. If relationship then deteriorates, possible withdraw.
          3. THIS IS THE SITUATION WERE AVOIDING KNOWLEDGE IS SMART. This portion of the chart has nothing to do with you if you avoid finding something out.
        3. You reasonable believe testimony will be false
          1. Civil – LAWYERS DISCRETION – REMEDIATION OBLIGATION APPLIES.
          2. Criminal – YOU MUST PRESENT IF CLIENT INSISTS but should counsel him on the rules of perjury – REMEDIATION OBLIGATION APPLIES.
        4. The rule requiring you to do something to correct false testimony applies to pre-trial perjury as well as trial testimony.
        5. The rule also trumps the disciplinary rule requiring that you preserve client confidences. Requirement of Remediation trumps confidentiality requirement of rule 1.6

 

      1. The prohibition on offering false testimony and use of “narrative” – People v. Johnson, 62 Cal. App. 4th 608 (1998)
        1. Facts
          1. Defense counsel knew his client was going to lie on stand, so he refuses to call him as a witness even though D wanted to go on stant. D is convicted, appeals claiming he was denied his constitional right to testify.
        2. Holding
          1. Criminal – Cannot present testimony unless state law differs from Rule 3.3. Some states allow “narrative approach.” You should informed client of your rule 3.3 duties and urge truthfulness or cooperation in remedying falsity. If relationship then deteriorates, possible withdraw.
          2. ABA IN COMMENT TO 3.3 – IN A JURISDCTION WHICH ALLOWS NARRATIVE, NOT A VIOLATION. YOU DO NOT ARGUE TO JUDGE OR JURY PERJURED TESTIMONY. STILL DO NOT ASSERT FALSITY. PROSEUCTION CAN NOT EXPLICITLY RESPOND ON THE FACT THAT IT IS “NARRATIVE”
          3. California – we have a system in where we want truth but we respect D right to be heard but D is undermining process. YOU STAND UP – MY CLIENT WILL TESTIFY IN THE NARRATIVE.
          4. YOU CANNOT SAY – MY CLIENT WILL NOW PERJURY – BC OF DUTY OF CARE.
          5. Does this complicate search for truth? Does its best to undermine, but prosecution – duty it to undercover
        3. State v McDowell – Washington SC – an atty may not substitute narrative questioning for the traiditiona question and answer forma unless counsel knows the cient intends to testify falsely.
        4. People v Bolton
          1. Lawyer thought client planned to perjure himself and asked to be relieve based on conflict of interest. Atty was relieved and client decided to represent himself. When client was convicted he appeals on the ground that his waiver of the right to counsel was coerced by decision to excuse atty.
          2. Holding
            1. Atty should have allowed client to testify in narrative
            2. Atty did not even know that his client’s testimony was fact – he simply doubted it, an attorney may ethically present evidenc that they suspect but do not personally know is false.

 

      1. The Obligation to correct
        1. Comment 10 –Rule 303 – when you learn you have offered false evidence you should talk to the client confidentiality, urge that the client withdraw or crrect the false testimony and advise the client of your duty of candor to the court and your corresping obligation to remedy the false testimony. If not ne of that gains clients cooperation, you are “to make such dislocusre to the tribunal as is reasonably necessary to remedy the situation. Even if requires disclosure probhibited by 1.6
        2. Idaho State Bar v. Warrick, 44 P.3d 1141 (2002) – YOU MUST ACT AS SOON AS PERJURY OCCURS
          1. Facts: Attorney Warrick charged for allowing false testimony to be presented without taking any remedial measures to correct the testimony at the time the testimony was presented.
          2. Holding
            1. If a lawyer allows a false testimony to be introduced without taking any measures to correct the testimony AT THE TIME when the testimony is presented, they are in violation of MR 3.3 and can be suspended from the practice of law. Must correct!
            2. The lawyer must take remedial measures at the time he comes to know of the falsity, not the next day or before the end of the trial. Warrick was therefore suspended from the practice of law for 30 days.
            3. Attorney should have remedies the perjury as soon as it occurs.
            4. This applies to pre-trial as well as trial testimony
            5. Trumps rule fo confidentiality (3.3>1.6)

 

        1. In re Disciplinary Proceeding against John E. Mack (1994) – 3.3(a)(3)’s requirement that you remedy false testimony applies to pre=trial perjury as wel as trial testimony. If your client lied pre-trial, you should correct ht perjury not later than on direct.
          1. Facts: Client claiming insurance for an accident lied about when she wrote a check in payment of a premium. She told her lawyer that she had lied and he did not tell her to correct her testimony.
          2. Holding: The lawyer was disciplined under Rule 3.3 even though he did not introduce the false testimony at trial. If a lawyer discovers that his client has lied to the court, he must urge the client to correct the statement; if that doesn’t work he must seek to withdraw; finally, if that fails he must correct the misrepresentation himself, even if it means revealing information that would be protected under Rule 1.6. The remedial action must be taken quickly.

 

      1. Client perjury and the Strickland Standard
        1. You have to report your client is they perjury
        2. Under Strickland v. Washington (US 1984) – an ineffective assistance of counsel claim requires a client to show that (1) the lawyer’s acts violated the standard of care followed by ordinarily competent criminal defense attorneys and (2) that there was reasonable probability that, but for counsel’s actions, the defendant would have had a better result at trial.
          1. If a client brings this claim for telling the truth where a lawyer tells the client not to lie, the client will not be successful.
          2. If a client wants to perjure himself and client tells him not to and then he loses, the client will not succeed on an ineffective assistance of counsel claim.
        3. Nix v. Whiteside (US 1986) – An attorney told a client not to perjure himself or he would withdraw. Client claims ineffective assistance of counsel.
          1. Holding: A refusal to allow perjury is not a deviation from professional norms.
        4. People v. Johnson, 62 Cal. App. 4th 608 (1998)In CA, exception to the rule from Nix v. Whiteside—testimony in the narrative: in court you tell judge that client will be testifying in narrative. Not on exam.

 

      1. Candor Toward A Tribunal
        1. 3.3(a)(1) – NEVER LIE TO COURTS – cannot make false statement of face or law to a tribunal and require to you correct false statement you previously made
        2. People v. Casey, 948 P.2d 1014 (Colo. 1997)
          1. Facts: Daughter has an id that belongs to someone else. She gives the fake id to the police and charged with underage drinking.
          2. Holding:
            1. Since the atty engaged in a “course of knowing conduct over a appreciable period of time, resulting in false and material information being submitted to the court.” A short suspension and requiring the atty to pass the MPRE is appropriate.
            2. You cannot appear in court representing the wrong name. You cannot go to court and say you represent a wrong name. You don’t want to violate 3.3 ever! Tribunals hate this.

 

      1. Handling evidence
        1. Model Rule 3.4 – FAIRNESS TO OPPOSING PARTY AND COUNSELA lawyer shall not:
          1. (a) unlawfully obstruct another party’s access to evidence or unlawfully alter, destroy or conceal a document or other material having potential evidentiary value. A lawyer shall not counsel or assist another person to do any such act;
          2. (b) falsify evidence, counsel or assist a witness to testify falsely, or offer an inducement to a witness that is prohibited by law;
          3. (c) knowingly disobey an obligation under the rules of a tribunal, except for an open refusal based on an assertion that no valid obligation exists;
          4. (d) in pretrial procedure, make a frivolous discovery request or fail to make reasonably diligent effort to comply with a legally proper discovery request by an opposing party;
          5. (e) in trial, allude to any matter that the lawyer does not reasonably believe is relevant or that will not be supported by admissible evidence, assert personal knowledge of facts in issue except when testifying as a witness, or state a personal opinion as to the justness of a cause, the credibility of a witness, the culpability of a civil litigant or the guilt or innocence of an accused; or
          6. (f) request a person other than a client to refrain from voluntarily giving relevant information to another party unless:
            1. (1) the person is a relative or an employee or other agent of a client; and
              (2) the lawyer reasonably believes that the person’s interests will not be adversely affected by refraining from giving such information.
        2. Restatement §119Physical Evidence Of A Client Crime
          1. With respect to physical evidence of a client crime, a lawyer:
          2. (1) may, when reasonably necessary for purposes of the representation, take possession of the evidence and retain it for the time reasonably necessary to examine it and subject it to tests that do not alter or destroy material characteristics of the evidence; but
          3. (2) following possession under Subsection (1), the lawyer must notify prosecuting authorities of the lawyer’s possession of the evidence or turn the evidence over to them.
        3. People v. Meredith, 29 Cal. 3d 682 (1981)
          1. Facts
            1. Investigator for D counsel took victim’s walet. Divided the money with D, then attempted to burn and dispose of it.
            2. Prosecution moved to have the existence/location of the wallet disclosed, however D counsel objected on grounds of atty-client privilege
          2. Holding
            1. An observation by D counsel or his investigator, which is the product of a privileged communication, may NOT be admitted UNLESS the defense by altering or removing physical evidence has precluded the prosecution from making the same observation
            2. You can take it and hand it over to prosecution or
            3. You can leave it where it is and say nothing but run risk it will be found during the investigation
            4. If your clients bring you an illegal product, you may dispose of it immediately (but not if it is evidence of a separate crime because then it would be considered obstruction of justice) or tell client to get ride of it.
            5. If you move it, you can be called on to testify; if you leave it there you cannot. So look at it, and do nothing, you are immune. Alter it, and you are not. If you alter the evidence, you don’t want that. The court suggests that you should just stipulate to where the thing was found. If you move or alter the evidence, then you can be put on stand.
            6. Testing: you can retain evidence for enough time as it takes to test it (the tests must be nondestructive). So it is not impermissible to move and inspect—but it does create some obligations to disclose.

 

      1. Discovery Conduct
        1. Model Rule 3.4(d)-you may not make a frivolous discovery request or fail to make reasonably diligent efforts to comply with a proper request.
        2. If you’re a litigator, the more you can’t not to give a doc/record of any kind, the more important it is that you do so.
        3. Redwood v. Dobson, 476 F3d 462 (7th Cir. 2007) – YOU CAN CONTROL YOUR SIDE OF THE ROOM
          1. Racial slurs and battery in deposition. In response the opposing counsel tells his client not to answer any questions.
          2. Holding
            1. Under FRCP 30(d) counsel for a witness may halt a deposition and apply for a protective order but may NOT instruct the witness to remain silent. Any objection during deposition must be stated concisely and in a non-argumentative and non-suggestive manner for the record.
            2. A counsel may order a deponent not to answer ONLY when necessary to preserve a privileged, to enforce a limitation directed by the ocourt or to present a motion
            3. Mutual enmity does not excuse the breakdown of decorum that occurred in the deposition. All the attorneys are sanctioned for conduct unbecoming of members of the bar.
          3. Tactics
            1. If you do this right, there are no rules.
            2. Move discovery back to the merits! Don’t allow the opposing counsel to control the temperature
            3. RECOGNIED YOU ARE BEING GAMED – If this happens, tell yourself “I am being gamed”This is not about me, this is a tactic and I win by not letting it work. You are not going to win by being quicker on the draw by a local costum. Don’t look for a shooting match.
            4. Don’t argue or get distracted.
            5. Unless it is the end of the day and you have no other choice, slow down! Give youself time to think and let shock dissipate. Don’t show vulnerability (crumbling or aggressive).
            6. Litigators – do not talk to the person gaming you, talk to the witness. Do not talk to your antagonist, that is what they want.
            7. Unless you are about to die, don’t take a break.

 

        1. Witness Coaching
          1. MR 3.4(b) – You may not counsel a witness to testify falsely or assist a witness in doing so
          2. Overt coaching in depos is sanctionable. Hard to say what coaching is. Also, hard to discover it.
          3. Real world preparation for deposition includes coach – suggesting to witness that they limit their answers to the extent possible and not egnange In open-ended dialogue. Also explain what is a helpful or unhelpful answer.
          4. No one is really disciplined but may not have a good outcome
          5. Not transparent – not too obvious

 

        1. Sanctions for failure to provide discovery
          1. Companies can be sanctioned. In many cases the court will then ask the lawyers to show cause why they should not be personally sanctioned for the discovery misconduct.
          2. Does get too cute or try to be too clever about client’s discovery obligations
          3. If you are a litigator, transactional person dealing with financial records, the more you want not to give a document or record to opposing party – the more important it is that you do so. Your reluctance is a good guide to whether you need to
            1. The more is killed you – the more you should do it
          4. Washington State Phys Ins. Exchg. & Assoc. v. Fisons Corp., 858 P.2d 1054 (WA 1993)
            1. Facts –
              1. Fisons produced drug that caused seizures in a little girl that led to brain damage. When asked to produce documents they withheld some important documents.
              2. Plaintiff asked drug company for all documents pertaining to their product.
              3. The drug company objects to request and says their no good. They are too broad, harassing, not reasonably calculated to lead to discovery. The company and lawyer objected to some of the document on the groun that they were about the ective ingredient in the drug, rather than the brand name itself. We will produce documents pertaining to active ingredients, not the drug itself. – redefines request.
              4. Action gets approve by law firm’s actions.
              5. P says, hey you didn’t give us this document!
            2. Holding
              1. These rules are obligations! You don’t get to contract these away. Pulling wool over the eyes over P’s eyes does not matter, you had a requirement under the rules.
              2. Although fair and reasoned resistance to discovery requests is not sanctionable, the drug company’s responses were misleading and contrary to the purposes of discovery – had earlier RPDs been complied w/ the docs would have been revealed much earlier in the litigation
              3. Company unnecessarily distinguished b/w the drug and the active ingredient which was not a viable distinction b/c the only reason it was made was to hide the incriminating documents
              4. Firm got heavy sanctions.
              5. If a party uses misleading or non-responses to request for discovery that would reasonably lead to the production of incriminating evidence, sanction is appropriate.
              6. Law firm is indicted as suppressing evidence. Law firm and company fined.
            3. 2 main differences between this and Redwood
              1. The redefinition move was transparent in form but not transparent in substance it gave the appearance of compliance while in fact serving the purpose of withholding some very relevant documents
              2. The high materiality of the documents being held. If the documents had been innocuous, this never would have turned into an opinion.

 

    1. Litigation privilege and the scope of legitimate demands
      1. Cal. Civ. Code §47Privileged publication or broadcast- Absolute privilege, bars all tort actions except for claim of malicious prosecution. Purpose is to afford litigants and witnesses the utmost freedom of accesss to the courts w/o fear of being harassed subsequently by derivative tort actions..

 

      1. Cal. Code Civ. P. 425.16 – Anti-SLAPP actions; motion to strike; discovery; remedies
        1. Rights of free speech and petition from the abuse of the judicial process by allowing a D in a defamation suit to bring a motion to strike any action that arises from any activity of the D that is constitutionally protected.
        2. Purpose is to protect the valid exercise of const.
        3. 2 step
          1. Prove the activity is constitutionally protected
          2. That they are likely to succeed on any claim they bring
          3. If not shown – stricken and fee award are mandatory

 

      1. Flatley v. Mauro, 39 Cal. 4th 299 (2006) – WHAT IS EXTORTION?
        1. Anti-SLAPP statute—very protective of privilege. This case is an example of how to commit a crime using your letterhead. The crime at issue is extortion.
        2. Facts:
          1. Flatley is an entertainer, inventing river dance
          2. He either assault or willingly has sex. Leaves and hired lawyer
          3. Flatley gets letter (sent to his attorney [42-45]) from Muaro
          4. Substance is that Flatley had sexual relations with client against her will and that if he doesn’t pay them money, they will file lawsuit for rape and follow him to every town where dance troupe performs and publicize what happened. It will become a matter of public record.
          5. Sending the letter is an important speech act, meaning these speech acts are deeds. Extortion and blackmail,
          6. Mauro files an anti-SLAPP motion – California 425.16.
        3. Holding
          1. Court concluded the letter and phone calls constituted criminal extortion.
          2. Because it was extortion, it is not protected under 426.15, and the court did not error in denying att’s motion to strike the man’s defamation claim
          3. But there is a substantive protection in civil code 47. It immunizes from most torts any conduct that occurs in litigation. The privilege could bar the substantive suit. You don’t want defendants coming back and suing plaintiffs for extortion based on settlement conferences. The line between extortion and aggressive lawyering is not that clear. Prefiling claims are ambiguous in terms of how far back in time §47 extends.
          4. Mauro gets suspended from the practice of law by virtue of committing extortion

 

      1. Sussman v. Bank of Israel-Another case involving a demand letter, in this case it was not extortion. This letter was more polite, more decorous, more oblique, and those elements help differentiate his letter from the letter in Flately.

 

    1. Abuse of Process
      1. Seltzer v. Morton, 154 P.3d 561 (Montana 2007)
        1. Facts:
          1. Seltzer is a professional art appraiser.
          2. At the request of an auction house, he rendered his opinion as to the authenticity of a painting that Morton owned.
          3. Morton demanded that Seltzer recant his opinion.
          4. He refused and Gibson Dunn filed a lawsuit against Seltzer.
        2. Holding:
          1. Standard for abuse of process is (1) an ulterior purpose and (2) a willful act in the use of the process not proper in the regular conduct of the proceeding. GD used the suit as an instrument of coercion rather than a legitimate means to solve a genuine dispute.
          2. They used the lawsuit to try and get seltzer to recant his opinion. You can’t sue to get someone to recant an opinion. There is a countersuit for abuse of process and the law firm lost to the tune of $16million. 

Discover more from Legal Three

Subscribe now to keep reading and get access to the full archive.

Continue reading